Vous êtes sur la page 1sur 90

CRIMINAL PROCEDURE

JURISDICTION

HANNAH SERRANA VS SANDIGANBAYAN

DOCTRINE:
​It is not only the salary grade that determines the jurisdiction of the Sandiganbayan—the
Sandiganbayan also has jurisdiction over other officers enumerated in P.D. No. 1606.

FACTS:
​Serana was a senior student of UP-Cebu who was also appointed by Pres. Estrada as
student regent of UP to serve a one-year term from Jan.1, 2000 to Dec. 31, 2000. On Sept. 2000,
petitioner together with her siblings and relatives, registered with the SEC the Office of the
Student Regent Foundation, Inc (OSFRI). On of the projects of the OSFRI was the renovation of
Vinzons Hall in UP Diliman, and Pres. Estrada gave P15M as financial assistance for the said
project. The source of funds, according to the information, was the Office of the President.
However, the renovation failed to materialize. The succeeding student regent and system-wide
alliances of students conseguently filed a complaint for Malversation of Public Funds and
Property with the Ombudsman. After due investigation, the Ombudsman instituted a criminal
case against Serana and her brother, charging them of Estafa.
Serana moved to quash the Information, contending that the Sandiganbayan does not have
jurisdiction over the offense charged nor over her person in her capacity as UP student regent.
She contends that Estafa falls under Crimes Against Property and not on the chapter on Crimes
Committed by Public Officers, only over which, she argues, the Sandiganbayan has jurisdiction.
Furthermore, she argues that it was not the governement that was duped, but Pres. Estrada,
because the money came from the Office of the President and not from government funds. As to
jurisdiction over her person, she contends that as a UP student regent, she is not a public officer
since she merely represents her peers, in contrast to the other regents who held their positions in
an ex officio capacity.
​The Sandiganbayan denied her motion for lack of merit.

ISSUE:
​Whether or not Sandiganbayan has jurisdiction over the offense charged and over Serana

HELD:
​YES. Jurisdiction of Sandiganbayan; Crime of Estafa. Plainly, estafa is one of those
felonies within the jurisdiction of the Sandiganbayan, subject to the twin requirements that: 1)
the offense is committed by public officials and employees mentioned in Section 4(A) of PD No.
1606, as amended, and that; 2) The offense is committed in relation to their office. It is well-
established that compensation is not an essential element of public office. At most, it is merely
incidental to the public office. Delegation of sovereign functions of the government, to be
exercised by him for the benefit of the public makes one a public officer.
A UP Student Regent is a Public Officer. A public office is the right, authority, and duty created
and conferred by law, by which for a given period, either fixed or enduring at the pleasure of the
power, an individual is interested with some portion of sovereign functions of the government, to
be exercised by him for the benefit of the public.
​Jurisdiction of the Sandiganbayan covers Board of Regents. The Sandiganbayan, also has
jurisdiction over the other officers enumerated in PD No. 1606. In Geduspan v. People, the SC
held that while the first part of Sec. 4(A) covers only officials with Salary grade 27 and higher
but who are by express provisions of law placed under the jurisdiction of the Sandiganbayan as
she is placed there by express provisions of law. Sec. 4(A)(1)(g) of PD No. 1606 explicitly
vested the Sandiganbayan with jurisdiction over Presidents, directors and trustees, or manager of
government-owned or controlled corporations, state universities, or educational foundations.
Petitioner falls under this category. As the Sandiganbayan pointed out, the Board of Regents
performs functions similar to those of a board of trustee of a non-stock corporation. By express
mandate of law, petitioner is, indeed, a public officer as contemplated by PD No. 1606.

GARCIA VS SANDIGANBAYAN

DOCTRINE:
​The Sandiganbayan has jurisdiction over actions for forfeiture under Republic Act No.
1379, albeit the proceeding thereunder is civil in nature—the civil liability for forfeiture cases
does not arise from the commission of a criminal offense.

FACTS:
​To recover unlawfully acquired funds and properties that the Garcias’ had allegedly
amassed and acquired, the Republic, through the OMB filed with the SB 2 petitions for the
forfeiture of those properties. The Garcias’ filed motion to dismiss on the ground of SB’s lack of
jurisdiction for lack of proper and valid service of summons: (1) Forfeiture I – the corresponding
summons on the case were all issued and all served on Gen. Garcia at his place of detention; (2)
Forfeiture II – the sheriff stated giving the copies of the summons to the OIC/Custodian of the
PNP Detention Center who in turn handed them to Gen. Garcia.
​The general signed his receipt of the summons, but as to those pertaining to the other
respondents, Gen. Garcia acknowledged receiving the same, but with the following qualifying
note: Im receiving the copies of Clarita, Ian Carl, Juan Paolo & Timothy but these copies will not
guarantee it being served to the above-named.

ISSUE:
​Whether the Sandiganbayan acquired jurisdiction on the person of Clarita Garcia and her
three sons, considering that summons were improperly served and that the plunder case (Crim.
Case No. 28107) has already been filed and pending with another division of the SB, i.e., Second
Division of the SB.

HELD:
​Forfeiture Cases and the Plunder Case Have Separate Causes of Action; the Former Is
Civil in Nature while the Latter Is Criminal
​A forfeiture case under RA 1379 arises out of a cause of action separate and different
from a plunder case, thus negating the notion that the crime of plunder charged in Crim. Case
No. 28107 absorbs the forfeiture cases. In a prosecution for plunder, what is sought to be
established is the commission of the criminal acts in furtherance of the acquisition of ill-gotten
wealth. In the language of Sec. 4 of RA 7080, for purposes of establishing the crime of plunder,
it is sufficient to establish beyond reasonable doubt a pattern of overt or criminal acts indicative
of the overall unlawful scheme or conspiracy [to amass, accumulate or acquire ill-gotten wealth].
On the other hand, all that the court needs to determine, by preponderance of evidence, under RA
1379 is the disproportion of respondents properties to his legitimate income, it being unnecessary
to prove how he acquired said properties. As correctly formulated by the Solicitor General, the
forfeitable nature of the properties under the provisions of RA 1379 does not proceed from a
determination of a specific overt act committed by the respondent public officer leading to the
acquisition of the illegal wealth.
​On the issue of lack of jurisdiction, petitioner argues that the SB did not acquire
jurisdiction over her person and that of her children due to a defective substituted service of
summons. There is merit in petitioners contention.
​Sec. 7, Rule 14 of the 1997 Revised Rules of Civil Procedure clearly provides for the
requirements of a valid substituted service of summons, thus:
​SEC. 7. Substituted service.If the defendant cannot be served within a reasonable time as
provided in the preceding section [personal service on defendant], service may be effected (a) by
leaving copies of the summons at the defendants residence with some person of suitable age and
discretion then residing therein, or (b) by leaving the copies at defendants office or regular place
of business with some competent person in charge thereof.
​It is basic that a court must acquire jurisdiction over a party for the latter to be bound by
its decision or orders. Valid service of summons, by whatever mode authorized by and proper
under the Rules, is the means by which a court acquires jurisdiction over a person.
​In the instant case, it is undisputed that summons for Forfeitures I and II were served
personally on Maj. Gen. Carlos Flores Garcia, who is detained at the PNP Detention Center, who
acknowledged receipt thereof by affixing his signature. It is also undisputed that substituted
service of summons for both Forfeitures I and II were made on petitioner and her children
through Maj. Gen. Garcia at the PNP Detention Center. However, such substituted services of
summons were invalid for being irregular and defective.

Requirements as laid down in Manotoc vs CA


​1. ​Impossibility of prompt personal service, i.e., the party relying on substituted
service or the sheriff must show that defendant cannot be served promptly or there is
impossibility of prompt service within a reasonable time. Reasonable time being “so much time
as is necessary under the circumstances for a reasonably prudent and diligent man to do,
conveniently, what the contract or duty requires that should be done, having a regard for the
rights and possibility of loss, if any[,] to the other party.” Moreover, we indicated therein that the
sheriff must show several attempts for personal service of at least three (3) times on at least two
(2) different dates.
​2. ​Specific details in the return, i.e., the sheriff must describe in the Return of
Summons the facts and circumstances surrounding the attempted personal service.
​3. ​Substituted service effected on a person of suitable age and discretion residing
at defendant’s house or residence; or on a competent person in charge of defendant’s office or
regular place of business.

ESCOBAL VS GARCHITORENA

DOCTRINE:
​The jurisdiction of the court over criminal cases is determined by the allegations in the
Information or the Complaint and the statute in effect at the time of the commencement of the
action, unless such statute provides for a retroactive application thereof.
​For the Sandiganbayan to have exclusive jurisdiction under Section 4(a) of P.D. No.
1606, as amended by P.D. No. 1861 over crimes committed by public officers in relation to their
office, it is essential that the facts showing the intimate relation between the office and the
offender and the discharge of official duties must be alleged in the Information—it is not enough
to merely allege in the Information that the crime charged was committed by the offender in
relation to his office because that would be a conclusion of law.

​FACTS: Petitioner Escobal is a graduate of the PMA, a member of the AFP and the
Philippine Constabulary, as well as the Intelligence Group of the Philippine National Police. On
March 16, 1990, the petitioner was conducting surveillance operations on drug trafficking at a
café bar and restaurant in Naga City when he somehow got involved with a shooting incident
that resulted to the death of Rodney Nueca.
​Escobal was preventively suspended from the service. When arraigned, he pleaded not
guilty. Thereafter, he filed a Motion to Quash the Information alleging that the court martial, not
the RTC, had jurisdiction over criminal cases involving PNP members and officers. RTC denied
the motion.
​Trial proceeded. The prosecution rested its case and petitioner presented his evidence. On
July 20, 1994, the petitioner filed a Motion to Dismiss the case. Citing Republic of the
Philippines v. Asuncion, et al., he argued that since he committed the crime in the performance
of his duties, the Sandiganbayan had exclusive jurisdiction over the case. The RTC dismissed the
motion but ordered the conduct of a preliminary hearing to determine whether or not the crime
charged was committed by the petitioner in relation to his office as a member of the PNP.
​On July 31, 1995, the trial court issued an Order declaring that the petitioner committed
the crime charged while not in the performance of his official function. The trial court added that
nonetheless, upon the enactment of R.A. No. 7975, the issue had become moot and academic
since the amendatory law transferred the jurisdiction over the offense charged from the
Sandiganbayan to the RTC. The petitioner did not have a salary grade of "27" as provided for in
or by Section 4(a)(1), (3) thereof.

​The trial court nevertheless ordered the prosecution to amend the Information pursuant to
the ruling in Republic v. Asuncion and R.A. No. 7975, and to include therein an allegation that
the offense charged was not committed by the petitioner in the performance of his
duties/functions, nor in relation to his office.
​The petitioner filed a MR of the said order, reiterating that based on his testimony and
those of his witnesses, the offense charged was committed by him in relation to his official
functions. He asserted that R.A. No. 7975, which was enacted on March 30, 1995, could not be
applied retroactively.
​The RTC ordered the public prosecutor to file a Re-Amended Information and to allege
that the offense charged was committed by the petitioner in the performance of his
duties/functions or in relation to his office; and, conformably to R.A. No. 7975, to thereafter
transmit the same to the Sandiganbayan.
The Sandiganbayan returned the records of the case to the RTC, contending that the latter has
jurisdiction over the case.

ISSUE:
​Whether the case falls in the jurisdiction of the Sandiganbayan or of the RTC
HELD:
​The case is within the jurisdiction of the RTC. Under Section 4(a) of P.D. No. 1606 as
amended by P.D. No. 1861, the Sandiganbayan had exclusive jurisdiction in all cases involving
the following:
​(1) Violations of Republic Act No. 3019, as amended, otherwise known as the Anti-Graft
​ ​ a​ nd Corrupt Practices Act, Republic Act No. 1379, and Chapter II, Section
2, Title VII of ​ ​ t​ he Revised Penal Code;
(​ 2) Other offenses or felonies committed by public officers and employees in relation to
​ ​ ​their office, including those employed in government-owned or controlled
corporations, ​ ​ ​ hether simple or complexed with other crimes, where the
w
penalty prescribed by law is ​ ​ ​ igher than prision correccional or
h
imprisonment for six (6) years, or a fine of ​ ​ ​ ​ 6,000.00 ....
P
​For the Sandiganbayan to have exclusive jurisdiction under the said law over crimes
committed by public officers in relation to their office, it is essential that the facts showing the
intimate relation between the office of the offender and the discharge of official duties must be
alleged in the Information. It is not enough to merely allege in the Information that the crime
charged was committed by the offender in relation to his office because that would be a
conclusion of law. The amended Information filed with the RTC against the petitioner does not
contain any allegation showing the intimate relation between his office and the discharge of his
duties. Hence, the RTC had jurisdiction over the offense charged when on November 24, 1995, it
ordered the re-amendment of the Information to include therein an allegation that the petitioner
committed the crime in relation to office. The trial court erred when it ordered the elevation of
the records to the Sandiganbayan. It bears stressing that R.A. No. 7975 amending P.D. No. 1606
was already in effect.
​Under Sec. 2 of said law, even if the offender committed the crime charged in relation to
his office but occupies a position corresponding to a salary grade below "27," the proper
Regional Trial Court or Municipal Trial Court, as the case may be, shall have exclusive
jurisdiction over the case. In this case, the petitioner was a Police Senior Inspector, with salary
grade "23." He was charged with homicide punishable by reclusion temporal. Hence, the RTC
had exclusive jurisdiction over the crime charged conformably to Sections 20 and 32 of Batas
Pambansa Blg. 129, as amended by Section 2 of R.A. No. 7691.
​The petitioner’s contention that R.A. No. 7975 should not be applied retroactively has no
legal basis. It bears stressing that R.A. No. 7975 is a substantive procedural law, which may be
applied retroactively.

PEOPLE VS HENRY GO

DOCTRINE:
​The Sandiganbayan is a special criminal court which has exclusive original jurisdiction in
all cases involving violations of Republic Act (R.A.) 3019 committed by certain public officers,
as enumerated in Presidential Decree (P.D.)
​1606 as amended by R.A. 8249. This includes private individuals who are charged as co-
principals, accomplices or accessories with the said public officers.

FACTS:
​An information was filed against Secretary Arturo Enrile and Henry T. Go before the
Sandiganbayan. The said information also alleged that there was a conspiray between Enrile and
Go.
​Go filed a motion to quash alleging that the operative facts adduced therein do not
constitute an offense under Section 3(g) of R.A. 3019. Respondent, citing the show cause order
of the SB, also contended that, independently of the deceased Secretary Enrile, the public officer
with whom he was alleged to have conspired, respondent, who is not a public officer nor was
capacitated by any official authority as a government agent, may not be prosecuted for violation
of Section 3(g) of R.A. 3019.

ISSUE:
​Whether the SB lost its jurisdiction over the person of the accused since he was not a
public officer.

HELD:
​It is true that by reason of Secretary Enrile's death, there is no longer any public officer
with whom respondent can be charged for violation of R.A. 3019. It does not mean, however,
that the allegation of conspiracy between them can no longer be proved or that their alleged
conspiracy is already expunged. The only thing extinguished by the death of Secretary Enrile is
his criminal liability. His death did not extinguish the crime nor did it remove the basis of the
charge of conspiracy between him and private respondent. Stated differently, the death of
Secretary Enrile does not mean that there was no public officer who allegedly violated Section 3
(g) of R.A. 3019. In fact, the Office of the Deputy Ombudsman for Luzon found probable cause
to indict Secretary Enrile for infringement of Sections 3 (e) and (g) of R.A. 3019.14 Were it not
for his death, he should have been charged.
​The requirement before a private person may be indicted for violation of Section 3(g) of
R.A. 3019, among others, is that such private person must be alleged to have acted in conspiracy
with a public officer. The law, however, does not require that such person must, in all instances,
be indicted together with the public officer. If circumstances exist where the public officer may
no longer be charged in court, as in the present case where the public officer has already died, the
private person may be indicted alone.

RAMISCAL VS SANDIGANBAYAN

DOCTRINE:
​In finding of probable cause, it is the Ombudsman who has the full discretion to
determine whether or not a criminal case should be filed in the Sandiganbayan, once the case has
been filed with the said court, it is the Sandiganbayan, and no longer the Ombudsman which has
full control of the case.

FACTS:
​Ramiscal Jr (Ramiscal) was a retired officer of AFP and the former president of AFP-
Retirement and Separation Benefits System (AFP-RSBS). During his incumbency, the BOD of
AFP-RSBS approved the acquisition of 15,020 sq. m. of land for development as housing
projects. On August 1, 1997 AFP-RSBS as represented by Ramiscal Jr., and Flaviano the
attorney-in-fact of 12 individual vendors executed and signed a bilateral Deed of Sale (1st Deed)
over the subject property at the agreed price of P 10,500.00 per sq. m. After the payment @ P
10,500.00 per sq. m., Flaviano executed and signed a unilateral Deed of Sale (2nd Deed) over the
same property with a purchase price of P 3,000.00 per sq. m. Thereafter the 2nd Deed was
presented by Flaviano for registration which became the basis of the Certificate of Title of the
said property.
​Ramiscal Jr filed his first Motion for Reconsideration date February 12, 1999 with a
supplemental motion dated May 28, 1999 regarding the findings of the Ombudsman. With this, a
panel of prosecutors was tasked to review the records of the case, they found out that Ramiscal
Jr., indeed participated in an affixed his signature on the contracts and found probable cause. The
Ombudsman acted positively on the findings of the prosecutor and scheduled the arraignment of
Ramiscal Jr. Howver, Ramiscal Jr., refused to enter a plea for petitioner on the ground that there
is a pending resolution of his second Motion for Reconsideration.

ISSUE:
1. Whether or not the second Motion for Reconsideration is valid and should hold his
arraignment.
2. Whether or not there is probable cause to file a case for violation of Section 3 (e) of the Anti-
Graft and Corrupt Practices Act and falsification of public documents.
HELD:
​No, Sec 7 of Rule 11 of the Rules provides that only one motion for reconsideration or
reinvestigation of an approved order or resolution shall be allowed xxxxx the filing of a motion
for reconsideration/reinvestigation shall not bar the filling of the corresponding information in
Court on the basis of the finding of probable cause in the resolution subject of the motion.
The arraignment may be suspended under Sec. 11 of Rule 116 of the Rules of Court are:
unsoundness of mind, prejudicial question and a pending petition for review of the resolution of
the prosecutor in the DOJ in which the suspension shall not exceed 60 days. Ramiscal Jr., failed
to show that any of the instances constituting a valid ground for suspension of arraignment
obtained in this case.
​With respect to the finding of probable cause, it is the Ombudsman who has the full
discretion to determine whether or not a criminal case should be filed in the Sandiganbayan, once
the case has been filed with the said court, it is the Sandiganbayan, and no longer the
Ombudsman which has full control of the case. Ramiscal Jr., failed to establish that
Sandiganbayan committed grave abuse of discretion, thus, there is probable cause in the filing of
the case.
PEOPLE VS BENIPAYO

DOCTRINES:
3. Jurisdiction in libel cases belong to the RTC to the exclusion of all other courts
4. The rule is well settled that the act of an accused in posting bail or in filing motions seeking
affirmative relief is tantamount to submission of his person to the jurisdiction of the court.
5. The Sandiganbayan is a special criminal court which has exclusive original jurisdiction in all
cases involving violations of Republic Act (R.A.) 3019 committed by certain public officers,
as enumerated in Presidential Decree (P.D.) 1606 as amended by R.A. 8249. This includes
private individuals who are charged as co-principals, accomplices or accessories with the
said public officers.
FACTS:
​Alfredo, then Chairman of the Commission of elections, was charged with libel before
the Office of the City Prosecutor by Photokina Marketing Corporation, which felt alluded to in a
speech made by Alfredo before the Bahay Kalinaw, University of the Philippines, and in an
television interview before “Point Blank”, a show hosted by Ces Orena- Drilon at ANC. The
Office of the City Prosecutor, in both instances, filed Informations for libel before the Regional
Trial Court. In both instances, Alfredo moved for the dismissal of the case, considering that at
the time he made the alleged utterances, he was an impeachable officer and the same was made
in relation to his duties, therefore, even assuming that he can be charged with libel, the same
should be lodged with the Sandiganbayan. In both instances, the RTC ruled in his favor, thus
Photokina elevated the case to the Supreme Court on question of law on whether the utterances
were made in relation to his office; and that the RTC had no jurisdiction over the case.

ISSUE:
​Whether or not (1) utterances of Benipayo were made in relation to his office, and (2)
RTC had jurisdiction over the libel case.

HELD:
​The Court observes that the parties have argued at length in their pleadings on the issue
of whether the alleged criminal acts of respondent are committed in relation to his office. They
are of the conviction that the resolution of the said question will ultimately determine which
court–the RTC or the Sandiganbayan–has jurisdiction over the criminal cases filed. The Court,
however, notes that both parties are working on a wrong premise. The foremost concern, which
the parties, and even the trial court, failed to identify, is whether, under our current laws,
jurisdiction over libel cases, or written defamations to be more specific, is shared by the RTC
with the Sandiganbayan. Indeed, if the said courts do not have concurrent jurisdiction to try the
offense, it would be pointless to still determine whether the crime is committed in relation to
office.
​Uniformly applied is the familiar rule that the jurisdiction of the court to hear and decide
a case is conferred by the law in force at the time of the institution of the action, unless a latter
statute provides for a retroactive application thereof. Article 360 of the Revised Penal Code
(RPC), as amended by Republic Act No. 4363, is explicit on which court has jurisdiction to try
cases of written defamations, thus: The criminal and civil action for damages in cases of written
defamations as provided for in this chapter, shall be filed simultaneously or separately with the
court of first instance [now, the Regional Trial Court] of the province or city where the libelous
article is printed and first published or where any of the offended parties actually resides at the
time of the commission of the offense xxx. [Underscoring and italics ours.]
​More than three decades ago, the Court, in Jalandoni v. Endaya, acknowledged the
unmistakable import of the said provision: There is no need to make mention again that it is a
court of first instance [now, the Regional Trial Court] that is specifically designated to try a libel
case. Its language is categorical; its meaning is free from doubt. This is one of those statutory
provisions that leave no room for interpretation. All that is required is application. What the law
ordains must then be followed.
​This exclusive and original jurisdiction of the RTC over written defamations is echoed in
Bocobo v. Estanislao, where the Court further declared that jurisdiction remains with the trial
court even if the libelous act is committed “by similar means,” and despite the fact that the
phrase “by similar means” is not repeated in the latter portion of Article 360. In these cases, and
in those that followed, the Court had been unwavering in its pronouncement that the expanded
jurisdiction of the municipal trial courts cannot be exercised over libel cases. Thus, in Manzano
v. Hon. Valera, we explained at length that: The applicable law is still Article 360 of the Revised
Penal Code, which categorically provides that jurisdiction over libel cases [is] lodged with the
Courts of First Instance (now Regional Trial Courts).
​This Court already had the opportunity to rule on the matter in G.R. No. 123263, People
vs. MTC of Quezon City, Branch 32 and Isah v. Red wherein a similar question of jurisdiction
over libel was raised. In that case, the MTC judge opined that it was the first level courts which
had jurisdiction due to the enactment of RA 7691. Upon elevation of the matter to us, respondent
judge’s orders were nullified for lack of jurisdiction, as follows: “WHEREFORE, the petition is
granted: the respondent Court’s Orders dated August 14, 1995, September 7, 1995, and October
18, 1995 are declared null and void for having been issued without jurisdiction; and said Court is
enjoined from further taking cognizance of and proceeding with Criminal Case No. 43-00548,
which it is commanded to remand to the Executive Judge of the Regional Trial Court of Quezon
City for proper disposition.”
​Another case involving the same question was cited as resolving the matter: “Anent the
question of jurisdiction, we ** find no reversible error committed by public respondent Court of
Appeals in denying petitioner’s motion to dismiss for lack of jurisdiction. The contention ** that
R.A. 7691 divested the Regional Trial Courts of jurisdiction to try libel cases cannot be
sustained. While libel is punishable by imprisonment of six months and one day to four years
and two months (Art. 360, Revised Penal Code) which imposable penalty is lodged within the
Municipal Trial Court’s jurisdiction under R.A. No. 7691 (Sec. 32 ), said law however, excludes
therefrom ** cases falling within the exclusive original jurisdiction of the Regional Trial Courts
**. The Court in Bocobo vs. Estanislao, 72 SCRA 520 and Jalandoni vs. Endaya, 55 SCRA 261,
correctly cited by the Court of Appeals, has laid down the rule that Regional Trial courts have
the exclusive jurisdiction over libel cases, hence, the expanded jurisdiction conferred by R.A.
7691 to inferior courts cannot be applied to libel cases.”
​Conformably with [these] rulings, we now hold that public respondent committed an
error in ordering that the criminal case for libel be tried by the MTC of Bangued.
For, although RA 7691 was enacted to decongest the clogged dockets of the Regional Trail
Courts by expanding the jurisdiction of first level courts, said law is of a general character. Even
if it is a later enactment, it does not alter the provision of Article 360 of the RPC, a law of a
special nature. “Laws vesting jurisdiction exclusively with a particular court, are special in
character, and should prevail over the Judiciary Act defining the jurisdiction of other courts (such
as the Court of First Instance) which is a general law.” A later enactment like RA 7691 does not
automatically override an existing law, because it is a well-settled principle of construction that,
in case of conflict between a general law and a special law, the latter must prevail regardless of
the dates of their enactment. Jurisdiction conferred by a special law on the RTC must therefore
prevail over that granted by a general law on the MTC.
​Moreover, from the provisions of R.A. 7691, there seems to be no manifest intent to
repeal or alter the jurisdiction in libel cases. If there was such intent, then the amending law
should have clearly so indicated because implied repeals are not favored. As much as possible,
effect must be given to all enactments of the legislature. A special law cannot be repealed,
amended or altered by a subsequent general law by mere implication. Furthermore, for an
implied repeal, a pre-condition must be found, that is, a substantial conflict should exist between
the new and prior laws. Absent an express repeal, a subsequent law cannot be construed as
repealing a prior one unless an irreconcilable inconsistency or repugnancy exists in the terms of
the new and old laws. The two laws, in brief, must be absolutely incompatible. In the law which
broadened the jurisdiction of the first level courts, there is no absolute prohibition barring
Regional Trial Courts from taking cognizance of certain cases over which they have been priorly
granted special and exclusive jurisdiction. Such grant of the RTC (previously CFI) was
categorically contained in the first sentence of the amended Sec. 32 of B.P. 129. The
inconsistency referred to in Section 6 of RA 7691, therefore, does not apply to cases of criminal
libel.
Lastly, in Administrative Order No. 104-96 issued 21 October 1996, this Court delineated the
proper jurisdiction over libel cases, hence settled the matter with finality:
“RE: DESIGNATION OF SPECIAL COURTS FOR KIDNAPPING, ROBBERY,
CARNAPPING, DANGEROUS DRUGS CASES AND OTHER HEINOUS CRIMES;
INTELLECTUAL PROPERTY RIGHTS VIOLATIONS AND JURISDICTION IN LIBEL
CASES.
xxxx
“LIBEL CASES SHALL BE TRIED BY THE REGIONAL TRIAL COURTS HAVING
JURISDICTION OVER THEM TO THE EXCLUSION OF THE METROPOLITAN TRIAL
COURTS, MUNICIPAL TRIAL COURTS IN CITIES, MUNICIPAL TRIAL COURTS AND
MUNICIPAL CIRCUIT TRIAL COURTS.” (Underscoring supplied)
​As we have constantly held in Jalandoni, Bocobo, People v. Metropolitan Trial Court of
Quezon City, Br. 32,[41] Manzano, and analogous cases, we must, in the same way, declare
herein that the law, as it still stands at present, dictates that criminal and civil actions for damages
in cases of written defamations shall be filed simultaneously or separately with the RTC to the
exclusion of all other courts. A subsequent enactment of a law defining the jurisdiction of other
courts cannot simply override, in the absence of an express repeal or modification, the specific
provision in the RPC vesting in the RTC, as aforesaid, jurisdiction over defamations in writing or
by similar means. The grant to the Sandiganbayan of jurisdiction over offenses committed in
relation to (public) office, similar to the expansion of the jurisdiction of the MTCs, did not divest
the RTC of its exclusive and original jurisdiction to try written defamation cases regardless of
whether the offense is committed in relation to office. The broad and general phraseology of
Section 4, Presidential Decree No. 1606, as amended by Republic Act No. 8249, cannot be
construed to have impliedly repealed, or even simply modified, such exclusive and original
jurisdiction of the RTC.
​Since jurisdiction over written defamations exclusively rests in the RTC without
qualification, it is unnecessary and futile for the parties to argue on whether the crime is
committed in relation to office. Thus, the conclusion reached by the trial court that the
respondent committed the alleged libelous acts in relation to his office as former COMELEC
chair, and deprives it of jurisdiction to try the case, is, following the above disquisition, gross
error. This Court, therefore, orders the reinstatement of Criminal Cases Nos. Q-02-109406 and
Q-02-109407 and their remand to the respective Regional Trial Courts for further proceedings.
Having said that, the Court finds unnecessary any further discussion of the other issues raised in
the petitions.”

LACSON VS EXECUTIVE SECRETARY

DOCTRINE:
​Under Section 4, par. b of Republic Act 8249, what determines the Sandiganbayan’s
jurisdiction is the official position or rank of the offender; In enacting Republic Act 8249, the
Congress simply restored the original provisions of Presidential Decree 1606 which does not
mention the criminal participation of the public officer as a requisite to determine the jurisdiction
of the Sandiganbayan.

FACTS:
​Eleven persons believed to be members of the Kuratong Baleleng gang, an organized
crime syndicate involved in bank robberies, were slain by elements of the Anti-Bank Robbery
and Intelligence Task Group (ABRITG). Among those included in the ABRITG were petitioners
and petitioner-intervenors.
​Acting on a media expose of SPO2 Eduardo delos Reyes, a member of the Criminal
Investigation Command, that what actually transpired was a summary execution and not a shoot-
out between the Kuratong Baleleng gang members and the ABRITG, Ombudsman Aniano
Desierto formed a panel of investigators to investigate the said incident. Said panel found the
incident as a legitimate police operation. However, a review board modified the panel’s finding
and recommended the indictment for multiple murder against twenty-six respondents including
herein petitioner, charged as principal, and herein petitioner-intervenors, charged as accessories.
After a reinvestigation, the Ombudsman filed amended informations before the Sandiganbayan,
where petitioner was charged only as an accessory.
​The accused filed separate motions questioning the jurisdiction of the Sandiganbayan,
asserting that under the amended informations, the cases fall within the jurisdiction of the
Regional Trial Court pursuant to Section 2 of R.A. 7975. They contend that the said law limited
the jurisdiction of the Sandiganbayan to cases where one or ore of the “principal accused” are
government officals with Salary Grade 27 or higher, or PNP officials with rank of Chief
Superintendent or higher. Thus, they did not qualify under said requisites. However, pending
resolution of their motions, R.A. 8249 was approved amending the jurisdiction of the
Sandiganbayan by deleting the word “principal” from the phrase “principal accused” in Section 2
of R.A. 7975.
​Petitioner questions the constitutionality of Section 4 of R.A. 8249, including Section 7
which provides that the said law shall apply to all cases pending in any court over which trial has
not begun as of the approval hereof.

ISSUE/S:
​Whether or not the multiple murder of the alleged members of the Kuratong Baleleng
was committed in relation to the office of the accused PNP officers which is essential to the
determination whether the case falls within the Sandiganbayan’s or Regional Trial Court’s
jurisdiction.

HELD:
RTC HAS JURISDICTION.
​In People vs. Montejo, it was held that an offense is said to have been committed in
relation to the office if it is intimately connected with the office of the offender and perpetrated
while he was in the performance of his official functions. Such intimate relation must be alleged
in the information which is essential in determining the jurisdiction of the Sandiganbayan.
However, upon examination of the amended information, there was no specific allegation of
facts that the shooting of the victim by the said principal accused was intimately related to the
discharge of their official duties as police officers. Likewise, the amended information does not
indicate that the said accused arrested and investigated the victim and then killed the latter while
in their custody. The stringent requirement that the charge set forth with such particularity as will
reasonably indicate the exact offense which the accused is alleged to have committed in relation
to his office was not established.
​Consequently, for failure to show in the amended informations that the charge of murder
was intimately connected with the discharge of official functions of the accused PNP officers, the
offense charged in the subject criminal cases is plain murder and, therefore, within the exclusive
original jurisdiction of the Regional Trial Court and not the Sandiganbayan.
​WHEREFORE, the constitutionality of Sections 4 and 7 of R.A. 8249 is hereby
sustained. The Addendum to the March 5, 1997 Resolution of the Sandiganbayan is
REVERSED. The Sandiganbayan is hereby directed to transfer Criminal Cases Nos. 23047 to
23057 (for multiple murder) to the Regional Trial Court of Quezon City which has exclusive
original jurisdiction over the said cases.

SANCHEZ VS DEMETRIO

DOCTRINE:
​The absence of a preliminary investigation does not impair the validity of the information
or otherwise render the same defective and neither does it affect the jurisdiction of the court over
the case or constitute a ground for quashing the information.

FACTS:
​Accused-appellants were found guilty beyond reasonable doubt of seven (7) counts of
rape with homicide on seven counts and sentenced each one of them to suffer the penalty of
seven reclusion perpetua. The prosecutions version of the events was based mainly on the
recollections of its star witnesses Aurelio Centeno and Vicencio Malabanan - coconspirators
turned state witnesses. Both admitted having taken part in the abduction of Eileen Sarmenta and
Allan Gomez, but denied any personal involvement in the rape of Eileen and the twin killings
that followed. In this appeal, the pith of the assigned errors and the focus of the appellants
arguments is the issue of witnesses Centeno and Malabanans credibility, whose open- court
narrations served as principal basis for the trial courts rendition of a guilty verdict.

ISSUE:
​Whether or not the court acted properly on denying the petition of Sanchez to quash on
the grounds that he is being charged with seven homicides arising from the death of only two
persons.

HELD:
​The court ruled that where there are two or more offenders who commit rape, the
homicide committed on the occasion or by reason of each rape, must be deemed as a constituent
of the special complex crime of rape with homicide. Therefore, there will be as many crimes of
rape with homicide as there are rapes committed. In effect, the presence of homicide qualifies the
crime of rape, thereby raising its penalty to the highest degree. Thus, homicide committed on the
occasion or by reason of rape, loses its character as an independent offense, but assumes a new
character, and functions like a qualifying circumstance. However, by fiction of law, it merged
with rape to constitute an constituent element of a special complex crime of rape with homicide
with a specific penalty which is in the highest degree.
​The petitioner and his six co-accused are not charged with only one rape committed by
him in conspiracy with the other six. Each one of the seven accused is charged with having
himself raped Sarmenta instead of simply helping Sanchez in committing only one rape. In other
words, the allegation of the prosecution is that the girl was raped seven times, with each of the
seven accused taking turns in abusing her with the assistance of the other six. Afterwards, their
lust satisfied, all seven of them decided to kill and thus silence Sarmenta. Every one of the seven
accused is being charged separately for actually raping Sarmenta and later killing her instead of
merely assisting the petitioner in raping and then slaying her. The separate informations filed
against each of them allege that each of the seven successive rapes is complexed by the
subsequent slaying of Sarmenta and aggravated by the killing of Allan Gomez by her seven
attackers. The separate rapes were committed in succession by the seven accused, culminating in
the slaying of Sarmenta.
​The matter of assigning values to declarations on the witness stand is best and most
competently performed by the trial judge who had the unmatched opportunity to observe the
witnesses and to assess their credibility by the various indicia available but not reflected in the
record. The trial courts impressions of the star witnesses for the State bind this Court, for we
accord great respect if not finality, to the findings of the trial court on the credibility of witnesses.
Further, all the appellants relied on the defense of denial/alibi but positive identification by
credible witnesses of the accused as the perpetrators of the crime, demolishes the alibi.
​Calauan Mayor Antonio Sanchez brought a Petition for Certiorari before this Court,
challenging the order of the respondent Judge Demetriou denying his motion to quash the
Information filed against him and six other persons for alleged rape and homicide. One of the
arguments of Mayor Sanchez was that there was discrimination against him because of the non-
inclusion of two other persons in the Information. We held that even this Court cannot order
the prosecution of a person against whom the prosecutor does not find sufficient evidence
to support at least a prima facie case. However, if there was an unmistakable showing of grave
abuse of discretion on the part of the prosecutors, Mayor Sanchez should have filed a Petition
for Mandamus to compel the filing of charges against said two other persons.

DISINI JR VS SECRETARY OF JUSTICE


DOCTRINE:
​It is well-settled that the power to issue subpoenas is not exclusively a judicial function.
Executive agencies have the power to issue subpoena as an adjunct of their investigatory powers.

FACTS:
The following provisions were questioned:
1. 2. 3. 4. 5. 6. 7. 8. 9. 10. 11. 12. 13. 14. 15. 16. 17. 18. 19. 20. 21.

ISSUE/S: Which court has jurisdiction over cases covered by RA 10175?

HELD:
​There shall be designated special cybercrime court manned by specially trained judges to
handle cybercrime cases.

RULE 110 – PROSECUTION OF OFFENSES

JIMENEZ VS SORONGON

DOCTRINE:
​The People is the real party in interest in a criminal case and only the OSG can represent
the People in criminal proceedings pending in the CA or in the SC.

FACTS:
​In 2003, Jimenez, the president of Unlad Shipping & Management Corporation, filed a
complaint-affidavit with the Office of the City Prosecutor of Mandaluyong City against
Antzoulatos, Alamil, Gaza and Avgoustis listed incorporators of Tsakos Maritime Services, Inc.
(TMSI), for syndicated and large scale illegal recruitment, alleging that Antzoulatos, et al.
Petitioners assail the validity of several provision of the Republic Act (R.A.) 10175, the
Cybercrime Prevention Act of 2012. Petitioners claim that the means adopted by the cybercrime
law for regulating undesirable cyberspace activities violate certain constitutional rights. The
government of course asserts that the law merely seeks to reasonably put order into cyberspace
activities, punish wrongdoings, and prevent hurtful attacks on the system.
Section 4(a)(1) on Illegal Access;
Section 4(a)(3) on Data Interference;
Section 4(a)(6) on Cyber-squatting;
Section 4(b)(3) on Identity Theft;
Section 4(c)(1) on Cybersex;
Section 4(c)(2) on Child Pornography;
Section 4(c)(3) on Unsolicited Commercial Communications;
Section 4(c)(4) on Libel;
Section 5 on Aiding or Abetting and Attempt in the Commission of Cybercrimes;
Section 6 on the Penalty of One Degree Higher;
Section 7 on the Prosecution under both the Revised Penal Code (RPC) and R.A. 10175;
Section 8 on Penalties;
Section 12 on Real-Time Collection of Traffic Data;
Section 13 on Preservation of Computer Data;
Section 14 on Disclosure of Computer Data;
Section 15 on Search, Seizure and Examination of Computer Data;
Section 17 on Destruction of Computer Data;
Section 19 on Restricting or Blocking Access to Computer Data;
Section 20 on Obstruction of Justice;
Section 24 on Cybercrime Investigation and Coordinating Center (CICC); and
Section 26(a) on CICCs Powers and Functions
​Sec. 21 provides that the RTC shall have jurisdiction over any violation of the provisions
of RA 10175 including any violation committed by a Filipino national regardless of the place of
commission. Jurisdiction shall lie if any of the elements was committed within the Philippines or
committed with the use of any computer system wholly or partly situated in the country, or when
by such commission any damage is caused to a natural or juridical person who, at the time the
offense was committed, was in the Philippines. falsely represented their stockholdings in TMSI’s
articles of incorporation to secure a license to operate as a recruitment agency from the
Philippine Overseas Employment Agency (POEA).
​An Information was then filed before the Regional Trial Court (RTC) Mandaluyong.
Subsequently, the City Prosecutor reconsidered resolution and filed a motion with the RTC to
withdraw the information. The RTC denied the motion to withdraw information as it found the
existence of probable cause to hold Antzoulatos, et al. for trial. Thus, the RTC ordered the
issuance of warrants of arrest against Antzoulatos, et al. Antzoulatos and Gaza filed an omnibus
motion for reconsideration and for deferred enforcement of the warrants of arrest, which was
denied.
​Alamil filed a motion for judicial determination of probable cause with a request to defer
enforcement of the warrants of arrest., which was denied for being moot and academic. Alamil
moved for reconsideration and for the inhibition of Judge Capco- Umali, for being biased or
partial.
​Judge Capco-Umali voluntarily inhibited herself from the case and did not resolve
Alamil’s motion for reconsideration and the Jimenez's motion to expunge. The case was later re-
raffled to Branch 214, presided by Judge Edwin D. Sorongon. The RTC granted Alamil’s motion
for reconsideration, and treated the motion for judicial determination as a motion to dismiss for
lack of probable cause. Jimenez moved for reconsideration, stressing the existence of probable
cause to prosecute the respondents and that respondent Alamil had no standing to seek any relief
from the RTC, which was denied. The RTC ordered the motion expunged from the records since
the motion did not have the public prosecutor’s conformity. Jimenez filed a notice of appeal.
Alamil moved to expunge the Jimenez' notice of appeal since the public prosecutor did not
authorize the appeal and the petitioner had no civil interest in the case.
The RTC denied the Jimenez's notice of appeal since Jimenez filed it without the conformity of
the Solicitor General, who is mandated to represent the People of the Philippines in criminal
actions appealed to the CA. Thus, the RTC ordered the notice of appeal expunged from the
records. Jimenez elevated his case to the CA via a Rule 65 petition for certiorari, which was
dismissed outright by the CA for Jimenez’s lack of legal personality to file the petition on behalf
of the People of the Philippines. The CA denied the motion for reconsideration that followed.

ISSUE:
​Whether or not Jimenez, the private complainant, has legal personality in assailing the
RTC Orders.

HELD:
​NO. It is well-settled that "every action must be prosecuted or defended in the name of
the real party in interest[,]" "who stands to be benefited or injured by the judgment in the suit, or
by the party entitled to the avails of the suit." Interest means material interest or an interest in
issue to be affected by the decree or judgment of the case, as distinguished from mere interest in
the question involved. By real interest is meant a present substantial interest, as distinguished
from a mere expectancy, or a future, contingent, subordinate or consequential interest. When the
plaintiff or the defendant is not a real party in interest, the suit is dismissible.
​Procedural law basically mandates that "[a]ll criminal actions commenced by complaint
or by information shall be prosecuted under the direction and control of a public prosecutor." In
appeals of criminal cases before the CA and before this Court, the OSG is the appellate counsel
of the People, pursuant to Section 35(1), Chapter 12, Title III, Book IV of the 1987
Administrative Code.
​The People is the real party in interest in a criminal case and only the OSG can represent
the People in criminal proceedings pending in the CA or in this Court. This ruling has been
repeatedly stressed in several cases and continues to be the controlling doctrine.
​While there may be rare occasions when the offended party may be allowed to pursue the
criminal action on his own behalf (as when there is a denial of due process), this exceptional
circumstance does not apply in the present case.

PEOPLE VS VALDEZ

DOCTRINE:
​The real nature of the criminal charge is determined not from the caption or preamble of
the information, or from the specification of the provision of law alleged to have been violated,
which are mere conclusions of law, but by the actual recital of the facts in the complaint or
information.

FACTS:
​On March 1, 2000, at around 8:00 in the evening, Estrella Sayson was at a canteen
(which also includes a jai alai betting station) located at 77 Corregidor St, Bago Bantay, Quezon
City. Estrella was preparing for the 2nd celebration of her 2nd husband, Wilfredo Lladones,
which was held later that evening. Estrella’s son, the deceased Moises Sayson, a former
policeman, and his wife, Susan Sayson, owned the said canteen and betting station. At about
9:00, Estrella’s other sons, Joselito and Ferdinand Sayson arrived at the canteen to greet their
stepfather.
​At about 10:00 of the same evening, the celebration was interrupted by the arrival of
Eduardo and Edwin who alighted from a motorcycle in front of the jai alai betting station.
Eduardo and Edwin, agitated, asked the jai alai teller, Jonathan Rubio, to come out. Jonathan was
then attending to other customers who were buying jai alai tickets. Moises approached Eduardo
and Edwin and tried to reason with them. He advised them not to force Jonathan to go out.
Estrella then heard one of the accused- appellants threaten Moises with the words “Gusto mo
unahin kita?” Moises replied “Huwag”. Successive gunshots were thereafter heard. Moises fell
and was continuously fired upon. Ferdinand immediately approached the scene to help his
brother. He was however, was also shot on the left temporal portion of his head and fell. Joselito,
ran away but was also hit at the back while running. The RTC convicted the 2 accused of 3
counts of murder and sentenced them to suffer Resclusion Perpetua for each count of murder. On
appeal, the CA affirmed the convictions.In this appeal, PO2 Valdez contended that the state failed
to establish the qualifying circumstance of treachery.

ISSUE:
​Whether the prosecution sufficiently established the qualifying circumstance of treachery.

HELD:
​For complaint or information to be sufficient, it must state the name of the accused; the
designation of the offense given by the statute; the acts or omissions complained of as
constituting the offense; the name of the offended party; the approximate time of the commission
of the offense, and the place wherein the offense was committed. What is controlling is not the
title of the complaint, nor the designation of the offense charged or the particular law or part
thereof allegedly violated, these being mere conclusions of law made by the prosecutor, but the
description of the crime charged and the particular facts therein recited. The acts or omissions
complained of must be alleged in such form as is sufficient to enable a person of common
understanding to know what offense is intended to be charged, and enable the court to pronounce
proper judgment. No information for a crime will be sufficient if it does not accurately and
clearly allege the elements of the crime charged. Every element of the offense must be stated in
the information. What facts and circumstances are necessary to be included therein must be
determined by reference to the definitions and essentials of the specified crimes. The requirement
of alleging the elements of a crime in the information is to inform the accused of the nature of
the accusation against him so as to enable him to suitably prepare his defense. The presumption
is that the accused has no independent knowledge of the facts that constitute the offense.
​The averments of the informations to the effect that the two accused "with intent to kill,
qualified with treachery, evident premeditation and abuse of superior strength did x x x assault,
attack and employ personal violence upon" the victims "by then and there shooting them with a
gun, hitting [them]" on various parts of their bodies "which were the direct and immediate cause
of their deaths" did not sufficiently set forth the facts and circumstances describing how
treachery attended each of the killings. It should not be difficult to see that merely averring the
killing of a person by shooting him with a gun, without more, did not show how the execution of
the crime was directly and specially ensured without risk to the accused from the defense that the
victim might make. Indeed, the use of the gun as an instrument to kill was not per se treachery,
for there are other instruments that could serve the same lethal purpose. Nor did the use of the
term treachery constitute a sufficient averment, for that term, standing alone, was nothing but a
conclusion of law, not an averment of a fact. In short, the particular acts and circumstances
constituting treachery as an attendant circumstance in murder were missing from the
informations.
​x x x. The requirement of sufficient factual averments is meant to inform the accused of
the nature and cause of the charge against him in order to enable him to prepare his defense. This
requirement accords with the presumption of innocence in his favor, pursuant to which he is
always presumed to have no independent knowledge of the details of the crime he is being
charged with. To have the facts stated in the body of the information determine the crime of
which he stands charged and for which he must be tried thoroughly accords with common sense
and with the requirements of plain justice, x x x.

MIGUEL VS SANDIGANBAYAN

DOCTRINE:
​The test of the information’s sufficiency is whether the crime is described in intelligible
terms and with such particularity with reasonable certainty so that the accused is duly informed
of the offense charged.

​FACTS: Vice Mayor and other local officials of Koronadal City, South Cotabato filed a
letter-complaint with the Office of the Ombudsman-Mindanao charging the petitioner, Fernando
Miguel, of violations of R.A. 3019 (Antigraft and Corrupt Practices Act) in connection with the
consultancy services for the proposed Koronadal Public Market.
​The Ombudsman directed the petitioner to submit his counter-affidavit. After moving for
an extension, the petitioner filed his counter-affidavit. Thereafter, the Ombudsman found
probable cause against the petitioner and some other private individuals for violation of RA 3019
and against the petitioner alone for Falsification of Public Documents under Art. 171 par. 4 of the
RPC. The Ombudsman filed the corresponding informations with the Sandiganbayan.
​The Sandiganbayan ordered the Office of the Special Prosecutor (OSP) to conduct a
reinvestigation. The Sandiganbayan gave the petitioner 10 days to file his counter-affidavit with
the OSP. Instead of submitting his counter-affidavit, petitioner asked for another 3-day extension
and afterwhich another 20-day extension.
​Despite the given extensions, the petitioner still failed to file his counter-affidavit
prompting prosecutor Norberto B. Ruiz to declare the petitioner to have waived his right to
submit countervailing evidence. Ombudsman approved the resolution. After several extensions
sought and granted, the petitioner filed a Motion to Quash and/or Reinvestigation for the
criminal cases against him. The Sandiganbayan denied the petitioner’s motion because of the
pending OSP reinvestigation this, despite the OSPs earlier termination of the reinvestigation for
the petitioner’s continuous failure to submit his counter- affidavit. After the arraignment, the
OSP file a Motion to Suspend Pendente Lite. The petitioner filed his Vigorous Opposition based
on the obvious and fatal defect in the information. The Sandiganbayan promulgated the assailed
resolution suspending the petitioner pendent lite.
​The petitioner moved for reconsideration of his suspension order and demanded pre-
suspension hearing. The Sandiganbayan denied his motion, prompting him to file this certiorari
petition to challenge the validity of the suspension order.

ISSUE:
​Whether the absence of an actual pre-suspension hearing renders invalid the suspension
order against the petitioner.

HELD:
​No. In Bedruz vs Sandiganbayan, the court considered the opposition of the accused
(Motion to Suspend Pendente Lite) as sufficient to dispense with the need to actually set the
prosecution’s motion for hearing.
​In the case at bar, while there was no pre-suspension hearing held to determine the
validity of the information/suspension, the court believed that the pleadings filed for and against
them achieved the goal of the procedure. The right to due process is satisfied not by just oral
arguments but by the filing and the consideration by the court of the parties’ pleadings,
memoranda and other position papers.
​Since a pre-suspension hearing is basically a due process requirement, when an accused
public official is given an adequate opportunity to be heard on his possible defenses against a
mandatory suspension under RA 3019, then an accused would have no reason to complain that
no actual hearing was conducted. It is well settled that to be heard does not only mean oral
arguments in court; one may be heard also through pleadings. Where opportunity to be heard,
either through oral arguments or pleadings, has been accorded, no denial of procedural due
process exists.

PEOPLE VS SORIA

DOCTRINE:
​The allegation in the information of the various ways of committing the of ense should be
regarded as a description of only one of ense and the information is not thereby rendered
defective on the ground of multifariousness.

FACTS:
Version of the Prosecution:
​The victim, AAA, is the daughter of the accused. On February 26, 2000, AAA and her
siblings enjoyed the spaghetti their father brought home for merienda. After eating, AAA went to
the bedroom to rest. Thereafter, appellant also entered the room and positioned himself on top of
AAA, took off her clothes and “felt” that he inserted his penis into her vagina. AAA felt intense
pain from her breast down to her vagina and thus told her father that it was painful. At that point,
appellant apologized to his daughter, stood up, and left the room. The whole incident was
witnessed by AAA’s brother, BBB. The pain persisted until AAA’s vagina started to bleed. She
thus told her aunt about it and they proceeded to a hospital for treatment. Her mother was also
immediately informed of her ordeal. AAA was taken into the custody of the Department of
Social Welfare and Development.

Version of the Defense:


​Appellant admitted that he was at home on the day and time of AAA’s alleged rape but
denied committing the same. Instead, he claimed that the filing of the rape case against him was
instigated by his wife, whom he confronted about her illicit affair with a man residing in their
community. According to appellant, he could not have molested AAA because he treated her
well. In fact, he was the only one sending his children to school since his wife already neglected
them and seldom comes home.
ISSUE:
6. Whether the allegations of the accused is credible to cast a reasonable doubt which would
warrant his acquittal?
7. Whether the information is sufficient to justify the conviction of the accused?
HELD:
1. ​Rape can now be committed either through sexual intercourse or by sexual assault.
Rape under paragraph 1 of Article 266-A is referred to as rape through sexual intercourse. Carnal
knowledge is the central element and it must be proven beyond reasonable doubt. On the other
hand, rape under paragraph 2 of Article 266-A is commonly known as rape by sexual assault.
The perpetrator commits this kind of rape by inserting his penis into another person’s mouth or
anal orifice, or any instrument or object into the genital or anal orifice of another person.
The RTC and the CA found the accused guilty of rape through sexual intercourse but It is evident
from the testimony of AAA that she was unsure whether it was indeed appellant’s penis which
touched her labia and entered her organ. AAA stated that she only knew that it was the “bird” of
her father which was inserted into her vagina after being told by her brother BBB. Clearly, AAA
has no personal knowledge that it was appellant’s penis which touched her labia and inserted into
her vagina. Hence, it would be erroneous to conclude that there was penile contact based solely
on the declaration of AAA’s brother, BBB, which declaration was hearsay due to BBB’s failure
to testify.
​The court however found it inconsequential that AAA could not specifically identify the
particular instrument or object that was inserted into her genital. What is important and relevant
is that indeed something was inserted into her vagina. Moreover, the prosecution satisfactorily
established that appellant accomplished the act of sexual assault through his moral ascendancy
and influence over “AAA” which substituted for violence and intimidation. Thus, there is no
doubt that appellant raped AAA by sexual assault.
​It is also improbable for appellant’s wife to have dared encourage their daughter AAA to
file the charges publicly expose the dishonor of the family unless the rape was indeed committed.
2. ​The Information in this case did not specify with certainty whether appellant committed
the rape through sexual intercourse under paragraph 1 of Article 266-A, or rape by sexual assault
as described in paragraph 2 thereof. The Information stated that appellant inserted his penis into
the genital of "AAA," which constituted rape by sexual intercourse under the first paragraph of
Article 266-A. At the same time, the Information alleged that appellant used force and
intimidation to commit an act of sexual assault. While these allegations cause ambiguity, they
only pertain to the mode or manner of how the rape was committed and the same do not
invalidate the Information or result in the automatic dismissal of the case. "[W]here an offense
may be committed in any of the different modes and the offense is alleged to have been
committed in two or more modes specified, the indictment is sufficient, notwithstanding the fact
that the different means of committing the same offense are prohibited by separate sections of the
statute. The allegation in the information of the various ways of committing the offense should
be regarded as a description of only one offense and the information is not thereby rendered
defective on the ground of multifariousness." Any objection from the appellant with respect to
the Information is held to have been waived failing any effort to oppose the same before trial. He
therefore can be convicted of rape through sexual intercourse or rape by sexual assault,
depending on the evidence adduced during trial.
​In determining whether appellant is indeed guilty of rape through sexual intercourse
under paragraph 1 of Article 266-A, it is essential to establish beyond reasonable doubt that he
had carnal knowledge of "AAA". There must be proof that his penis touched the labia of "AAA"
or slid into her female organ, and not merely stroked the external surface thereof, to ensure his
conviction of rape by sexual intercourse.
​We reviewed the testimony of "AAA" and found nothing therein that would show that
she was raped through sexual intercourse. While "AAA" categorically stated that she felt
something inserted into her vagina, her testimony was sorely lacking in important details that
would convince us with certainty that it was indeed the penis of appellant that was placed into
her vagina.
​Based on the evidence adduced, the accused is found guilty beyond reasonable doubt for
the crime of rape by sexual assault.
UNION BANK VS PEOPLE

DOCTRINE:
​The venue of action and of jurisdiction are deemed sufficiently alleged where the
Information states that the offense was committed or some of its essential ingredients occurred at
a place within the territorial jurisdiction of the court

FACTS:
​Desi Tomas was charged in court for perjury under Article 183 of the Revised Penal Code
(RPC) for making a false narration in a Certificate against Forum Shopping.
​The accusation stemmed from Union Bank’s two (2) complaints for sum of money with
prayer for a writ of replevin against the spouses Eddie and Eliza Tamondong and a John Doe,
filed before the RTC, Branch 109, Pasay City; and filed and raffled to the MeTC, Branch 47,
Pasay City. Both complaints showed that Tomas executed and signed the Certification against
Forum Shopping.
​Accordingly, she was charged of deliberately violating Article 183 of the RPC by falsely
declaring under oath in the Certificate against Forum Shopping in the second complaint that she
did not commence any other action or proceeding involving the same issue in another tribunal or
agency.
​Tomas filed a Motion to Quash, arguing, among others, that the venue was improperly
laid since it is the Pasay City court (where the Certificate against Forum Shopping was submitted
and used) and not the MeTC-Makati City (where the Certificate against Forum Shopping was
subscribed) that has jurisdiction over he perjury case. Second, she argued that the facts charged
do not constitute an offense because: (a) the third element of perjury – the willful and deliberate
assertion of falsehood – was not alleged with particularity without specifying what the other
action or proceeding commenced involving the same issues in another tribunal or agency; (b)
there was no other action or proceeding pending in another court when the second complaint was
filed; and (c) she was charged with perjury by giving false testimony while the allegations in the
Information make out perjury by making a false affidavit. The MeTC-Makati City denied the
Motion to Quash and subsequently denied Tomas’ motion for reconsideration.
​Union Bank and Tomas filed a petition for certiorari before the RTC-Makati City to annul
and set aside the MeTC- Makati City orders on the ground of grave abuse of discretion, which
was dismissed. The RTC-Makati City subsequently denied Union Bank and Tomas’ motion for
reconsideration. Hence, the review under Rule 45.

ISSUE:
​Whether or not Makati City, where the Certificate Against Forum Shopping was
notarized, is the proper venue for the case of perjury.

HELD:
​YES. Venue is an essential element of jurisdiction in criminal cases. It determines not
only the place where the criminal action is to be instituted, but also the court that has the
jurisdiction to try and hear the case. The reason for this rule is two- fold. First, the jurisdiction of
trial courts is limited to well-defined territories such that a trial court can only hear and try cases
involving crimes committed within its territorial jurisdiction. Second, laying the venue in the
locus criminis is grounded on the necessity and justice of having an accused on trial in the
municipality of province where witnesses and other facilities for his defense are available.
​Unlike in civil cases, a finding of improper venue in criminal cases carries jurisdictional
consequences. In determining the venue where the criminal action is to be instituted and the
court which has jurisdiction over it, Section 15(a), Rule 110 of the 2000 Revised Rules of
Criminal Procedure provides that subject to existing laws, the criminal action shall be instituted
and tried in the court or municipality or territory where the offense was committed or where any
of its essential ingredients occurred.
​Place of commission of the offense. – The complaint or information is sufficient if it can
be understood from its allegations that the offense was committed or some of its essential
ingredients occurred at some place within the jurisdiction of the court, unless the particular place
where it was committed constitutes an essential element of the offense charged or is necessary
for its identification.
​Both provisions categorically place the venue and jurisdiction over criminal cases not
only in the court where the offense was committed, but also where any of its essential ingredients
took place. In other words, the venue of action and of jurisdiction are deemed sufficiently alleged
where the Information states that the offense was committed or some of its essential ingredients
occurred at a place within the territorial jurisdiction of the court.

RULE 111 – PROSECUTION OF CIVIL ACTION

SOLIDUM VS PEOPLE

DOCTRINE:
8. Civil liability must not rest on speculation but on competent evidence.
9. The civil action for the recovery of civil liability that is deemed instituted with the criminal
action refers only to that arising from the offense charged.
FACTS:
​Two days after his birth, Gerald Gercayo, who was born with an imperforate anus,
underwent colostomy, enabling him to excrete through a colostomy bag attached to the side of
his body. In 1995, Gerald, then three years old, was admitted at the Ospital ng Maynila for a pull-
through operation. Dr. Leandro Resurreccion headed the surgical team, and was assisted by Dr.
Joselito Luceño, Dr. Donatella Valeña and Dr. Joseph Tibio. The anesthesiologists included Dr.
Marichu Abella, Dr. Arnel Razon and Dr. Fernando Solidum.
​During the operation, Gerald experienced bradycardia, and went into a coma. His coma
lasted for two weeks, but he regained consciousness only after a month. He could no longer see,
hear or move.
​Ma. Luz Gercayo (Luz) then lodged a complaint for reckless imprudence resulting in
serious physical injuries with the City Prosecutor’s Office of Manila. Upon a finding of probable
cause, the City Prosecutor’s Office filed an information solely against Dr. Solidum
Dr. Solidum were pronounced guilty of reckless imprudence resulting in serious physical injuries
by the Regional Trial Court (RTC) and the Court of Appeals (CA).
He then filed an appeal to the SC. The SC acquitted Dr. Solidum because circumstances, taken
together, did not prove beyond reasonable doubt that Dr. Solidum had been recklessly imprudent
in administering the anesthetic agent to Gerald.

ISSUES:
10. Whether or not Dr. Solidum, through his acquittal, is exempt from civil liability.
11. Whether or not Ospital ng Maynila should be adjudged jointly and severally liable with the
accused.

HELD:
1. YES.
​The circumstances that have been established do not present the factual and legal bases
for validly doing so. His acquittal did not derive only from reasonable doubt. There was really no
firm and competent showing how the injury to Gerard had been caused. That meant that the
manner of administration of the anesthesia by Dr. Solidum was not necessarily the cause of the
hypoxia that caused the bradycardia experienced by Gerard. Consequently, to adjudge Dr.
Solidum civilly liable would be to speculate on the cause of the hypoxia. We are not allowed to
do so, for civil liability must not rest on speculation but on competent evidence.
2. NO
​In criminal prosecutions, the civil action for the recovery of civil liability that is deemed
instituted with the criminal action refers only to that arising from the offense charged. Ospital ng
Maynila, being an artificial entity, had not been charged along with Dr. Solidum. The lower
courts thereby acted capriciously and whimsically, which rendered their judgment against Ospital
ng Maynila void as the product of grave abuse of discretion amounting to lack of jurisdiction.
For one, Ospital ng Maynila was not at all a party in the proceedings. Hence, its fundamental
right to be heard was not respected from the outset. The RTC and the CA should have been alert
to this fundamental defect. Verily, no person can be prejudiced by a ruling rendered in an action
or proceeding in which he was not made a party. Such a rule would enforce the constitutional
guarantee of due process of law.
​Moreover, Ospital ng Maynila could be held civilly liable only when subsidiary liability
would be properly enforceable pursuant to Article 103 of the Revised Penal Code. But the
subsidiary liability seems far-fetched here. The conditions for subsidiary liability to attach to
Ospital ng Maynila should first be complied with. Firstly, pursuant to Article 103 of the Revised
Penal Code, Ospital ng Maynila must be shown to be a corporation "engaged in any kind of
industry." The term industry means any department or branch of art, occupation or business,
especially one that employs labor and capital, and is engaged in industry. However, Ospital ng
Maynila, being a public hospital, was not engaged in industry conducted for profit but purely in
charitable and humanitarian work. Secondly, assuming that Ospital ng Maynila was engaged in
industry for profit, Dr. Solidum must be shown to be an employee of Ospital ng Maynila acting
in the discharge of his duties during the operation on Gerald. Yet, he definitely was not such
employee but a consultant of the hospital. And, thirdly, assuming that civil liability was adjudged
against Dr. Solidum as an employee (which did not happen here), the execution against him was
unsatisfied due to his being insolvent.

CASTILLO VS SALVADOR

DOCTRINE:
​An acquittal based on reasonable doubt on the guilt of the accused is not exempt from
civil liability, which may be proved by preponderance of evidence only.

FACTS:
​Phillip R. Salvador and his brother, Ramon Castillo, was charged with Estafa under
Article 315, paragraph 2 (a) of the Revised Penal Code by petitioner Cristina B. Castillo, a
businesswoman engaged in real estate business, educational institution, boutique, and trading
business. She was then enticed by Phillip and Ramon to engage in freight and remittance
business.
​As petitioner had deeply fallen in love with respondent Salvador and since she trusted
him very much as he even acted as a father to her children while her annulment was ongoing, she
agreed to embark on the remittance business. She agreed with respondent and Ramon that any
profit derived from the business would be equally divided among them and that respondent
would be in charge of promotion and marketing in Hong Kong, and Ramon would take charge of
the operations of business in the Philippines and she would be financing the business.
​The business has not operated yet as petitioner was still raising the amount of
US$100,000.00 as capital for the actual operation. When petitioner already had the money, she
handed the same to respondent Salvador, which was witnessed by her disabled half-brother
Enrico B. Tan. However, the proposed business never operated as respondent only stayed in
Hong Kong for three days. When she asked respondent about the money and the business, the
latter told her that the money was deposited in a bank. However, upon further query, respondent
confessed that he used the money to pay for his other obligations. Since then, the US$100,000.00
was not returned at all. Upon their arraignment, Phillip and Ramon pleaded not guilty to the
offense charged. Trial on the merits thereafter ensued. The RTC rendered a Decision finding
Phillip guilty beyond reasonable doubt and ordered him to pay Castillo, US$100,000.00 or its
equivalent in Philippine currency. Ramon was acquitted for insufficiency of evidence. Phillip
appealed his conviction to the CA, which overturned the RTC decision. Castillo filed a petition
for review on certiorari on the civil aspect of the case, arguing that the Court should have at least
retained the amount of damages to her.

ISSUE:
​Whether or not the acquitted accused are still liable for damages.
HELD:
​ ES. In Manantan v. CA, we discussed the consequences of an acquittal on the civil
Y
liability of the accused as follows.
​Our law recognizes two kinds of acquittal, with different effects on the civil liability of
the accused. First is an acquittal on the ground that the accused is not the author of the actor
omission complained of. This instance closes the door to civil liability, for a person who has been
found to be not the perpetrator of any act or omission cannot and can never be held liable for
such act or omission. There being no delict, civil liability ex delicto is out of the question, and
the civil action, if any, which may be instituted must be based on grounds other than the delict
complained of. This is the situation contemplated in Rule III of the Rules of Court. The second
instance is an acquittal based on reasonable doubt on the guilt of the accused. In this case, even if
the guilt of the accused has not been satisfactorily established, he is not exempt from civil
liability, which may be proved by preponderance of evidence only. This is the situation
contemplated in Article 29 of the Civil Code, where the civil action for damages is "for the same
act or omission." x x x.
​A reading of the CA decision would show that Phillip was acquitted because the
prosecution failed to prove his guilt beyond reasonable doubt. [S]ince the acquittal is based on
reasonable doubt, [Phillip] is not exempt from civil liability which may be proved by
preponderance of evidence only. In Encinas v. National Bookstore, Inc., we explained the
concept of preponderance of evidence as follows:
​x x x Preponderance of evidence is the weight, credit, and value of the aggregate
evidence on either side and is usually considered to be synonymous with the term "greater
weight of the evidence" or "greater weight of the credible evidence." Preponderance of evidence
is a phrase which, in the last analysis, means probability of the truth. It is evidence which is more
convincing to the court as worthy of belief than that which is offered in opposition thereto.
​In discrediting [Castillo's] allegation that she gave [Phillip] US$100,000.00 in May 2002,
the CA found that: (1) [Castillo] failed to show how she was able to raise the money in such a
short period of time and even gave conflicting versions on the source of the same; (2)
[Castillo]failed to require respondent to sign a receipt so she could have a record of the
transaction and offered no plausible reason why the money was allegedly hand-carried toHong
Kong; (3) [Castillo's] claim of trust as reason for not requiring [Phillip] to sign a receipt was
inconsistent with the way she conducted her previous transactions with him; and (4) [Castillo's]
behavior after the alleged fraud perpetrated against her was inconsistent with the actuation of
someone who had been swindled.

LIM VS. KOU CO PING

DOCTRINE:
​If the action for the civil liability ex delicto is instituted prior to or subsequent to the
filing of the criminal action, its proceedings are suspended until the final outcome of the criminal
action. Because of the distinct and independent nature of the two kinds of civil liabilities,
jurisprudence holds that the of ended party may pursue the two types of civil liabilities
simultaneously or cumulatively, without of ending the rules on forum shopping, litis pendentia,
or res judicata.

FACTS:
​FR Cement Corporation (FRCC), owner/operator of a cement manufacturing plant, issued
several withdrawal authorities for the account of cement dealers and traders, Fil-Cement Center
and Tigerbilt. These withdrawal authorities state the number of bags that the dealer/trader paid
for and can withdraw from the plant. Each withdrawal authority contained a provision that it is
valid for six months from its date of issuance, unless revoked by FRCC Marketing Department.
​Fil-Cement Center and Tigerbilt, through their administrative manager, Gail Borja
(Borja), sold the withdrawal authorities covering 50,000 bags of cement to Co for the amount of
P3.15 million or P63.00 per bag. Co sold these withdrawal authorities to Lim allegedly at the
price of P64.00 per bag or a total of P3.2 million. Using the withdrawal authorities, Lim
withdrew the cement bags from FRCC on a staggered basis. She successfully withdrew 2,800
bags of cement, and sold back some of the withdrawal authorities, covering 10,000 bags, to Co.
​FRCC did not allow Lim to withdraw the remaining 37,200 bags covered by the
withdrawal authorities. Lim clarified the matter with Co and Borja, who explained that the plant
implemented a price increase and would only release the goods once Lim pays for the price
difference or agrees to receive a lesser quantity of cement. Lim objected and maintained that the
withdrawal authorities she bought were not subject to price fluctuations. Lim sought legal
recourse after her demands for Co to resolve the problem with the plant or for the return of her
money had failed.
​An Information for Estafa through Misappropriation or Conversion was filed against Co.
The private complainant, Lily Lim, participated in the criminal proceedings to prove her
damages. She prayed for Co to return her money amounting to P2,380,800.00, foregone profits,
and legal interest, and for an award of moral and exemplary damages, as well as attorney’s fees.
The RTC of Pasay acquitted Co for insufficiency of evidence. After the trial on the civil aspect of
the criminal case, the Pasig City RTC also relieved Co of civil liability to Lim.
​Lim sought a reconsideration of the above Order, arguing that she has presented
preponderant evidence that Co committed estafa against her. The trial court denied the motion.
Lim filed her notice of appeal.
​A month later, Lim filed a complaint for specific performance and damages before
Branch 21 of the RTC of Manila. The defendants in the civil case were Co and all other parties to
the withdrawal authorities, Tigerbilt, Fil-Cement Center, FRCC, Southeast Asia Cement, and La
Farge Corporation. The complaint asserted two causes of action: breach of contract and abuse of
rights.

ISSUE:
​Whether or not Lim commited forum shopping in filing the civil case for specific
performance and damages during the pendency of her appeal on the civil aspect of the criminal
case for estafa.

HELD:
​NO. A single act or omission that causes damage to an offended party may give rise to
two separate civil liabilities on the part of the offender: (1) civil liability ex delicto, that is, civil
liability arising from the criminal offense under Article 100 of the Revised Penal Code, and (2)
independent civil liability, that is, civil liability that may be pursued independently of the
criminal proceedings. The independent civil liability may be based on “an obligation not arising
from the act or omission complained of as a felony,” as provided in Article 31 of the Civil Code
(such as for breach of contract or for tort). It may also be based on an act or omission that may
constitute felony but, nevertheless, treated independently from the criminal action by specific
provision of Article 33 of the Civil Code (“in cases of defamation, fraud and physical injuries”).
The civil liability arising from the offense or ex delicto is based on the acts or omissions that
constitute the criminal offense; hence, its trial is inherently intertwined with the criminal action.
For this reason, the civil liability ex delicto is impliedly instituted with the criminal offense. If
the action for the civil liability ex delicto is instituted prior to or subsequent to the filing of the
criminal action, its proceedings are suspended until the final outcome of the criminal action. The
civil liability based on delict is extinguished when the court hearing the criminal action declares
that “the act or omission from which the civil liability may arise did not exist.”
​The independent civil liabilities are separate from the criminal action and may be pursued
independently, as provided in Articles 31 and 33 of the Civil Code, which state that:
​ART. 31. When the civil action is based on an obligation not arising from the act or
​ ​ ​ mission complained of as a felony, such civil action may proceed
o
independently of the ​ ​ c​ riminal proceedings and regardless of the result of
the latter.
​ART. 33. In cases of defamation, fraud, and physical injuries a civil action for damages,
​ ​ e​ ntirely separate and distinct from the criminal action, may be brought by
the injured ​ ​ ​ arty. Such civil action shall proceed independently of the
p
criminal prosecution, and shall r​ equire only a preponderance of evidence.
​ ecause of the distinct and independent nature of the two kinds of civil liabilities,
B
jurisprudence holds that the offended party may pursue the two types of civil liabilities
simultaneously or cumulatively, without offending the rules on forum shopping, litis pendentia,
or res judicata.

CASUPANAN VS LAROYA

DOCTRINE:
​The accused can file a civil action for quasi-delict for the same act or omission he is
accused of in the criminal case. This is expressly allowed in paragraph 6, Section 1 of the present
Rule 111 which states that the counterclaim of the accused “may be litigated in a separate civil
action.”

FACTS:
​Two vehicles, one driven by respondent Laroya and the other owned by petitioner
Capitulo and driven by petitioner Casupanan, figured in an accident. As a result, two cases were
filed with the MCTC.
​Laroya filed a criminal case against Casupanan for reckless imprudence resulting in
damage to property, while Casupanan and Capitulo filed a civil case against Laroya for quasi-
delict. When the civil case was filed, the criminal case was then at its preliminary investigation
stage. Laroya, defendant in the civil case, filed a motion to dismiss the civil case on the ground
of forum-shopping considering the pendency of the criminal case. The MCTC granted the
motion and dismissed the civil case.
​On Motion for Reconsideration, Casupanan and Capitulo insisted that the civil case is a
separate civil action which can proceed independently of the criminal case. The MCTC denied
the motion for reconsideration. Casupanan and Capitulo filed a petition for certiorari under Rule
65 before the RTC assailing the MCTC’s Order of dismissal.
​The RTC dismissed the petition for lack of merit. The Capas RTC ruled that the order of
dismissal issued by the MCTC is a final order which disposes of the case and therefore the
proper remedy should have been an appeal. The Capas RTC further held that a special civil
action for certiorari is not a substitute for a lost appeal. Finally, the Capas RTC declared that
even on the premise that the MCTC erred in dismissing the civil case, such error is a pure error
of judgment and not an abuse of discretion. Casupanan and Capitulo filed a Motion for
Reconsideration but the Capas RTC denied the same.

ISSUE:
​Whether Casupanan and Capitulo, who are not the offended parties in the criminal case,
can file a separate civil action against the offended party in the criminal case.

HELD:
​Yes. Under Section 1 of the present Rule 111, the independent civil action in Articles 32,
33, 34 and 2176 of the Civil Code is not deemed instituted with the criminal action but may be
filed separately by the offended party even without reservation. The commencement of the
criminal action does not suspend the prosecution of the independent civil action under these
articles of the Civil Code. The suspension in Section 2 of the present Rule 111 refers only to the
civil action arising from the crime, if such civil action is reserved or filed before the
commencement of the criminal action.
​Thus, the offended party can file two separate suits for the same act or omission. The first
a criminal case where the civil action to recover civil liability ex-delicto is deemed instituted, and
the other a civil case for quasi-delict—without violating the rule on non-forum shopping. The
two cases can proceed simultaneously and independently of each other. The commencement or
prosecution of the criminal action will not suspend the civil action for quasi-delict. The only
limitation is that the offended party cannot recover damages twice for the same act or omission
of the defendant. In most cases, the offended party will have no reason to file a second civil
action since he cannot recover damages twice for the same act or omission of the accused. In
some instances, the accused may be insolvent, necessitating the filing of another case against his
employer or guardians.
​Similarly, the accused can file a civil action for quasi-delict for the same act or omission
he is accused of in the criminal case. This is expressly allowed in paragraph 6, Section 1 of the
present Rule 111 which states that the counterclaim of the accused “may be litigated in a separate
civil action.” This is only fair for two reasons.
​First, the accused is prohibited from setting up any counterclaim in the civil aspect that is
deemed instituted in the criminal case. The accused is therefore forced to litigate separately his
counterclaim against the offended party. If the accused does not file a separate civil action for
quasi-delict, the prescriptive period may set in since the period continues to run until the civil
action for quasi-delict is filed.
​Second, the accused, who is presumed innocent, has a right to invoke Article 2177 of the
Civil Code, in the same way that the offended party can avail of this remedy which is
independent of the criminal action. To disallow the accused from filing a separate civil action for
quasi-delict, while refusing to recognize his counterclaim in the criminal case, is to deny him due
process of law, access to the courts, and equal protection of the law.
Thus, the civil action based on quasi-delict filed separately by Casupanan and Capitulo is proper.

PEOPLE VS ROMERO

DOCTRINE:
​The death of the accused pending appeal of his conviction extinguishes his criminal
liability as well as the civil liability ex delicto.

FACTS:
​Complainant Ernesto A. Ruiz was a radio commentator of Radio DXRB, Butuan City. He
came to know the business of Surigao San Andres Industrial Development Corporation
(SAIDECOR) when he interviewed accused Martin Romero and Ernesto Rodriguez regarding
the corporation’s investment operations in Butuan City and Agusan del Norte. Romero was the
president and general manager of SAIDECOR, while Rodriguez was the operations manager.
SAIDECOR started its operation on August 24, 1989 as a marketing business. Later, it engaged
in soliciting funds and investments from the public. The corporation guaranteed an 800% return
on investment within fifteen (15) or twenty one (21) days. Investors were given coupons
containing the capital and the return on the capital collectible on the date agreed upon. It stopped
operations in September, 1989.
​Complainant Ernesto A. Ruiz went to SAIDECOR office in Butuan City to make an
investment, accompanied by his friend Jimmy Acebu, and SAIDECOR collection agent Daphne
Parrocho. After handing over the amount of one hundred fifty thousand pesos (P150,000.00) to
Ernesto Rodriguez, complainant received a postdated Butuan City Rural Bank check instead of
the usual redeemable coupon. The check indicated P1,000,200.00 as the amount in words, but
the amount in figures was for P1,200,000.00, as the return on the investment. Complainant did
not notice the discrepancy. When the check was presented to the bank for payment on October 5,
1989, it was dishonored for insufficiency of funds, as evidenced by the check return slip issued
by the bank.
​Both accused could not be located and demand for payment was made only sometime in
November 1989 during the preliminary investigation of this case. Accused responded that they
had no money. Daphne Parrocho testified that complainant, with his friend Jimmy Acebu,
approached her to invest the amount of P150,000.00 at SAIDECOR. As she has reached her
quota, and therefore, no longer authorized to receive the amount, she accompanied them to the
office of SAIDECOR at Ong Yiu District, Butuan City. Accused Ernesto Rodriguez accepted the
investment and issued the check signed by him and Martin Romero.
​For their defense, accused Martin Romero testified that he issued a check in the amount
of P1,200,000.00 corresponding to the total of the P150,000.00 investment and the 800% return
thereon. He claimed that the corporation had a deposit of fourteen million pesos
(P14,000,000.00) at the time of the issuance of the check and four million pesos (P4,000,000.00)
at the time SAIDECOR stopped operations. Romero knew these things because he used to
monitor the funds of the corporation with the bank. He was not aware that the check he issued
was dishonored because he never had the occasion to meet the complainant again after the
September 14, 1989 transaction. He only came to know about this when the case was already
filed in court sometime in the second or third week of January 1990.
​On appeal, both accused did not deny that complainant made an investment with
SAIDECOR in the amount of P150,000.00. However, they denied that deceit was employed in
the transaction. They assigned as errors: (1) their conviction under P.D. 1689 due to the
prosecution’s failure to establish their guilt beyond reasonable doubt; and (2) the trial court’s
failure to consider the joint stipulation of facts in their favor. Ernesto Rodriguez, died pending
appeal.

ISSUE:
​What is the effect of the death of an accused on the case?

HELD:
​Pursuant to the doctrine established in People vs. Bayotas, the death of the accused
pending appeal of his conviction extinguishes his criminal liability as well as the civil liability ex
delicto. The criminal action is extinguished inasmuch as there is no longer a defendant to stand
as the accused, the civil action instituted therein for recovery of civil liability ex delicto is ipso
facto extinguished, grounded as it is on the criminal case.
​Corollarily, the claim for civil liability survives notwithstanding the death of the accused,
if the same may also be predicated on a source of obligation other than delict. Thus, the outcome
of this appeal pertains only to the remaining accused-appellant, Martin L. Romero.

MAGISTRADO VS PEOPLE

DOCTRINE:
​There is no prejudicial question if the civil and the criminal action can, according to law,
proceed independently of each other.

FACTS:
​Private respondent Elena M. Librojo filed a criminal complaint for perjury against
petitioner Magestrado with the Office of the City Prosecutor. After the filing of petitioner’s
counter-affidavit and the appended pleadings, the Office of the City Prosecutor recommended the
filing of an information for perjury against petitioner. Thus, Assistant City Prosecutor Josephine
Z. Fernandez filed an information for perjury with the MeTC.
​Petitioner filed a motion for suspension of proceedings based on a prejudicial question.
Petitioner alleged that Civil Case No. Q-98-34349, a case for recovery of a sum of money
pending before the Regional Trial Court (RTC) of Quezon City, Branch 84, and Civil Case No.
Q-9834308, a case for Cancellation of Mortgage, Delivery of Title and Damages, pending before
the RTC of Quezon City, Branch 77, must be resolved first before Criminal Case No. 90721 may
proceed since the issues in the said civil cases are similar or intimately related to the issues raised
in the criminal action.

ISSUE:
​Whether it is proper to suspend Criminal Case No. 90721 for perjury pending final
outcome of Civil Case No. Q98-34349 and Civil Case No. Q-98-34308 on the ground of
prejudicial question.

HELD:
​No. Rule 111 of the Rules of Court provides that:
​Sec. 6. Suspension by reason of prejudicial question.—A petition for suspension of the
criminal action based upon the pendency of a prejudicial question in a civil action may be filed
in the office of the prosecutor or the court conducting the preliminary investigation. When the
criminal action has been filed in court for trial, the petition to suspend shall be filed in the same
criminal action at any time before the prosecution rests.
​Sec. 7. Elements of prejudicial question.—The elements of a prejudicial question are: (a)
the previously instituted civil action involves an issue similar or intimately related to the issue
raised in the subsequent criminal action; and (b) the resolution of such issue determines whether
or not the criminal action may proceed.
​The rationale behind the principle of suspending a criminal case in view of a prejudicial
question is to avoid two conflicting decisions. A prejudial question is defined as that which arises
in a case the resolution of which is a logical antecedent of the issue involved therein, and the
cognizance of which pertains to another tribunal. The prejudicial question must be determinative
of the case before the court but the jurisdiction to try and resolve the question must be lodged in
another court or tribunal. It is a question based on a fact distinct and separate from the crime but
so intimately connected with it that it determines the guilt or innocence of the accused.
​For a prejudicial question in a civil case to suspend criminal action, it must appear not
only that said case involves facts intimately related to those upon which the criminal prosecution
would be based but also that in the resolution of the issue or issues raised in the civil case, the
guilt or innocence of the accused would necessarily be determined.
​Thus, for a civil action to be considered prejudicial to a criminal case as to cause the
suspension of the criminal proceedings until the final resolution of the civil case, the following
requisites must be present: (1) the civil case involves facts intimately related to those upon which
the criminal prosecution would be based; (2) in the resolution of the issue or issues raised in the
civil action, the guilt or innocence of the accused would necessarily be determined; and (3)
jurisdiction to try said question must be lodged in another tribunal.
​If the resolution of the issue in the civil action will not determine the criminal
responsibility of the accused in the criminal action based on the same facts, or there is no
necessity “that the civil case be determined first before taking up the criminal case,” therefore,
the civil case does not involve a prejudicial question. Neither is there a prejudicial question if the
civil and the criminal action can, according to law, proceed independently of each other.
However, the court in which an action is pending may, in the exercise of sound discretion, and
upon proper application for a stay of that action, hold the action in abeyance to abide by the
outcome of another case pending in another court, especially where the parties and the issues are
the same, for there is power inherent in every court to control the disposition of cases on its
dockets with economy of time and effort for itself, for counsel, and for litigants.
​Where the rights of parties to the second action cannot be properly determined until the
questions raised in the first action are settled, the second action should be stayed. The power to
stay proceedings is incidental to the power inherent in every court to control the disposition of
the cases on its dockets, considering its time and effort, those of counsel and the litigants. But if
proceedings must be stayed, it must be done in order to avoid multiplicity of suits and prevent
vexatious litigations, conflicting judgments, confusion between litigants and courts. It bears
stressing that whether or not the trial court would suspend the proceedings in the criminal case
before it is submitted to its sound discretion.

PIMENTEL VS PIMENTEL

DOCTRINE:
​Annulment of marriage is not a prejudicial question in criminal case for parricide.
Further, the resolution of the civil action is not a prejudicial question that would warrant the
suspension of the criminal action. There is a prejudicial question when a civil action and a
criminal action are both pending, and there exists in the civil action an issue which must be
preemptively resolved before the criminal action may proceed because howsoever the issue
raised in the civil action is resolved would be determinative of the guilt or innocence of the
accused in the criminal case.

FACTS:
​On October 25, 2004, Maria Chrysantine Pimentel y Lacap filed an action for frustrated
parricide against Joselito R. Pimentel with the RTC Quezon City. Less than four months
thereafter, Joselito received summons to appear before the RTC in Antipolo City for the pre-trial
and trial of for Declaration of Nullity of Marriage between Maria and Joselito on the ground of
psychological incapacity. Petitioner filed an urgent motion to suspend the proceedings before the
RTC Quezon City on the ground of the existence of a prejudicial question. He asserted that since
the relationship between the offender and the victim is a key element in parricide, the outcome of
the civil case for Declaration of Nullity of Marriage would have a bearing in the criminal case
filed against him before the RTC Quezon City.
​The RTC Quezon City denied the motion, holding that the pendency of the case before
the RTC Antipolo is not a prejudicial question that warrants the suspension of the criminal case
before it. It held that the issues in the parricide case are the injuries sustained by respondent and
whether the case could be tried even if the validity of petitioner’s marriage with respondent is in
question.
​Petitoner filed a petition for certiorari with application of WPI and/or TRO before the
Court of Appeals, assailing the denial of RTC Quezon City.
​The CA dismissed the petition. The CA ruled that even if the marriage between petitioner
and respondent would be declared void, it would be immaterial to the criminal case because prior
to the declaration of nullity, the alleged acts constituting the crime of frustrated parricide had
already been committed. The Court of Appeals ruled that all that is required for the charge of
frustrated parricide is that at the time of the commission of the crime, the marriage is still
subsisting.
ISSUE:
​Whether the resolution of the action in annulment of marriage is a prejudicial question;
HELD:
​NO. The rule is clear that the civil action must be instituted first before the filing of the
criminal action. Clearly, the civil case for annulment was filed after the filing of the criminal case
for frustrated parricide. As such, the requirement of Section 7, Rule 111 of the 2000 Rules on
Criminal Procedure was not met since the civil action was filed subsequent to the filing of the
criminal action.
​Further, the resolution of the civil action is not a prejudicial question that would warrant
the suspension of the criminal action.
​There is a prejudicial question when a civil action and a criminal action are both pending,
and there exists in the civil action an issue which must be preemptively resolved before the
criminal action may proceed because howsoever the issue raised in the civil action is resolved
would be determinative of the guilt or innocence of the accused in the criminal case.
A prejudicial question is defined as:
“x x x one that arises in a case the resolution of which is a logical antecedent of the issue
involved therein, and the cognizance of which pertains to another tribunal. It is a question based
on a fact distinct and separate from the crime but so intimately connected with it that it
determines the guilt or innocence of the accused, and for it to suspend the criminal action, it must
appear not only that said case involves facts intimately related to those upon which the criminal
prosecution would be based but also that in the resolution of the issue or issues raised in the civil
case, the guilt or innocence of the accused would necessarily be determined.”
​The issue in the civil case for annulment of marriage under Article 36 of the Family Code
is whether petitioner is psychologically incapacitated to comply with the essential marital
obligations. The issue in parricide is whether the accused killed the victim. In this case, since
petitioner was charged with frustrated parricide, the issue is whether he performed all the acts of
execution which would have killed respondent as a consequence but which, nevertheless, did not
produce it by reason of causes independent of petitioner’s will.16 At the time of the commission
of the alleged crime, petitioner and respondent were married. The subsequent dissolution of their
marriage, in case the petition for Declaration of Nullity of Marriage is granted, it will have no
effect on the alleged crime that was committed at the time of the subsistence of the marriage. In
short, even if the marriage between petitioner and respondent is annulled, petitioner could still be
held criminally liable since at the time of the commission of the alleged crime, he was still
married to respondent.

J.M. DOMINGUEZ VS LICLICAN

DOCTRINE:
​As jurisprudence elucidates, a prejudicial question generally exists in a situation where a
civil action and a criminal action are both pending, and there exists in the former an issue that
must be preemptively resolved before the latter may proceed, because howsoever the issue raised
in the civil action is resolved would be determinative juris et de jure of the guilt or innocence of
the accused in the criminal case. The rationale behind the principle is to avoid two conflicting
decisions, and its existence rests on the concurrence of two essential elements: (i) the civil action
involves an issue similar or intimately related to the issue raised in the criminal action; and (ii)
the resolution of such issue determines whether or not the criminal action may proceed.

FACTS:
​During the annual stockholders meeting of petitioner JM Dominguez Agronomic
Company, Inc. (JMD) held at the Baguio City Country Club, the election for its new set of
directors was conducted. This event was presided by then company president respondent Cecilia
Liclican (Liclican), and attended by her co-respondents Norma Isip (Isip) and Purita Rodriguez,
and by petitioners Helen Dagdagan (Dagdagan), Patrick Pacis, Kenneth Pacis, and Shirley
Dominguez (Dominguez) as well. Conflict ensued when petitioners Patrick and Kenneth Pacis
were allegedly not allowed to vote on the ground that they are not registered stockholders of
JMD. As pointed out, it was their mother and grandmother, both deceased, who are the
stockholders in JMD, and that there is still no settlement of their respective estates to effectively
transfer their shares in the company to Patrick and Kenneth Pacis. But since the remaining
stockholders with outstanding shares constituted a quorum, the election of officers still
proceeded.
​In reaction to the foregoing developments, petitioners filed a Complaint against
respondents before the RTC Baguio Branch 59 for nullification of meetings, election and acts of
directors and officers, injunction and other reliefs. After a failed mediation, was referred for
appropriate Judicial Dispute Resolution (JDR) to Branch 7 of the RTC. Meanwhile, petitioner
stockholders immediately took hold of corporate properties, represented themselves to JMD’s
tenants as the true and lawful directors of the company, and collected and deposited rents due the
company to its bank account.
​ ubsequently, JMD, represented by petitioners, executed an Affidavit-Complaint
S
charging respondents Liclican and Isip with qualified theft. Petitioners alleged in the complaint
that Liclican and Isip, without any authority whatsoever, conspired to withdraw the amount of
P852,024.19 from the corporation’s savings account with the Equitable-PCI Bank; and that the
following day, they issued a check in the amount of P200,000, payable to cash, and to be drawn
against JMD’s account with Robinson’s Savings Bank. In a separate complaint, the corporation
claimed that respondents Liclican and Isip likewise issued a check payable to one Atty. Francisco
Lava, Jr. for P200,000 to be debited from the corporation’s account.
​The City Prosecutor of Baguio City recommends for approval of the Informations for
Qualified Theft against LICLICAN and ISIP. Thereafter, Judge Tiongson-Tabora of RTC Baguio
found probable cause and issued a warrant of arrest.
​In due time, respondents lodged a petition for certiorari with the CA, to annul and set
aside the two Orders by the RTC, branch 7 anchored, among others, on the alleged existence of a
prejudicial question. According to respondents, petitioner stockholders, by filing the complaint-
affidavit, are already assuming that they are the legitimate directors of JMD, which is the very
issue in the intra-corporate dispute pending in the RTC, Branch 59.
​The CA granted the petition, holding that Judge Tiongson-Tabora should have refrained
from determining probable cause since she is well aware of the pendency of the issue on the
validity of JMD’s elections. As the judge overseeing the JDR of the said intra-corporate dispute,
she knew that there was still doubt as to who the rightfully elected directors of JMD are and,
corollarily, who would have the authority to initiate the criminal proceedings for qualified theft.
The CA further noted that even as corporate officers, as they claim to be, petitioners Dagdagan
and Patrick Pacis cannot file the Complaint-Affidavit in the exercise of corporate powers without
authority from the board of directors under Sec. 23,18 in relation to Sec. 2519 of the Corporation
Code :
Section 23. The board of directors or trustees.—Unless otherwise provided in this Code, the
corporate powers of all corporations formed under this Code shall be exercised, all business
conducted and all property of such corporations controlled and held by the board of directors or
trustees to be elected from among the holders of stocks, or where there is no stock, from among
the members of the corporation, who shall hold office for one (1) year until their successors are
elected and qualified. x x x

ISSUE/S:
12. Whether or not the civil case constituted a prejudicial question warranting the suspension of
criminal proceedings;
13. Whether or not there was grave abuse of discretion on the part of Judge Tabora in
disregarding the pending case on the validity of JMD’s election, by issuing the warrants;

HELD:
(1)
​YES. As jurisprudence elucidates, a prejudicial question generally exists in a situation
where a civil action and a criminal action are both pending, and there exists in the former an
issue that must be preemptively resolved before the latter may proceed, because howsoever the
issue raised in the civil action is resolved would be determinative juris et de jure of the guilt or
innocence of the accused in the criminal case. The rationale behind the principle is to avoid two
conflicting decisions, and its existence rests on the concurrence of two essential elements: (i) the
civil action involves an issue similar or intimately related to the issue raised in the criminal
action; and (ii) the resolution of such issue determines whether or not the criminal action may
proceed.
​Here, the CA aptly observed the intra-corporate dispute, posed a prejudicial question to
Criminal Case. To be sure, the Civil involves the same parties herein, and is for nullification of
JMD’s meetings, election and acts of its directors and officers, among others. Court intervention
was sought to ascertain who between the two contesting group of officers should rightfully be
seated at the company’s helm. Without resolution of the civil case, petitioners’ authority to
commence and prosecute the Criminal case against respondents for qualified theft in JMD’s
behalf remained questionable, warranting the suspension of the criminal proceedings.
(2)
​YES. In the case at bar, the CA correctly ruled that Judge Tiongson-Tabora acted with
grave abuse of discretion when she ordered the arrests of respondents Isip and Liclican despite
the existence of a prejudicial question. Judge Tiongson-Tabora cannot deny knowledge of the
pendency of Civil Case No. 6623-R as the judge presiding over its JDR. As correctly held by the
CA.
​Judge Tiongson-Tabora is well-aware of the existence of said prejudicial question that
should have barred the filing of the criminal complaint against petitioners Liclican and Isip, for
the simple reason that a juridical person can only act through its officers, and the issue in the
main case submitted for JDR before Judge Tiongson-Tabora is one for nullification of meetings,
election and act of directors and officers, injunction and other reliefs. Thus, she knows for a fact
that there is a question as to who are the legitimate directors of JMD such that there is doubt as to
whether private respondents are in a position to act for JMD.

RULE 112 – PRELIMINARY INVESTIGATION

FENEQUITO VS VERGARA, JR.

DOCTRINE:
​It is wrong for petitioners to argue that it is the OSG which has authority to file an appeal
with the RTC. Section 35 (l), Chapter 12, Title III of Book IV of Executive Order No. 292,
otherwise known as the Administrative Code of 1987, mandates the OSG to represent “the
Government in the Supreme Court and the Court of Appeals in all criminal proceedings.” On the
other hand, Section 11 of Presidential Decree No. 1275, entitled “Reorganizing the Prosecution
Staff of the Department of Justice and the Offices of the Provincial and City Fiscals,
Regionalizing the Prosecution Service, and Creating the National Prosecution Service,” which
was the law in force at the time the appeal was filed, provides that the provincial or the city fiscal
(now referred to as prosecutor) “shall have charge of the prosecution of all crimes, misdemeanors
and violations of city or municipal ordinances in the courts of such province or city and shall
therein discharge all the duties incident to the institution of criminal prosecutions.”

FACTS:
​An Information for falsification of public documents was filed with the MTC of Manila
by the Assistant City Prosecutor of Manila against herein petitioners. Herein petitioners filed a
Motion to Dismiss the Case Based on Absence of Probable Cause. After respondent’s
Comment/Opposition was filed, the MeTC issued an Order dismissing the case on the ground of
lack of probable cause. Aggrieved, respondent, with the express conformity of the public
prosecutor, appealed the case to the Regional Trial Court (RTC) of Manila. The RTC rendered
judgment setting aside the Order of the MeTC and directing the said court to proceed to trial.
Petitioners then elevated the case to the CA via a petition for review. The CA rendered its
presently assailed Resolution dismissing the petition. The CA ruled that the Decision of the RTC
is interlocutory in nature and, thus, is not appealable. Petitioners filed a Motion for
Reconsideration but the CA denied the same.
​Hence, the instant petition based on the following grounds that the (1) CA erred in
outright dismissal of the petition on the ground that the remedy is improper and (2) RTC Ruling
is final and unappealable. The petitioners relies on “Strict enforcement of the Rules may be
suspended whenever the purposes of justice so require.”
ISSUE/S:
14. Whether or not the CA erred in dismissing the petition on improper remedy;
15. Whether or not the RTC ruling is final and unappealable;
HELD:
(1)
​NO. The Court notes at the outset that one of the grounds relied upon by the CA in
dismissing petitioners’ petition for review is the latter’s failure to submit copies of pleadings and
documents relevant and pertinent to the petition filed, as required under Section 2, Rule 42 of the
Rules of Court. While petitioners filed a Motion for Reconsideration, they, however, failed to
comply with these requirements. Worse, they did not even mention anything about it in the said
Motion. Section 3, Rule 42 of the same Rules provides:
​“Sec. 3. Effect of failure to comply with requirements.—The failure of the petitioner to
comply with any of the foregoing requirements regarding the payment of the docket and other
lawful fees, the deposit for costs, proof of service of the petition, and the contents of and the
documents which should accompany the petition shall be sufficient ground for the dismissal
thereof.”
​Moreover, it is a settled rule that the right to appeal is neither a natural right nor a part of
due process; it is merely a statutory privilege, and may be exercised only in the manner and in
accordance with the provisions of law. An appeal being a purely statutory right, an appealing
party must strictly comply with the requisites laid down in the Rules of Court. Deviations from
the Rules cannot be tolerated. The rationale for this strict attitude is not difficult to appreciate as
the Rules are designed to facilitate the orderly disposition of appealed cases. In an age where
courts are bedeviled by clogged dockets, the Rules need to be followed by appellants with
greater fidelity. Their observance cannot be left to the whims and caprices of appellants. In the
instant case, petitioners had all the opportunity to comply with the Rules. Nonetheless, they
remained obstinate in their non-observance even when they sought reconsideration of the ruling
of the CA dismissing their petition. Such obstinacy is incongruous with their late plea for
liberality in construing the Rules. On the above basis alone, the Court finds that the instant
petition is dismissible.

(2)
​NO. A final order is one that which disposes of the whole subject matter or terminates a
particular proceeding or action, leaving nothing to be done but to enforce by execution what has
been determined. Upon the other hand, an order is interlocutory if it does not dispose of a case
completely, but leaves something more to be done upon its merits. The RTC Decision is beyond
cavil interlocutory in nature. It is essentially a denial of petitioners’ motion to quash because it
leaves something more to be done x x x, i.e., the continuation of the criminal proceedings until
the guilt or innocence of the accused is determined. Specifically, the MeTC has yet to arraign the
petitioners, then proceed to trial and finally render the proper judgment.
​Petitioners contend that the PNP Crime Laboratory Questioned Document Report,
submitted as evidence by respondent to the prosecutor’s office, showed that the findings therein
are not conclusive and, thus, insufficient to support a finding of probable cause.
​The Court is not persuaded. It is clear from a perusal of the cited PNP Crime Laboratory
Questioned Document Report No. 048-03 that the document examiner found that the signatures
appearing in the questioned Deed of Sale as compared to the standard signatures “reveal
divergences in the manner of execution and stroke structure [which is] an indication that they
WERE NOT WRITTEN BY ONE AND THE SAME PERSON.” The Court agrees with the
prosecutor’s pronouncement in its Resolution dated September 22, 2003, that although the
findings of the PNP Crime Laboratory were qualified by the statement contained in the Report
that “no definite conclusion can be rendered due to the fact that questioned signatures are
photocopies wherein minute details are not clearly manifested,” the fact that an expert witness
already found that the questioned signatures were not written by one and the same person already
creates probable cause to indict petitioners for the crime of falsification of public document.
In the instant case, the Court finds no justification to depart from the ruling of the RTC that the
offense charged was committed and that herein petitioners are probably guilty thereof.
With respect to respondent’s legal personality to appeal the Order of the MeTC, suffice it to say
that the appeal filed with the RTC was made with the express conformity of the public prosecutor
who handles the case.
​It is wrong for petitioners to argue that it is the OSG which has authority to file an appeal
with the RTC. Section 35 (l), Chapter 12, Title III of Book IV of Executive Order No. 292,
otherwise known as the Administrative Code of 1987, mandates the OSG to represent “the
Government in the Supreme Court and the Court of Appeals in all criminal proceedings.” On the
other hand, Section 11 of Presidential Decree No. 1275, entitled “Reorganizing the Prosecution
Staff of the Department of Justice and the Offices of the Provincial and City Fiscals,
Regionalizing the Prosecution Service, and Creating the National Prosecution Service,” which
was the law in force at the time the appeal was filed, provides that the provincial or the city fiscal
(now referred to as prosecutor) “shall have charge of the prosecution of all crimes, misdemeanors
and violations of city or municipal ordinances in the courts of such province or city and shall
therein discharge all the duties incident to the institution of criminal prosecutions.” In
consonance with the above-quoted provision, it has been held by this Court that the fiscal
represents the People of the Philippines in the prosecution of offenses before the trial courts at
the metropolitan trial courts, municipal trial courts, municipal circuit trial courts and the regional
trial courts. Since the appeal, in the instant case was made with the RTC of Manila, it is clear that
the City Prosecutor or his assistant (in this case, the Assistant City Prosecutor) had authority to
​file the same.
​Moreover, petitioners’ reliance on Presidential Decree No. 911 is misplaced, as the cited
provision refers only to cases where the assistant fiscal or state prosecutor’s power to file an
information or dismiss a case is predicated or conditioned upon the prior authority or approval of
the provincial or city fiscal or the Chief State Prosecutor. There is nothing in the said law which
provides that in cases of appeal an Assistant City Prosecutor or a State Prosecutor may file the
same only upon prior authority or approval of the City Prosecutor or the Chief State Prosecutor.
Stated differently, unless otherwise ordered, an Assistant City Prosecutor or a State Prosecutor
may file an appeal with the RTC, questioning the dismissal by the MeTC of a case for lack of
probable cause, even without prior authority or approval of the City Prosecutor or the Chief State
Prosecutor.
BURGUNDY REALTY CORPORATION VS REYES

DOCTRINE:
​It must be remembered that the finding of probable cause was made after conducting a
preliminary investigation. A preliminary investigation constitutes a realistic judicial appraisal of
the merits of a case. Its purpose is to determine whether (a) a crime has been committed; and (b)
whether there is a probable cause to believe that the accused is guilty thereof.
​It is not disputed that decisions or resolutions of prosecutors are subject to appeal to the
Secretary of Justice who, under the Revised Administrative Code, exercises the power of direct
control and supervision over said prosecutors; and who may thus affirm, nullify, reverse or
modify their rulings. Review as an act of supervision and control by the justice secretary over the
fiscals and prosecutors finds basis in the doctrine of exhaustion of administrative remedies which
holds that mistakes, abuses or negligence committed in the initial steps of an administrative
activity or by an administrative agency should be corrected by higher administrative authorities,
and not directly by courts.
​This Court need not overemphasize that in a preliminary investigation, the public
prosecutor merely determines whether there is probable cause or sufficient ground to engender a
well-founded belief that a crime has been committed, and that the respondent is probably guilty
thereof and should be held for trial. It does not call for the application of rules and standards of
proof that a judgment of conviction requires after trial on the merits. The complainant need not
present at this stage proof beyond reasonable doubt. A preliminary investigation does not require
a full and exhaustive presentation of the parties’ evidence. Precisely, there is a trial to allow the
reception of evidence for both parties to substantiate their respective claims.

FACTS:
​Private respondent Josefa “Jing” C. Reyes (Reyes), sometime in 1996, offered her
services to petitioner as the latter’s real estate agent in buying parcels of land in Calamba,
Laguna, which are to be developed into a golf course. She informed petitioner that more or less
ten (10) lot owners are her clients who were willing to sell their properties. Convinced of her
representations, petitioner released the amount of P23,423,327.50 in her favor to be used in
buying those parcels of land. Reyes, instead of buying those parcels of land, converted and
misappropriated the money given by petitioner to her personal use and benefit. Petitioner sent a
formal demand for Reyes to return the amount of P23,423,327.50, to no avail despite her receipt
of the said demand. As such, petitioner filed a complaint for the crime of Estafa against Reyes
before the Assistant City Prosecutor’s Office of Makati City.
​Reyes, while admitting that she acted as a real estate agent for petitioner, denied having
converted or misappropriated the involved amount of money. She claimed that the said amount
was used solely for the intended purpose and that it was petitioner who requested her services in
procuring the lots. According to her, it was upon the petitioner’s prodding that she was
constrained to contact her friends who were also into the real estate business, including one
named Mateo Elejorde. Meanwhile, Reyes received information that her sub-broker Mateo
Elejorde had been depositing the involved money entrusted to him under his personal account.
On March 28, 2000, through a board resolution, petitioner allegedly authorized Reyes to
institute, proceed, pursue and continue with whatever criminal or civil action against Mateo
Elejorde, or such person to whom she may have delivered or entrusted the money she had
received in trust from the firm, for the purpose of recovering such money. Thus, Reyes filed a
complaint for the crime of estafa against Mateo Elejorde before the City Prosecutor’s Office of
Makati City.
​Thereafter, an Information for the crime of Estafa under Article 315, par. 1 (b) of the
Revised Penal Code (RPC) was filed against Reyes and raffled before the RTC Makati City.
Undeterred, Reyes filed a petition for review before the Department of Justice (DOJ), but it was
dismissed by the Secretary of Justice. Aggrieved, Reyes filed a motion for reconsideration, and
in a Resolution on the said motion was granted.
​Petitioner filed a motion for reconsideration, but was denied by the Secretary of Justice.
Eventually, petitioner filed a petition for certiorari under Rule 65 of the Rules of Court with the
CA. The latter, however, affirmed the questioned Resolutions of the Secretary of Justice. Hence,
the petition for review to the Supreme Court.

ISSUE:
​Whether of not the CA erred in not finding that the element of misappropriation was not
sufficiently established in the case but instead concurred with the decision of the DOJ Secretary;

HELD:
​It is not disputed that decisions or resolutions of prosecutors are subject to appeal to the
Secretary of Justice who, under the Revised Administrative Code,9 exercises the power of direct
control and supervision over said prosecutors; and who may thus affirm, nullify, reverse or
modify their rulings. Review as an act of supervision and control by the justice secretary over the
fiscals and prosecutors finds basis in the doctrine of exhaustion of administrative remedies which
holds that mistakes, abuses or negligence committed in the initial steps of an administrative
activity or by an administrative agency should be corrected by higher administrative authorities,
and not directly by courts.
​In the present case, after review and reconsideration, the Secretary of Justice reversed the
investigating prosecutor’s finding of probable cause that all the elements of the crime of estafa
are present. Estafa, under Article 315 (1) (b) of the Revised Penal Code, is committed by―
ART. 315. Swindling (estafa).―Any person who shall defraud another by any of the means
mentioned herein below:
1. With unfaithfulness or abuse of confidence, namely: (a) xxx
(b) By misappropriating or converting, to the prejudice of another, money, goods, or any other
personal property received by the offender in trust or on commission, or for administration, or
under any other obligation involving the duty to make delivery of or to return the same, even
though such obligation be totally or partially guaranteed by a bond; or by denying having
received such money, goods, or other property; x x x
In reversing the finding of probable cause that the crime of estafa has been committed, the
Secretary of Justice reasoned out that, [the] theory of conversion or misappropriation is difficult
to sustain and that under the crime of estafa with grave abuse of confidence, the presumption is
that the thing has been devoted to a purpose or is different from that for which it was intended
but did not take place in this case. The CA, in sustaining the questioned resolutions of the
Secretary of Justice, ruled that the element of misappropriation or conversion is wanting. It
further ratiocinated that the demand for the return of the thing delivered in trust and the failure of
the accused to account for it, are circumstantial evidence of misappropriation, however, the said
presumption is rebuttable and if the accused is able to satisfactorily explain his failure to produce
the thing delivered in trust, he may not be held liable for estafa.
​It must be remembered that the finding of probable cause was made after conducting a
preliminary investigation. A preliminary investigation constitutes a realistic judicial appraisal of
the merits of a case. Its purpose is to determine whether (a) a crime has been committed; and (b)
whether there is a probable cause to believe that the accused is guilty thereof.
​This Court need not overemphasize that in a preliminary investigation, the public
prosecutor merely determines whether there is probable cause or sufficient ground to engender a
well-founded belief that a crime has been committed, and that the respondent is probably guilty
thereof and should be held for trial. It does not call for the application of rules and standards of
proof that a judgment of conviction requires after trial on the merits. The complainant need not
present at this stage proof beyond reasonable doubt. A preliminary investigation does not require
a full and exhaustive presentation of the parties’ evidence. Precisely, there is a trial to allow the
reception of evidence for both parties to substantiate their respective claims.
​A review of the records would show that the investigating prosecutor was correct in
finding the existence of all the elements of the crime of estafa. Reyes did not dispute that she
received in trust the amount of P23,423,327.50 from petitioner as proven by the checks and
vouchers to be used in purchasing the parcels of land. Petitioner wrote a demand letter for Reyes
to return the same amount but was not heeded. Hence, the failure of Reyes to deliver the titles or
to return the entrusted money, despite demand and the duty to do so, constituted prima facie
evidence of misappropriation.
​To reiterate, probable cause has been defined as the existence of such facts and
circumstances as would excite the belief in a reasonable mind, acting on the facts within the
knowledge of the prosecutor, that the person charged was guilty of the crime for which he was
prosecuted.22 Probable cause is a reasonable ground of presumption that a matter is, or may be,
well founded on such a state of facts in the mind of the prosecutor as would lead a person of
ordinary caution and prudence to believe, or entertain an honest or strong suspicion, that a thing
is so. The term does not mean “actual or positive cause” nor does it import absolute certainty. It
is merely based on opinion and reasonable belief. Thus, a finding of probable cause does not
require an inquiry into whether there is sufficient evidence to procure a conviction. It is enough
that it is believed that the act or omission complained of constitutes the offense charged.
​WHEREFORE, premises considered, the present Petition is hereby GRANTED and,
accordingly, the Decision and Resolution of the Court of Appeals is REVERSED and SET
ASIDE.

HEIRS OF NESTOR TRIA VS OBIAS

DOCTRINE:
The justice secretary is not precluded from exercising his power of review over the investigating
prosecutor even after the information has already been filed in court; The justice secretary’s
subsequent resolution withdrawing the information or dismissing the case does not cause the
court to lose jurisdiction over the case.

FACTS:
​On May 22, 1998, at around 10:00 o’clock in the morning at the Pili Airport in
Camarines Sur, Engr. Nestor Tria, Regional Director of the Department of Public Works and
Highways (DPWH), Region V and concurrently Officer-In-Charge of the 2nd Engineering
District of Camarines Sur, was shot by a gunman while waiting to board his flight to Manila. He
was brought to a hospital but died the following day from the lone gunshot wound on his nape.
Subsequently, the incident was investigated by the NBI.
​On July 31, 1998, NBI Regional Director Alejandro R. Tenerife, Chairman of Task Force
Tria, recommended to the Provincial Prosecutor of Camarines Sur the indictment of Roberto
Aclan, Juanito, Ona and Atty. Epifania "Fanny" Gonzales- Obias, for the murder of Engr. Tria.
On the basis of statements given by 26 individuals, autopsy and ballistic examination reports,
and relevant documents gathered the NBI found that: ACLAN and ONA had been conducting an
almost daily stakeout, for about two weeks prior to the incident, at Dir. TRIA’s office. They
would observe TRIA’s arrival and departure from office and would even ask the security guard
on duty if TRIA has already arrive or left the office.
​Around 8:00 o’clock in the morning of May 22, 1998, ACLAN and ONA were spotted in
their usual places at the DPWH Office. Shortly after, Administrative Officer JOSE PECUNDO
announced to those who had some documents for signature of Director TRIA to proceed to Pili
Airport where TRIA would sign them before leaving for Manila. Upon hearing this,
​ACLAN and ONA left hurriedly on board a red motorcycle. Shortly after 10:00 a.m. on
that day, Director TRIA arrived at the Airport. After signing some documents at the parking lot
he proceeded towards the pre-departure area on the second floor of the airport building. ONA,
who was waiting on the stairway, immediately followed TRIA as the latter was going up the
stairs. As TRIA was approaching the pre-departure area he was met by Atty. [E]PIFANIA
OBIAS who shook his hands and started conversing with him. It was at this juncture that a
gunshot rang out and TRIA dropped like a log on the floor, bleeding profusely from a gunshot
wound at the back of his head. Atty. EPIFANIA OBIAS, on the other hand, admitted that she was
with ACLAN in the early morning of May 22, 1998; that at about 7:00 a.m. on that day she went
to the residence of Director TRIA at Liboton, Naga City, had a brief talk with the latter and left
immediately after agreeing to meet at the airport later on. She also volunteered the information
that ROBERTO ACLAN was not the gunman who had fired the fatal shot at Director TRIA. She
was also the last person seen talking with Director TRIA when the latter was gunned down. A
practicing lawyer, Atty. OBIAS also engages herself in real estate business on the side.
​In 1997 she had brokered a sale of real estate between and among spouses JEREMIAS, as
Vendors, and Spouses NESTOR and PURA TRIA, as Vendees, over a land in Balatas, Naga City.
It was Atty. OBIAS who received, for and in behalf of the vendors, the full payment of P2.8
Million of the sale but the latter deliberately avoided the TRIA family and, despite verbal and
written demands, she failed and refused, as she still fails and refuses, to fulfill her legal
obligation to the TRIA family. During the preliminary investigation conducted by the Office of
the Provincial Prosecutor, respondent filed her Counter- Affidavit denying that she was in
anyway involved with the killing of Engr. Tria and denied most of the allegations made by the
NBI.
​On July 2, 1999, the Office of the Provincial Prosecutor of Camarines Sur issued a
resolution8 directing the filing of an information for murder against Aclan and Ona but
dismissing the case for insufficiency of evidence as against herein respondent, Atty. Epifania
Obias. This was, however, modified on January 25, 2000 by then Justice Secretary Serafin
Cuevas directing the Provincial Prosecutor to include respondent in the information for murder
filed against Aclan and Ona. Respondent along with Aclan and Ona filed a motion for
reconsideration of the DOJ’s January 25, 2000 resolution. In the meantime, the information
charging Aclan and Ona has already been filed with the Regional Trial Court (RTC) of Pili,
Camarines Sur. Upon request however, the venue was transferred to the RTC Quezon City by
resolution of this Court in A.M. No. 00-3145-RTC.16 After so much petition and appeal, the
DOJ eventually directed the Provincial Prosecutor to forward the records of the case to the Office
of the President in compliance. In his Order dated March 24, 2004, Presidential Assistant Manuel
C. Domingo granted respondent’s motion for reconsideration and reversed the DOJ resolutions.
The OP concluded there was no interlocking circumstantial evidence of respondent’s acts before,
during and after the killing of Engr. Tria that would establish conspiracy among Aclan, Ona and
respondent to commit the crime. Accordingly, the case against respondent was dismissed for
insufficiency of evidence. Petitioners filed a series of motions, which reached the CA, who
denied their petition. Hence, this appeal.

ISSUE/S:
16. Whether non-referral by the OP to the DOJ of the appeal or motion for reconsideration filed
by the respondent had deprived them of the opportunity to confront and cross-examine the
witnesses on those affidavits belatedly submitted by the respondent. –-- No.
17. Whether the CA gravely abused its discretion in affirming the OP’s reversal of the ruling of
the Secretary of Justice. –-- Yes.
HELD:
​Under the procedure for preliminary investigation provided in Section 3, Rule 112 of the
Revised Rules of Criminal Procedure, as amended, in case the investigating prosecutor conducts
a hearing where there are facts and issues to be clarified from a party or witness, "[t]he parties
can be present at the hearing but without the right to examine or cross-examine. They may,
however, submit to the investigating officer questions which may be asked to the party or witness
concerned." Hence, the non-referral by the OP to the DOJ of the motion for reconsideration of
respondent, in the exercise of its discretion, did not violate petitioners’ right to due process. The
findings of the prosecutor with respect to the existence or non-existence of probable cause is
subject to the power of review by the DOJ.
​Indeed, the Secretary of Justice may reverse or modify the resolution of the prosecutor,
after which he shall direct the prosecutor concerned either to file the corresponding information
without conducting another preliminary investigation, or to dismiss or move for dismissal of the
complaint or information with notice to the parties. In reversing the DOJ’s finding of probable
cause, the OP found merit in the argument of the respondent that the DOJ’s finding that she was
with Aclan when she went to the residence of Engr. Tria early in the morning of May 22, 1998,
was not sufficiently established. The OP gave more weight to the affidavit of Calayag stating that
Aclan was not around when they and respondent, among other visitors, were at Engr. Tria’s
house at that time -- than that account given by SA Eduarte, which was uncorroborated.
​As to the double sale allegedly committed by the respondent from which the latter’s
strong motive to liquidate Engr. Tria was inferred, the OP found this as a mere expression of
opinion by the investigators considering that Engr. Tria’s widow, Mrs. Pura Tria, categorically
admitted her knowledge of the said transaction. Neither was the OP persuaded by the NBI’s "kiss
of death" theory since it is but a customary way of greeting a friend to shake hands and hence it
cannot imply that respondent utilized this as a signal or identification for the gunman to shoot
Engr. Tria. Respondent’s alleged indifference immediately after Engr. Tria was gunned down
while conversing with her, was also negated by the affidavit of an employee of Philippine Air
Lines based at the Pili Airport, stating that right after the incident took place he saw respondent
in the radio room in shock and was being given water by another person.
​Considering the totality of evidence, the OP was convinced there was nothing suspicious
or abnormal in respondent’s behavior before, during and after the fatal shooting of Engr. Tria as
to engender a well-founded belief of her complicity with the killing of Engr. Tria. Petitioners,
however, maintain that the records are replete with abundant proof of respondent’s complicity in
the murder of Engr. Tria.
​Probable cause is defined as the existence of such facts and circumstances as would
excite the belief in a reasonable mind, acting on the facts within the knowledge of the prosecutor,
that the person charged was guilty of the crime for which he was prosecuted. It is a reasonable
ground of presumption that a matter is, or may be, well-founded, such a state of facts in the mind
of the prosecutor as would lead a person of ordinary caution and prudence to believe, or entertain
an honest or strong suspicion, that a thing is so.
​The term does not mean "actual and positive cause" nor does it import absolute certainty.
It is merely based on opinion and reasonable belief. A finding of probable cause merely binds
over the suspect to stand trial; it is not a pronouncement of guilt. On the other hand, conspiracy
exists when two or more persons come to an agreement concerning the commission of a felony
and decide to commit it. Direct proof of previous agreement to commit a crime is not necessary.
Conspiracy may be shown through circumstantial evidence, deduced from the mode and manner
in which the offense was perpetrated, or inferred from the acts of the accused themselves when
such lead to a joint purpose and design, concerted action, and community of interest.
​We reverse the OP’s ruling that the totality of evidence failed to establish a prima facie
case against the respondent as a conspirator in the killing of Engr. Tria. To begin with, whether or
not respondent actually conspired with Aclan and Ona need not be fully resolved during the
preliminary investigation. The absence or presence of conspiracy is factual in nature and
involves evidentiary matters. The same is better left ventilated before the trial court during trial,
where the parties can adduce evidence to prove or disprove its presence.
​Preliminary investigation is executive in character. It does not contemplate a judicial
function. It is essentially an inquisitorial proceeding, and often, the only means of ascertaining
who may be reasonably charged with a crime. Prosecutors control and direct the prosecution of
criminal offenses, including the conduct of preliminary investigation, subject to review by the
Secretary of Justice. The duty of the Court in appropriate cases is merely to determine whether
the executive determination was done without or in excess of jurisdiction or with grave abuse of
discretion. Resolutions of the Secretary of Justice are not subject to review unless made with
grave abuse.
​After a careful evaluation of the entire evidence on record, we find no such grave abuse
when the Secretary of Justice found probable cause to charge the respondent with murder in
conspiracy with Aclan and Ona.
​The following facts and circumstances established during preliminary investigation were
sufficient basis to incite reasonable belief in respondent’s guilt: (a) Motive - respondent had
credible reason to have Engr. Tria killed because of the impending criminal prosecution for
estafa from her double sale of his lot prior to his death, judging from the strong interest of Engr.
Tria’s family to run after said property and/or proceeds of the second sale to a third party; (b)
Access - respondent was close to Engr. Tria’s family and familiar with his work schedule, daily
routine and other transactions which could facilitate in the commission of the crime eventually
carried out by a hired gunmen, one of whom (Aclan) she and her father categorically admitted
being in her company while she visited Engr. Tria hours before the latter was fatally shot at the
airport; (c) Suspicious Behavior -- respondent while declaring such close personal relationship
with Engr. Tria and even his family, failed to give any satisfactory explanation why she reacted
indifferently to the violent killing of her friend while they conversed and shook hands at the
airport.
​Indeed, a relative or a friend would not just stand by and walk away from the place as if
nothing happened, as what she did, nor refuse to volunteer information that would help the
authorities investigating the crime, considering that she is a vital eyewitness. Not even a call for
help to the people to bring her friend quickly to the hospital. She would not even dare go near
Engr. Tria’s body to check if the latter was still alive. All the foregoing circumstances, in our
mind, and from the point of view of an ordinary person, lead to a reasonable inference of
respondent’s probable participation in the well-planned assassination of Engr. Tria.
​We therefore hold that the OP in reversing the DOJ Secretary’s ruling, and the CA in
affirming the same, both committed grave abuse of discretion. Clearly, the OP and CA arbitrarily
disregarded facts on record which established probable cause against the respondent. The
petition is GRANTED. The January 25, 2000 Resolution of then Justice Secretary Serafin
Cuevas is hereby REINSTATED and UPHELD.

UY VS JAVELLANA

DOCTRINE:
​The Revised Rule on Summary Procedure does not provide for a preliminary
investigation prior to the filing of a criminal case under said Rule.
​Section 1, Rule 112 of the Revised Rules of Criminal Procedure only requires that a
preliminary investigation be conducted before the filing of a complaint or information for an of
ense where the penalty prescribed by law is at least four (4) years, two (2) months and one (1)
day without regard to the fine.

FACTS:
​This administrative case arose from a verified complaint for "gross ignorance of the law
and procedures, gross incompetence, neglect of duty, conduct improper and unbecoming of a
judge, grave misconduct and others," filed by Public Attorneys Gerlie M. Uy (Uy) and Ma.
Consolacion T. Bascug (Bascug) of the (PAO), La Carlotta District, against Presiding Judge
Javellana of the MeTC, La Castellana, Negros Occidental. Public Attorneys Uy and Bascug
alleged the following in their complaint:
​Judge Javellana was grossly ignorant of the Revised Rule on Summary Procedure. Public
Attorneys Uy and Bascug cited several occasions as examples: In Crim. Case No. 04-097,
entitled People v. Cornelio, for Malicious Mischief, Judge Javellana issued a warrant of arrest
after the filing of said case despite Section 16 of the Revised Rule on Summary Procedure; Crim.
Case No. 04-075, entitled People v. Celeste, et al., for Trespass to Dwelling, Judge Javellana did
not grant the motion to dismiss for non-compliance with the Lupon requirement under Sections
18 and 19(a) of the Revised Rule on Summary Procedure, insisting that said motion was a
prohibited pleading; Also in People v. Celeste, et al., Judge Javellana refused to dismiss outright
the complaint even when the same was patently without basis or merit, as the affidavits of therein
complainant and her witnesses were all hearsay evidence; and Crim. Case No. 02-056, entitled
People v. Lopez, et al., for Malicious Mischief, Judge Javellana did not apply the Revised Rule
on Summary Procedure and, instead, conducted a preliminary examination and preliminary
investigation in accordance with the Revised Rules of Criminal Procedure, then set the case for
arraignment and pre-trial, despite confirming that therein complainant and her witnesses had no
personal knowledge of the material facts alleged in their affidavits, which should have been a
ground for dismissal of said case.
​Judge Javellana violated Section 6(b), Rule 112 of the Revised Rules of Criminal
Procedure and issued warrants of arrest without propounding searching questions to the
complainants and their witnesses to determine the necessity of placing the accused under
immediate custody.
​As a result, Judge Javellana issued warrants of arrest even when the accused had already
voluntarily surrendered or when a warrantless arrest had been effected. Judge Javellana failed to
observe the constitutional rights of the accused as stated in Section 12(1), Article III of the
Constitution. Judge Javellana set Crim. Case No. 03-097, entitled People v. Bautista, for
preliminary investigation even when the accused had no counsel, and proceeded with said
investigation without informing the accused of his rights to remain silent and to have a counsel
Judge Javellana stressed that the charges against him were baseless and malicious; and the acts
being complained of involved judicial discretion and, thus, judicial in nature and not the proper
subject of an administrative complaint. Consequently, Judge Javellana sought the dismissal of the
instant complaint against him. The Office of the Court Administrator (OCA), in its report, found
Judge Javellana liable for gross ignorance of the law or procedure when he did not apply the
Revised Rule on Summary Procedure in cases appropriately covered by said Rule.

ISSUE:
​Whether or not Judge Javellana was grossly ignorant of the Revised Rule on Summary
Procedure.

HELD:
​Judge Javellana committed a blatant error in denying the Motion to Dismiss filed by the
accused in People v. Celeste, et al. and in insisting that said Motion was a prohibited pleading,
even though the case was never previously referred to the Lupong Tagapamayapa as required by
Sections 18 and 19(a) ofthe Revised Rule on Summary Procedure. A case which has not been
previously referred to the Lupong Tagapamayapa shall be dismissed without prejudice. A motion
to dismiss on the ground of failure to comply with the Lupon requirement is an exception to the
pleadings prohibited by the Revised Rule on Summary Procedure. Given the express provisions
of the Revised Rule on Summary Procedure, we find irrelevant Judge Javellana’s argument that
referral to the Lupon is not a jurisdictional requirement. The following facts are undisputed:
People v. Celeste, et al. were not referred to the Lupon, and the accused filed a Motion to
Dismiss based on this ground. Judge Javellana should have allowed and granted the Motion to
Dismiss (albeit without prejudice) filed by the accused in People v. Celeste, et al. (hindi ko sure)
Judge Javellana did not provide any reason as to why he needed to conduct a preliminary
investigation in People v. Lopez, et al. Judge Javellana cannot be allowed to arbitrarily conduct
proceedings beyond those specifically laid down by the Revised Rule on Summary Procedure,
thereby lengthening or delaying the resolution of the case, and defeating the express purpose of
said Rule. Without any showing that the accused in People v. Cornelio and People v. Lopez, et al.
were charged with the special cases of malicious mischief particularly described in Article 328 of
the Revised Penal Code the appropriate penalty for the accused would be arresto mayor in its
medium and maximum periods which under Article 329(a) of the Revised Penal Code, would be
imprisonment for two (2) months and one (1) day to six (6) months. Clearly, these two cases
should be governed by the Revised Rule on Summary Procedure.

PEOPLE VS VALENCIA

DOCTRINE:
18. Information can be filed without a preliminary investigation against an accused arrested
without warrant.
19. Preliminary investigation; deemed waived when not invoked.
FACTS:
​Accused-appellant Alejandro Valencia appeals the Decision of the Regional Trial Court
of Manila, in 2 Criminal Cases convicting him of Homicide with the use of an unlicensed firearm
and Less Serious Physical Injuries.
​Arlyn Barredo-Jimenez, her two children, Annabelle and Samuel, Jr., aged five and three,
respectively, and her mother, are residents of 2008 F. Muñoz St., Paco, Manila. At about 9:00
p.m. of March 19, 1989, as she was about to eat supper, she noticed appellant standing five steps
away from the open door of her house and holding a sumpak, a homemade shotgun. Seized with
fear, she closed the door. After a few moments, she heard a burst of gunfire. This was followed
by cries of pain from her children inside the house. Seeing her children bloodied, she
immediately went outside and shouted for help. As she did so, she saw appellant running away,
carrying the sumpak. Two neighbors assisted Jimenez in bringing the injured children to the
Philippine General Hospital. That same evening, Patrolman Renato Marquez, a homicide
investigator, interviewed Jimenez at the hospital about the shooting incident. Since she was still
experiencing shock over the incident Jimenez forgot to mention the name of appellant as the one
who shot her children. Acting on the report of a barangay tanod, Patrolmen Roberto Cajiles,
Romeo de la Peña and Carlos Castañeda, assigned at the Ong Detachment, Police Station No. 5,
conducted an investigation of the shooting incident in the house of Jimenez. At the time, Jimenez
and her injured children were already in the hospital. Nevertheless, Pat. Cajiles was able to
interview the mother of Jimenez, the barangay captain, a certain Josie, and appellant’s brother,
Rolando, who all mentioned appellant as the gunwielder. Moreover, the policemen discovered
the presence of six pellet holes and one big hole with the size of the circumference of a shotgun
bullet on the door of the house of Jimenez. Three pellets were also found at the crime scene.
​Early next morning, the three policemen were led by Rolando Valencia to the residence
of Sonia Castillo, his aunt, where he believed appellant was sleeping. The police apprehended
appellant there and took him to the Ong Detachment for initial investigation. He was indorsed to
the police headquarters for further investigation in the evening of March 22, 1989. At 12:20 a.m.
of the following day, one of the injured children, Annabelle, died as a result of the gunshot
wounds she suffered. The other child, Samuel Jr., who was shot in the right forearm, was
discharged from the hospital one week after the incident, but needed 2 more weeks for healing.
On March 26, 1989, Arlyn Jimenez executed a sworn statement wherein she identified appellant
as the culprit. On March 30, 1989, a certain Ramon Bacnotan executed a sworn statement and
turned over to the police the sumpak allegedly used by appellant in the shooting of the two
children. 2 Criminal Cases were filed against Valencia, for Homicide with the use of an
unlicensed firearm and Less Serious Physical Injuries. When arraigned, the accused-appellant
pleaded "Not Guilty." Trial then proceeded resulting in accused-appellant’s conviction.

ISSUE/S:
​WON the finding of the court of Accused-appellant’s guilt beyond reasonable doubt is
correct in spite of the fact that there was allegedly no preliminary investigation, and that no
sufficient evidence exists proving his guilt.

HELD:
​YES. A person who is lawfully arrested, without a warrant pursuant to paragraph 1(b),
Section 5, Rule 113, Rules of Court 23 should be delivered to the nearest police station and
proceeded against in accordance with Rule 112, Section 7. 24 Under said Section 7, Rule 112, 25
the prosecuting officer can file the Information in court without a preliminary investigation,
which was done in the accused-appellant’s case.
​Since the records do not show whether the accused-appellant asked for a preliminary
investigation after the case had been filed in court, as in fact, the accused-appellant signified his
readiness to be arraigned, 27 the Court can only conclude that he waived his right to have a
preliminary investigation, 28 when he did, in fact, pleaded "Not Guilty" upon his arraignment.

PCGG VS NAVARRO-GUTIERREZ

DOCTRINE:
20. Probable cause, for the purpose of filing a criminal information, has been defined as such
facts as are sufficient to engender a well-founded belief that a crime has been committed and
that respondent is probably guilty thereof. The term does not mean "actual or positive cause"
nor does it import absolute certainty. It is merely based on opinion and reasonable belief.
Probable cause does not require an inquiry whether there is sufficient evidence to procure a
conviction. It is enough that it is believed that the act or omission complained of constitutes
the offense charged.
21. Preliminary investigation is merely an inquisitorial mode of discovering whether or not there
is reasonable basis to believe that a crime has been committed and that the person charged
should be held responsible for it. Being merely based on opinion and belief, a finding of
probable cause does not require an inquiry as to whether there is sufficient evidence to secure
a conviction. "[A preliminary investigation] is not the occasion for the full and exhaustive
display of [the prosecution's] evidence. The presence and absence of the elements of the
crime is evidentiary in nature and is a matter of defense that may be passed upon after a full-
blown trial on the merits." Hence, "the validity and merits of a party's defense or accusation,
as well as the admissibility of testimonies and evidence, are better ventilated during trial
proper than at the preliminary investigation level."
FACTS:
​PCGG filed against former officers/directors of the Development Bank of the Philippines
(DBP), namely, Ferry, Tengco, Zosa, Zalamea, Castell, and Sison, as well as former
officers/stockholders of National Galleon Shipping Corporation (Galleon), namely, Cuenca,
Tinio, and Roque charging them of violating Sections 3 (e) and (g) of RA 3019.
​PCGG alleged that on October 8, 1992, then President Fidel V. Ramos (President Ramos)
issued Administrative Order No. 13, creating the Presidential Ad Hoc Fact-Finding Committee
on Behest Loans (Ad Hoc Committee) in order to identify various anomalous loans entered into
by the Philippine Government in the past.
​Thereafter, the Ad Hoc Committee, with the assistance of a Technical Working Group
(TWG) examined and studied documents relative to loan accounts extended by GFIs to various
corporations during the regime of the late President Ferdinand E. Marcos (President Marcos) -
one of which is the loan account granted by the DBP to Galleon.
​TWG found anomalies after examining the loans of Galleon. PCGG then files a case
against the individual officers. Only Roque, Zalamea, Tengco, and Castell filed their counter-
affidavits. In their defense, Roque stated that he was only a minor stockholder and in no position
to influence such loan. Zamalea was only chairman of DBP after the transaction. Tenco argued
that the charges already prescribed, and Castell argued that his job is only supervision of
employees.
​Ombudsman found no probable cause against them hence the case was dismissed on the
basis that pieces of evidence attached to the case records were not sufficient to establish probable
cause against the individual respondents, considering that the documents presented by the PCGG
consisted mostly of executive summaries and technical reports, which are hearsay, self-serving,
and of little probative value. PCGG move for reconsideration but was also denied.

ISSUE:
​Whether or not there was a grave abuse of discretion on the part of the Ombudsman in
finding no probable cause to indict the respondents.

HELD:
​Yes. It must be stressed that the Court has consistently refrained from interfering with the
discretion of the Ombudsman to determine the existence of probable cause and to decide whether
or not an Information should be filed. Nonetheless, the Court is not precluded from reviewing the
Ombudsman's action when there is a charge of grave abuse of discretion. Grave abuse of
discretion implies a capricious and whimsical exercise of judgment tantamount to lack of
jurisdiction. The Ombudsman's exercise of power must have been done in an arbitrary or
despotic manner which must be so patent and gross as to amount to an evasion of a positive duty
or a virtual refusal to perform the duty enjoined or to act at all in contemplation of law. The
Court's pronouncement in Ciron v. Gutierrez is instructive on this matter, to wit:
​xxx this Court's consistent policy has been to maintain noninterference in the
determination of the Ombudsman of the existence of probable cause, provided there is no grave
abuse in the exercise of such discretion. This observed policy is based not only on respect for the
investigatory and prosecutory powers granted by the Constitution to the Office of the
Ombudsman but upon practicality as well. Otherwise, the functions of the Court will be seriously
hampered by innumerable petitions assailing the dismissal of investigatory proceedings
conducted by the Office of the Ombudsman with regard to complaints filed before it, in much the
same way that the courts would be extremely swamped with cases if they could be compelled to
review the exercise of discretion on the part of the fiscals or prosecuting attorneys each time they
decide to file an information in court or dismiss a complaint by a private complainant.
​In this regard, it is worthy to note that the conduct of preliminary investigation
proceedings - whether by the Ombudsman or by a public prosecutor - is geared only to determine
whether or not probable cause exists to hold an accused-respondent for trial for the supposed
crime that he committed. In Fenequito v. Vergara, Jr., the Court defined probable cause and the
parameters in finding the existence thereof in the following manner, to wit:
​Probable cause, for the purpose of filing a criminal information, has been defined as such
facts as are sufficient to engender a well-founded belief that a crime has been committed and that
respondent is probably guilty thereof. The term does not mean "actual or positive cause" nor
does it import absolute certainty. It is merely based on opinion and reasonable belief. Probable
cause does not require an inquiry whether there is sufficient evidence to procure a conviction. It
is enough that it is believed that the act or omission complained of constitutes the offense
charged.
​A finding of probable cause needs only to rest on evidence showing that, more likely than
not, a crime has been committed by the suspects. It need not be based on clear and convincing
evidence of guilt, not on evidence establishing guilt beyond reasonable doubt, and definitely not
on evidence establishing absolute certainty of guilt. In determining probable cause, the average
man weighs facts and circumstances without resorting to the calibrations of the rules of evidence
of which he has no technical knowledge. He relies on common sense. What is determined is
whether there is sufficient ground to engender a well-founded belief that a crime has been
committed, and that the accused is probably guilty thereof and should be held for trial. It does
not require an inquiry as to whether there is sufficient evidence to secure a conviction.
(Emphases and underscoring supplied)
​Preliminary investigation is merely an inquisitorial mode of discovering whether or not
there is reasonable basis to believe that a crime has been committed and that the person charged
should be held responsible for it. Being merely based on opinion and belief, a finding of probable
cause does not require an inquiry as to whether there is sufficient evidence to secure a
conviction. "[A preliminary investigation] is not the occasion for the full and exhaustive display
of [the prosecution's] evidence. The presence and absence of the elements of the crime is
evidentiary in nature and is a matter of defense that may be passed upon after a full-blown trial
on the merits." Hence, "the validity and merits of a party's defense or accusation, as well as the
admissibility of testimonies and evidence, are better ventilated during trial proper than at the
preliminary investigation level."
​Guided by the foregoing considerations, the Court finds that the Ombudsman gravely
abused its discretion in dismissing the criminal complaint against individual respondents for lack
of probable cause, as will be explained hereunder.
​As already stated, individual respondents were accused of violating Section 3 (e) of RA
3019, the elements of which are as follows: (a) that the accused must be a public officer
discharging administrative, judicial, or official functions (or a private individual acting in
conspiracy with such public officers); (b) that he acted with manifest partiality, evident bad faith,
or inexcusable negligence; and (c) that his action caused any undue injury to any party, including
the government, or giving any private party unwarranted benefits, advantage, or preference in the
discharge of his functions. In the same vein, they were likewise charged with violation of Section
3 (g) of the same law, which has the following elements: (a) that the accused is a public officer;
(b) that he entered into a contract or transaction on behalf of the government; and (c) that such
contract or transaction is grossly and manifestly disadvantageous to the government. Notably,
private individuals may also be charged with violation of Section 3 (g) of RA 3019 if they
conspired with public officers.
​Finally, it was error for the Ombudsman to simply discredit the TWG's findings contained
in the Executive Summary which were adopted by the Ad Hoc Committee for being hearsay,
self-serving, and of little probative value. It is noteworthy to point out that owing to the initiatory
nature of preliminary investigations, the technical rules of evidence should not be applied in the
course of its proceedings. In the recent case of Estrada v. Ombudsman, the Court declared that
hearsay evidence is admissible in determining probable cause in preliminary investigations
because such investigation is merely preliminary, and does not finally adjudicate rights and
obligations of parties. Citing a case decided by the Supreme Court of the United States, it was
held that probable cause can be established with hearsay evidence, as long as there is substantial
basis for crediting the hearsay, viz.:
​Justice Brion's pronouncement in Unilever that "the determination of probable cause does
not depend on the validity or merits of a party's accusation or defense or on the admissibility or
veracity of testimonies presented" correctly recognizes the doctrine in the United States that the
determination of probable cause can rest partially, or even entirely, on hearsay evidence, as long
as the person making the hearsay statement is credible. In United States v. Ventresca, the United
States Supreme Court held:
​While a warrant may issue only upon a finding of "probable cause," this Court has long
held that "the term 'probable cause' . . . means less than evidence which would justify
condemnation," x x x and that a finding of "probable cause" may rest upon evidence which is not
legally competent in a criminal trial, x x x As the Court stated in Brinegar v. United States x x x,
"There is a large difference between two things to be proved (guilt and probable cause), as well
as between the tribunals which determine them, and therefore a like difference in the quanta and
modes of proof required to establish them." Thus, hearsay may be the bases for issuance of the
warrant "so long as there ... [is] a substantial basis for crediting the hearsay." x x x And, in
Aguilar, we recognized that "an affidavit may be based on hearsay information and need not
reflect the direct
​personal observations of the affiant," so long as the magistrate is "informed of some of
the underlying circumstances" supporting the affiant's conclusions and his belief that any
informant involved "whose identity need not be disclosed..." was "credible" or his information
"reliable." x x x.
​Thus, probable cause can be established with hearsay evidence, as long as there is
substantial basis for crediting the hearsay. Hearsay evidence is admissible in determining
probable cause in a preliminary investigation because such investigation is merely preliminary,
and does not finally adjudicate rights and obligations of parties, x x x.

DE LIMA VS REYES
DOCTRINE:
​The prosecutor in a preliminary investigation does not determine the guilt or innocence of
the accused. He does not exercise adjudication nor rule-making functions. Preliminary
investigation is merely inquisitorial, and is often the only means of discovering the persons who
may be reasonably charged with a crime and to enable the fiscal to prepare his complaint or
information. It is not a trial of the case on the merits and has no purpose except that of
determining whether a crime has been committed and whether there is probable cause to believe
that the accused is guilty thereof. While the fiscal makes that determination, he cannot be said to
be acting as a quasi-court, for it is the courts, ultimately, that pass judgment on the accused, not
the fiscal.

FACTS:
​The Secretary of Justice has the discretion, upon motion or motu proprio, to act on any
matter that may cause a probable miscarriage of justice in the conduct of a preliminary
investigation. This action may include, but is not limited to, the conduct of a reinvestigation.
Furthermore, a petition for certiorari under Rule 65 questioning the regularity of preliminary
investigation becomes moot after the trial court completes its determination of probable cause
and issues a warrant of arrest.
​Dr. Gerardo Ortega (Dr. Ortega), was a veterinarian and anchor of several radio shows in
Palawan. On January 24, 2011, at around 10:30 am, he was shot dead inside the Baguio
Wagwagan Ukay-ukay in San PedroPuerto Princesa City, Palawan.
Marlon B. Recamata was arrested. On the same day, he made an extrajudicial confession
admitting that he shot Dr. Ortega and also implicated Rodolfo "Bumar" O. Edrad (Edrad),
Dennis C. Aranas, and Armando "Salbakotah" R. Noel, Jr. Edrad executed a Sinumpaang
Salaysay before the Counter-Terrorism Division of the National Bureau of Investigation where
he alleged that it was former Palawan Governor Mario Joel T. Reyes (former Governor Reyes)
who ordered the killing of Dr. Ortega.
​Secretary of Justice Leila De Lima issued Department Order No. 0918 creating a special
panel of prosecutors (First Panel) to conduct preliminary investigation.
​Dr. Ortega's wife, filed a Supplemental Affidavit-Complaint implicating former Governor
Reyes as the mastermind of her husband's murder On June 8, 2011, the First Panel concluded its
preliminary investigation and issued the Resolution dismissing the Affidavit-Complaint.
On September 7, 2011, the Secretary of Justice issued Department Order No. 710 creating a new
panel of investigators (Second Panel) to conduct a reinvestigation of the case "in the interest of
service and due process" to address the offer of additional evidence denied by the First Panel.
But it was also revoked.
​Pursuant to the revocation, Second Panel issued a Subpoena requiring former Governor
Reyes to appear before them and to submit his counter-affidavit and supporting evidence.
Dr. Ortega filed before the Secretary of Justice a Petition for Review (Ad Cautelam) assailing the
First Panel's Resolution.
​Governor Reyes filed before the Court of Appeals a Petition for Certiorari and
Prohibition with Prayer for a Writ of Preliminary Injunction and/or Temporary Restraining Order
assailing the creation of the Second Panel. He argued that the Secretary of Justice gravely abused
her discretion when she constituted a new panel. He also argued that the parties were already
afforded due process and that the evidence to be addressed by the reinvestigation was neither
new nor material to the case.
​On March 12, 2012, the Second Panel issued the Resolution finding probable cause and
recommending the filing of informations on all accused, including former Governor Reyes.
Regional Trial Court of Palawan subsequently issued warrants of arrest but the warrants were
ineffective since the Gov. Reyes allegedly left the country days before the warrants could be
served.
​Gov. Reyes filed a petition for review alleging that the resolution of the second panel was
void. CA affirmed. Court of Appeals stated that the Secretary of Justice had not shown the
alleged miscarriage of justice sought to be prevented by the creation of the Second Panel since
both parties were given full opportunity to present their evidence before the First Panel. It also
ruled that the evidence examined by the Second Panel was not additional evidence but "forgotten
evidence" that was already available before the First Panel during the conduct of the preliminary
investigation.

ISSUE:
​Whether the Court of Appeals erred in ruling that the Secretary of Justice committed
grave abuse of discretion when she issued Department Order No. 710.

HELD:
​The determination by the Department of Justice of the existence of probable cause is not
a quasi-judicial proceeding. However, the actions of the Secretary of Justice in affirming or
reversing the findings of prosecutors may still be subject to judicial review if it is tainted with
grave abuse of discretion.
​Under the Rules of Court, a writ of certiorari is directed against "any tribunal, board or
officer exercising judicial or quasi- judicial functions." A quasi-judicial function is "the action,
discretion, etc., of public administrative officers or bodies, who are required to investigate facts,
or ascertain the existence of facts, hold hearings, and draw conclusions from them, as a basis for
their official action and to exercise discretion of a judicial nature." Otherwise stated, an
administrative agency performs quasi-judicial functions if it renders awards, determines the
rights of opposing parties, or if their decisions have the same effect as the judgment of a court.
In a preliminary investigation, the prosecutor does not determine the guilt or innocence of an
accused. The prosecutor only determines "whether there is sufficient ground to engender a well-
founded belief that a crime has been committed and the respondent-is probably guilty thereof,
and should be held for trial." As such, the prosecutor does not perform quasi-judicial functions.
In Santos v. Go:
​The prosecutor in a preliminary investigation does not determine the guilt or innocence of
the accused. He does not exercise adjudication nor rule-making functions. Preliminary
investigation is merely inquisitorial, and is often the only means of discovering the persons who
may be reasonably charged with a crime and to enable the fiscal to prepare-his complaint or
information. It is not a trial of the case on the merits and has no purpose except that of
determining whether a crime has been committed and whether there is probable cause to believe
that the accused is guilty thereof. While the fiscal makes that determination, he cannot be said to
be acting as a quasi-court, for it. is the courts, ultimately, that pass judgment on the accused, not
the fiscal.
​Though some cases describe the public prosecutors power to conduct a preliminary
investigation as quasi-judicial in nature, this is true only to the extent that, like quasi-judicial
bodies, the prosecutor is an officer of the executive department exercising powers akin to those
of a court, and the similarity ends at this point. A quasi-judicial body is as an organ of
government other than a court and other than a legislature which affects the rights of private
parties through either adjudication or rule-making. A quasi-judicial agency performs
adjudicatory functions such that its awards, determine the rights of parties, and their decisions
have the same effect as judgments of a court. Such is not the case when a public prosecutor
conducts a preliminary investigation to determine probable cause to file an information against a
person charged with a criminal offense, or when the Secretary of Justice is reviewing the formers
order or resolutions.
​In Spouses Dacudao v. Secretary of Justice, a petition for certiorari, prohibition, and
mandamus was filed against the Secretary of Justice's issuance of a department order. The
assailed order directed all prosecutors to forward all cases already filed against Celso de los
Angeles of the Legacy Group to the Secretariat of the Special Panel created by the Department of
Justice.
​This court dismissed the petition on the ground that petitions for certiorari and
prohibition are directed only to tribunals that exercise judicial or quasi-judicial functions. The
issuance of the department order was a purely administrative or executive function of the
Secretary of Justice. While the Department of Justice may perform functions similar to that of a
court of law, it is not a quasi-judicial agency:
​The fact that the DOJ is the primary prosecution arm of the Government does not make it
a quasi-judicial office or agency. Its preliminary investigation of cases is not a quasi-judicial
proceeding. Nor does the DOJ exercise a quasi-judicial function when it reviews the findings of a
public prosecutor on the finding of probable cause in any case. Indeed, in Bautista v. Court of
Appeals, the Supreme Court has held that a preliminary investigation is not a quasi-judicial
proceeding, stating:
. . . [t]he prosecutor in a preliminary investigation does not determine the guilt or innocence of
the accused. He does not exercise adjudication nor rule-making functions. Preliminary
investigation is merely inquisitorial, and is often the only means of discovering the persons who
may be reasonably charged with a crime and to enable the fiscal to prepare his complaint or
information. It is not a trial of the case on the merits and has no purpose except that of
determining whether a crime has been committed and whether there is probable cause to believe
that the accused is guilty thereof. While the fiscal makes that determination, he cannot be said to
be acting as a quasi-court, for it is the courts, ultimately, that pass judgment on the accused, not
the fiscal.
​There may be some decisions of the Court that have characterized the public prosecutor's
power to conduct a preliminary investigation as quasi-judicial in nature. Still, this
characterization is true only to the extent that the public prosecutor, like a quasi-judicial body, is
an officer of the executive department exercising powers akin to those of a court of law.
But the limited similarity, between the public prosecutor and a quasi-judicial body quickly ends
there. For sure, a quasi- judicial body is an organ of government other than a court of law or a
legislative office that affects the rights of private parties through either adjudication or rule-
making; it performs adjudicatory functions, and its awards and adjudications determine the rights
of the parties coming before it; its decisions have the same effect as the judgments of a court of
law. In contrast, that is not the effect whenever a public prosecutor conducts a preliminary
investigation to determine probable cause in order to file a criminal information against a person
properly charged with the offense, or whenever the Secretary of Justice reviews the public
prosecutor's orders or resolutions.

RULE 113 – ARREST

SARAUM VS PEOPLE

DOCTRINE:
​The valid warrantless arrest gave the officers the right to search the shanty for objects
relating to the crime and seize the drug paraphernalia they found. In the course of their lawful
intrusion, they inadvertently saw the various drug paraphernalia. As these items were plainly
visible, the police officers were justified in seizing them. Considering that Saraum’s arrest was
legal, the search and seizure that resulted from it were likewise lawful. The various drug
paraphernalia that the police officers found and seized in the shanty are, therefore, admissible in
evidence for having proceeded from a valid search and seizure. Since the confiscated drug
paraphernalia are the very corpus delicti of the crime charged, the Court has no choice but to
sustain the judgment of conviction.

FACTS:
​Saraum was charged with violation of Section 12, Article II (Possession of Paraphernalia
for Dangerous Drugs) of Republic Act(R.A.)No. 9165, or theComprehensive Dangerous Drugs
Act of 2002.The accusatory portion of the Information reads:
​That on or about the 17th day of August, 2006, at about 12:45 A.M., The accused did
then and there have in his possession the following: 1 = One (1) lighter, 2 = One (1) rolled tissue
paper, 3 = One (1) aluminum tin foil, which are instruments and/or equipments fit or intended for
smoking, consuming, administering, ingesting, or introducing any dangerous drug into the body.
Sarum pleaded not guilty and was released due to his application of bail.
​According to the prosecution,the police officers conducted a buy-bust operation and
coordinated with the Philippine Drug Enforcement Agency (PDEA) regarding the operation.
​During the operation, "Pata" eluded arrest as he tried to run towards his shanty. Inside the
house, which was divided with a curtain as partition, the buy-bust team also saw Saraum and
Peter Esperanza, who were holding drug paraphernalia apparently in preparation to have a
"shabu" pot session. They recovered from Saraum’s possession a lighter, rolled tissue paper, and
aluminum tin foil (tooter). PO3 Larrobis confiscated the items, placed them in the plastic pack of
misua wrapper, and made initial markings ("A" for Saraum and "P" for Esperanza). At the police
station, PO3 Larrobis marked as "AIS-08- 17-2006" the paraphernalia recovered from Saraum.
After the case was filed, the subject items were turned over to the property custodian of the
Office of City Prosecutor.
​Saraum denied the commission of the alleged offense. He testified that on the date and
time in question, he was passing by Lorega Cemetery on his way to the house of his parents-in-
law when he was held by men with firearms. They were already with "Antik" and "Pata," both of
whom were his neighbors. Believing that he had not committed anything illegal, he resisted the
arrest. He learned of the criminal charge only when he was brought to the court. RTC ruled
against Sarum. CA sustained the RTCs decision.

ISSUE:
​Whether or not the Arrest of saraum was valid
HELD:
​Yes. Considering that Saraum failed to show any arbitrariness, palpable error, or
capriciousness on the findings of fact of the trial and appellate courts, such findings deserve great
weight and are deemed conclusive and binding. Besides, a review of the records reveals that the
CA did not err in affirming his conviction.
​The elements of illegal possession of equipment, instrument, apparatus and other
paraphernalia for dangerous drugs under Section 12, Article II of R.A. No. 9165 are: (1)
possession or control by the accused of any equipment, apparatus or other paraphernalia fit or
intended for smoking, consuming, administering, injecting, ingesting, or introducing any
dangerous drug into the body; and (2) such possession is not authorized by law. In this case, the
prosecution has convincingly established that Saraum was in possession of drug paraphernalia,
particularly aluminum tin foil, rolled tissue paper, and lighter, all of which were offered and
admitted in evidence.
​Saraum was arrested during the commission of a crime, which instance does not require a
warrant in accordance with Section 5 (a), Rule 113 of the Revised Rules on Criminal Procedure.
In arrest in flagrante delicto, the accused is apprehended at the very moment he is committing or
attempting to commit or has just committed an offense in the presence of the arresting officer. To
constitute a valid in flagrante delicto arrest, two requisites must concur: (1) the person to be
arrested must execute an overt act indicating that he has just committed, is actually committing,
or is attempting to commit a crime; and (2) such overt act is done in the presence or within the
view of the arresting officer.
​Here, the Court is unconvinced with Saraum’s statement that he was not committing a
crime at the time of his arrest. PO3 Larrobis described in detail how they were able to apprehend
him, who was then holding a disposable lighter in his right hand and a tin foil and a rolled tissue
paper in his left hand, while they were in the course of arresting somebody. The case is clearly
one of hot pursuit of "Pata," who, in eluding arrest, entered the shanty where Saraum and
Esperanza were incidentally caught in possession of the illegal items. Saraum did not proffer any
satisfactory explanation with regard to his presence at the vicinity of the buy-bust operation and
his possession of the seized items that he claims to have "countless, lawful uses." On the
contrary, the prosecution witnesses have adequately explained the respective uses of the items to
prove that they were indeed drug paraphernalia. There is, thus, no necessity to make a laboratory
examination and finding as to the presence or absence of methamphetamine hydrochloride or any
illegal substances on said items since possession itself is the punishable act.
​The valid warrantless arrest gave the officers the right to search the shanty for objects
relating to the crime and seize the drug paraphernalia they found. In the course of their lawful
intrusion, they inadvertently saw the various drug paraphernalia. As these items were plainly
visible, the police officers were justified in seizing them. Considering that Saraum’s arrest was
legal, the search and seizure that resulted from it were likewise lawful. The various drug
paraphernalia that the police officers found and seized in the shanty are, therefore, admissible in
evidence for having proceeded from a valid search and seizure. Since the confiscated drug
paraphernalia are the very corpus delicti of the crime charged, the Court has no choice but to
sustain the judgment of conviction.
​Even if We consider the arrest as invalid, Saraum is deemed to have waived any objection
thereto when he did not raise the issue before entering his plea. "The established rule is that an
accused may be estopped from assailing the legality of his arrest if he failed to move for the
quashing of the Information against him before his arraignment. Any objection involving the
arrest or the procedure in the court's acquisition of jurisdiction over the person of an accused
must be made before he enters his plea; otherwise the objection is deemed waived." In this case,
counsel for Saraum manifested its objection to the admission of the seized drug paraphernalia,
invoking illegal arrest and search, only during the formal offer of evidence by the prosecution.

COMERCIANTE VS PEOPLE

DOCTRINE:
​The balance lies in the concept of "suspiciousness" present where the police officer finds
himself or herself in. This may be undoubtedly based on the experience of the police officer.
Experienced police officers have personal experience dealing with criminals and criminal
behavior. Hence, they should have the ability to discern - based on facts that they themselves
observe - whether an individual is acting in a suspicious manner. Clearly, a basic criterion would
be that the police officer, with his or her personal knowledge, must observe the facts leading to
the suspicion of an illicit act.

FACTS:
​Comerciante was caught having in his possession, custody and control Two (2) heat-
sealed transparent plastic sachet (sic) each containing 0.15 gram (sic) and 0.28 gram (sic) of
white crystalline substance with a total of 0.43 grams which was found positive to the test for
Methamphetamine Hydrochloride commonly known as "shabu", a dangerous drug.
​According to the prosecution, the Police were patrolling the area while on their way to
visit a friend at Private Road, Barangay Hulo, Mandaluyong City. when they spotted two (2) men
- later identified as Comerciante and a certain Erick Dasilla7 (Dasilla) - standing and showing
"improper and unpleasant movements," with one of them handing plastic sachets to the other.
Thinking that the sachets may contain shabu, they immediately stopped and approached
Comerciante and Dasilla. The police officer arrested Comerciante and Dasilla, and confiscated
two (2) plastic sachets containing white crystalline substance from them. A laboratory
examination later confirmed that said sachets contained methamphetamine hydrochloride or
shabu.
​After the prosecution rested its case, Dasilla filed a demurrer to evidence, which was
granted by the RTC, thus his acquittal. However, due to Comerciante's failure to file his own
demurrer to evidence, the RTC considered his right to do so waived and ordered him to present
his evidence.
​In his defense, Comerciante averred that PO3 Calag was looking for a certain "Barok",
who was a notorious drug pusher in the area, when suddenly, he and Dasilla, who were just
standing in front of a jeepney along Private Road, were arrested and taken to a police station.
There, the police officers claimed to have confiscated illegal drugs from them and were asked
money in exchange for their release. When they failed to accede to the demand, they were
brought to another police station to undergo inquest proceedings, and thereafter, were charged
with illegal possession of dangerous drugs.
​The RTC found that PO3 Calag conducted a valid warrantless arrest on Comerciante,
which yielded two (2) plastic sachets containing shabu. In this relation, the RTC opined that
there was probable cause to justify the warrantless arrest, considering that PO3 Calag saw, in
plain view, that Comerciante was carrying the said sachets when he decided to approach and
apprehend the latter. Further, the RTC found that absent any proof of intent that PO3 Calag was
impelled by any malicious motive, he must be presumed to have properly performed his duty
when he arrested Comerciante. CA Affirmed the RTCs ruling.

ISSUE:
​Whether or not the there was a valid warrantless arrest and “stop and frisk” conducted by
the Police

HELD:
​No, the Court finds it highly implausible that PO3 Calag, even assuming that he has
perfect vision, would be able to identify with reasonable accuracy especially from a distance of
around 10 meters, and while aboard a motorcycle cruising at a speed of 30 kilometers per hour
miniscule amounts of white crystalline substance inside two (2) very small plastic sachets held
by Comerciante. The Court also notes that no other overt act could be properly attributed to
Comerciante as to rouse suspicion in the mind of PO3 Calag that the former had just committed,
was committing, or was about to commit a crime. Verily, the acts of standing around with a
companion and handing over something to the latter cannot in any way be considered criminal
acts. In fact, even if Comerciante and his companion were showing "improper and unpleasant
movements" as put by PO3 Calag, the same would not have been sufficient in order to effect a
lawful warrantless arrest under Section 5 (a), Rule 113 of the Revised Rules on Criminal
Procedure.31 That his reasonable suspicion bolstered by (a) the fact that he had seen his fellow
officers arrest persons in possession of shabu; and (b) his trainings and seminars on illegal drugs
when he was still assigned in the province are insufficient to create a conclusion that what he
purportedly saw in Comerciante was indeed shabu.
​Neither has the prosecution established that the rigorous conditions set forth in Section 5
(b), Rule 113, have been complied with, i.e., that an offense had in fact just been committed and
the arresting officer had personal knowledge of facts indicating that the accused had committed
it. As already discussed, the factual backdrop of the instant case failed to show that PO3 Calag
had personal knowledge that a crime had been indisputably committed by Comerciante. Verily, it
is not enough that the arresting officer had reasonable ground to believe that the accused had just
committed a crime; a crime must, in fact, have been committed first, which does not obtain in
this case.
​In this relation, the Court finds respondent's assertion that there was a valid "stop and
frisk" search made on Comerciante untenable. In People v. Cogaed, the Court had an opportunity
to exhaustively explain "stop and frisk" searches:
"Stop and frisk" searches (sometimes referred to as Terry searches) are necessary for law
enforcement. That is, law enforcers should be given the legal arsenal to prevent the commission
of offenses. However, this should be balanced with the need to protect the privacy of citizens in
accordance with Article III, Section 2 of the Constitution.
​The balance lies in the concept of "suspiciousness" present where the police officer finds
himself or herself in. This may be undoubtedly based on the experience of the police officer.
Experienced police officers have personal experience dealing with criminals and criminal
behavior. Hence, they should have the ability to discern - based on facts that they themselves
observe - whether an individual is acting in a suspicious manner. Clearly, a basic criterion would
be that the police officer, with his or her personal knowledge, must observe the facts leading to
the suspicion of an illicit act.
​x x x x Normally, "stop and frisk" searches do not give the law enforcer an opportunity to
confer with a judge to determine probable cause. In Posadas v. Court of Appeals, one of the
earliest cases adopting the "stop and frisk" doctrine in Philippine jurisprudence, this court
approximated the suspicious circumstances as probable cause:
​The probable cause is that when the petitioner acted suspiciously and attempted to flee
with the buri bag there was a probable cause that he was concealing something illegal in the bag
and it was the right and duty of the police officers to inspect the same.
​For warrantless searches, probable cause was defined as a reasonable ground of suspicion
supported by circumstances sufficiently strong in themselves to warrant a cautious man to
believe that the person accused is guilty of the offense with which he is charged.
Malacat v. Court of Appeals clarifies the requirement further. It does not have to be probable
cause, but it cannot be mere suspicion. It has to be a genuine reason to serve the purposes of the
"stop and frisk" exception:
​Other notable points of Terry are that while probable cause is not required to conduct a
"stop and frisk," it nevertheless holds that mere suspicion or a hunch will not validate a "stop and
frisk." A genuine reason must exist, in light of the police officer's experience and surrounding
conditions, to warrant the belief that the person detained has weapons concealed about him.
​In his dissent for Esquillo v. People, Justice Bersamin reminds us that police officers
must not rely on a single suspicious circumstance. There should be "presence of more than one
seemingly innocent activity, which, taken together, warranted a reasonable inference of criminal
activity." The Constitution prohibits "unreasonable searches and seizures." Certainly, reliance on
only one suspicious circumstance or none at all will not result in a reasonable search.

LUZ VS PEOPLE

DOCTRINE:
​At the time a person is arrested, it shall be the duty of the arresting officer to inform the
latter of the reason for the arrest and must show that person the warrant of arrest, if any. Persons
shall be informed of their constitutional rights to remain silent and to counsel, and that any
statement they might make could be used against them.

FACTS:
​PO2 Emmanuel L. Alteza testified that he saw the accused driving a motorcycle without
a helmet and this prompted him to flag down the accused for violating a municipal ordinance
which requires all motorcycle drivers to wear helmet while driving said motor vehicle. He
invited the accused to come inside their sub-station since the place where he flagged down the
accused is almost in front of the sub-station to where he is assigned as a traffic enforcer. While
he and SPO1 Rayford Brillante were issuing a citation ticket for violation of municipal
ordinance, he noticed that the accused was uneasy and kept on getting something from his jacket.
He was alerted and so, he told the accused to take out the contents of the pocket of his jacket as
the latter may have a weapon inside it. The accused obliged and slowly put out the contents of
the pocket of his jacket which included two (2) plastic sachets of suspected shabu. The RTC
convicted petitioner of illegal possession of dangerous drugs. It found the prosecution evidence
sufficient to show that he had been lawfully arrested for a traffic violation and then subjected to a
valid search, which led to the discovery on his person of two plastic sachets later found to
contain shabu. Upon review, the CA affirmed the RTCs Decision.

ISSUE:
​Whether or not the search and seizure of the alleged subject shabu was incident to a
lawful arrest.

HELD:
​Court of Appeals decision is reversed. There was no valid arrest of petitioner. When he
was flagged down for committing a traffic violation, he was not, ipso facto and solely for this
reason, arrested.
​Arrest is the taking of a person into custody in order that he or she may be bound to
answer for the commission of an offense. It is effected by an actual restraint of the person to be
arrested or by that persons voluntary submission to the custody of the one making the arrest.
Neither the application of actual force, manual touching of the body, or physical restraint, nor a
formal declaration of arrest, is required. It is enough that there be an intention on the part of one
of the parties to arrest the other, and that there be an intent on the part of the other to submit,
under the belief and impression that submission is necessary. Under R.A. 4136, or the Land
Transportation and Traffic Code, the general procedure for dealing with a traffic violation is not
the arrest of the offender, but the confiscation of the drivers license of the latter.
​At the time that he was waiting for PO3 Alteza to write his citation ticket, petitioner
could not be said to have been under arrest. There was no intention on the part of PO3 Alteza to
arrest him, deprive him of his liberty, or take him into custody. Prior to the issuance of the ticket,
the period during which petitioner was at the police station may be characterized merely as
waiting time. In fact, as found by the trial court, PO3 Alteza himself testified that the only reason
they went to the police sub- station was that petitioner had been flagged down almost in front of
that place. Hence, it was only for the sake of convenience that they were waiting there. There
was no intention to take petitioner into custody.Even if one were to work under the assumption
that petitioner was deemed arrested upon being flagged down for a traffic violation and while
awaiting the issuance of his ticket, then the requirements for a valid arrest were not complied
with. At the time a person is arrested, it shall be the duty of the arresting officer to inform the
latter of the reason for the arrest and must show that person the warrant of arrest, if any. Persons
shall be informed of their constitutional rights to remain silent and to counsel, and that any
statement they might make could be used against them. It may also be noted that in this case,
these constitutional requirements were complied with by the police officers only after petitioner
had been arrested for illegal possession of dangerous drugs. GRANTED.

ANTIQUERA VS PEOPLE

DOCTRINE:
​The failure of the accused to object to the irregularity of his arrest by itself is not enough
to sustain his conviction. A waiver of an illegal warrantless arrest does not carry with it a waiver
of the inadmissibility of evidence seized during the illegal warrantless arrest.

FACTS:
​At around 4:45 A.M. of February 11, 2004, police officers Gregorio and Laurence while
onboard a patrol car, saw two unidentified men rush out of a house in David St., Pasay City.
Sensing something amiss, the police officers approached the house and peeked inside the
partially opened door, where they saw George holding an improvised tooter and a pink lighter,
and beside him, his live-in partner, Corazon. Because of this, they entered the house, and arrested
George and Corazon. A search of the immediate surroundings revealed a wooden box containing
improvised tooter, scoop 10 sachets of suspected shabu, and strips of aluminum oil. Because of
they, they were charged with illegal possession of drugs paraphernalia. Only George appealed the
decision rendered by the RTC convicting him as charged, since Corazon jumped bail. The Court
of Appeals denied his appeal, hence he elevated his case to the Supreme Court. Both lower
courts justified the conviction of George, citing his arrest was a valid warrantless arrest under
Section 5, Rule 113 of the Rules of Court.

HELD:
​The Supreme Court: The prosecution’s theory, upheld by both the RTC and the CA, is
that it was a case of valid warrantless arrest in that the police officers saw accused Antiquera and
Cruz through the door of their house, in the act of having a pot session. That valid warrantless
arrest gave the officers the right as well to search the living room for objects relating to the crime
and thus seize the paraphernalia they found there.
​The prosecution contends that, since the seized paraphernalia tested positive for shabu,
they were no doubt used for smoking, consuming, administering, injecting, ingesting, or
introducing dangerous drug into the body in violation of Section 12 of Republic Act 9165. That
the accused tested negative for shabu, said the prosecution, had no bearing on the crime charged
which was for illegal possession of drug paraphernalia, not for illegal use of dangerous drugs.
The prosecution added that even assuming that the arrest of the accused was irregular, he is
already considered to have waived his right to question the validity of his arrest when he
voluntarily submitted himself to the court’s jurisdiction by entering a plea of not guilty.
​Section 5(a), Rule 113 of the Rules of Criminal Procedure provides that a “peace officer
or a private person may, without a warrant, arrest a person when, in his presence, the person to be
arrested has committed, is actually committing, or is attempting to commit an offense.” This is
an arrest in flagrante delicto. The overt act constituting the crime is done in the presence or
within the view of the arresting officer. But the circumstances here do not make out a case of
arrest made in flagrante delicto.
1. The police officers claim that they were alerted when they saw two unidentified men suddenly
rush out of 107 David Street, Pasay City. Since they suspected that a crime had been committed,
the natural thing for them to do was to give chase to the jeep that the two fleeing men boarded,
given that the officers were in a patrol car and a tricycle. Running after the fleeing suspects was
the more urgent task but the officers instead gave priority to the house even when they heard no
cry for help from it.
2. Admittedly, the police officers did not notice anything amiss going on in the house from the
street where they stood. Indeed, even as they peeked through its partially opened door, they saw
no activity that warranted their entering it.
​Clearly, no crime was plainly exposed to the view of the arresting officers that authorized
the arrest of accused Antiquera without warrant under the above-mentioned rule. Considering
that his arrest was illegal, the search and seizure that resulted from it was likewise illegal.
Consequently, the various drug paraphernalia that the police officers allegedly found in the house
and seized are inadmissible, having proceeded from an invalid search and seizure. Since the
confiscated drug paraphernalia is the very corpus delicti of the crime charged, the Court has no
choice but to acquit the accused.
​One final note. The failure of the accused to object to the irregularity of his arrest by
itself is not enough to sustain his conviction. A waiver of an illegal warrantless arrest does not
carry with it a waiver of the inadmissibility of evidence seized during the illegal warrantless
arrest.” Accused acquitted.

PEOPLE VS VASQUEZ

DOCTRINE:
​Any objection, defect or irregularity attending an arrest must be made before the accused
enters his plea on arraignment.

FACTS:
​This is an appeal from the Decision of CA which affirmed the joint decision of RTC in a
consolidated case, convicting the appellant Donald Vasquez y Sandigan (Don) of the crimes of
illegal sale and illegal possession of regulated drugs. Initially the case of illegal possession of
drugs was raffled but upon motion it was consolidated with the case of illegal sale of drugs. On
arraignment, the appellant pleaded not guilty to both charges. The pre-trial conference of the
cases was held, but the same was terminated without the parties entering into any stipulation of
facts. During the trial of the case the prosecution stated the events. There was a confidential
informant reported to PO2 Trambulo about the illegal drug activities. Fajardo form a buy-bust
team. It was in the buy-bust operation that Don was arrested. RTC convicted the appellant of the
crimes charged. The RTC gave more credence to the prosecution’s evidence given that the
presumption of regularity in the performance of official duty on the part of the police officers
was not overcome. On appeal the Court of Appeals affirmed the conviction of the appellant.
Hence this appeal. He argues that the police officers did not have a search warrant or a warrant of
arrest at the time he was arrested. This occurred despite the fact that the police officers allegedly
had ample time to secure a warrant of arrest against him. Inasmuch as his arrest was illegal, the
appellant avers that the evidence obtained as a result thereof was inadmissible in court.

ISSUE:
​Whether the appellant Don may assail the validity of arrest.
HELD:
​NO. At the outset, the Court rules that the appellant can no longer assail the validity of
his arrest. We reiterated in People v. Tampis that "any objection, defect or irregularity attending
an arrest must be made before the accused enters his plea on arraignment. Having failed to move
for the quashing of the information against them before their arraignment, appellants are now
estopped from questioning the legality of their arrest. Any irregularity was cured upon their
voluntary submission to the trial court’s jurisdiction."53 Be that as it may, the fact of the matter
is that the appellant was caught in flagrante delicto of selling illegal drugs to an undercover
police officer in a buy-bust operation. His arrest, thus, falls within the ambit of Section 5(a), Rule
11354 of the Revised Rules on Criminal Procedure when an arrest made without warrant is
deemed lawful. Having established the validity of the warrantless arrest in this case, the Court
holds that the warrantless seizure of the illegal drugs from the appellant is likewise valid. We
held in People v. Cabugatan that:
​This interdiction against warrantless searches and seizures, however, is not absolute
and such warrantless searches and seizures have long been deemed permissible by
jurisprudence in instances of (1) search of moving vehicles, (2) seizure in plain view, (3) customs
searches, (4) waiver or consented searches, (5) stop and frisk situations (Terry search), and
search incidental to a lawful arrest. The last includes a valid warrantless arrest, for, while as a
rule, an arrest is considered legitimate if effected with a valid warrant of arrest, the Rules of
Court recognize permissible warrantless arrest, to wit: (1) arrest in flagrante delicto, (2) arrest
effected in hot pursuit, and (3) arrest of escaped prisoners. (Citation omitted.)Thus, the
appellant cannot seek exculpation by invoking belatedly the invalidity of his arrest and the
subsequent search upon his person.

RULE 114 – BAIL

FLORESTA VS UBIADAS

DOCTRINE:
​Judges owe it the public and the legal profession to know the very law they are supposed
to apply to a given controversy.

FACTS:
​Then Provincial Prosecutor, now Regional Trial Court Judge Dorentino Z. Floresta
administratively charged Judge Eliodoro G. Ubiadas of the Regional Trial Court (RTC) with
gross ignorance of the law, grave abuse of authority and violations of the Code of Judicial
Conduct in hearing and deciding several cases.
​Judge Floresta faults Judge Ubiadas for dismissing a criminal case for illegal entry, for
lack of jurisdiction. Complainant likewise faults Judge Ubiadas for failure to resolve, as he has
yet to resolve, the Motion for Reconsideration and/or Clarificationof the Order dismissing said
criminal case, despite the lapse of more than two years since the filing of the motion. By such
failure, he charges Judge Ubiadas with violation of Canon 3, Rule 3.05 of the Code of Judicial
Conduct which enjoins judges to dispose of the court’s business promptly and decide cases
within the required periods, and of SC Circular No. 13 (July 1, 1987) which requires lower
courts to resolve cases or matters before them within three months or ninety days from date of
submission.
​Judge Floresta furthermore faults Judge Ubiadas for granting, “without giving notice to
the prosecution,” the petition for bail of Jose Mangohig, Jr. who was arrested by virtue of a
warrant issued by the Municipal Trial Court of Subic, Zambales which found probable cause
against him for violation of Section 5(b), Art. III of Republic Act No. 7610 (“Special Protection
of Children Against Child Abuse, Exploitation and Discrimination Act”). Finally, he faults Judge
Ubiadas for disqualifying petitioner judge from appearing in a criminal case despite petitioner
judge’s designation to handle the prosecution of the case by the Ombudsman.

ISSUE:
​Whether or not Judge Ubiadas acted with gross ignorance of the law, grave abuse of
authority and violations of the Code of Judicial Conduct in hearing and deciding cases

HELD:
​Judge Eliodoro G. Ubiadas is found GUILTY of undue delay in resolving a motion and
of gross ignorance of the law or procedure in granting an application for bail without affording
the prosecution due process.
​On innumerable occasions this Court has impressed upon judges that, as mandated by the
Code of Judicial Conduct, they owe it to the public and the legal profession to know the very law
they are supposed to apply to a given controversy. They are called upon to exhibit more than just
a cursory acquaintance with statutes and procedural rules, to be conversant with the basic law,
and to maintain the desired professional competence.
​The propriety of the dismissal, on motion of the accused, on jurisdictional grounds is,
however, a matter for judicial adjudication and the proper recourse of a party aggrieved by the
decision of a judge is to appeal to the proper court, not file an administrative complaint.
However, having failed to resolve the Motion for Reconsideration, Judge Ubiadas is liable for
undue delay in rendering a decision or order which is a less serious charge under Section 9 of
Rule 140 of the Rules of Court.
​The Court takes the occasion to reiterate the injunction that a judge is called upon to
balance the interests of the accused who is entitled to the presumption of innocence until his guilt
is proven beyond reasonable doubt, and to enable him to prepare his defense without being
subject to punishment prior to conviction, against the right of the State to protect the people and
the peace of the community from dangerous elements.
In the exercise of his power to investigate and prosecute on its own or on complaint by any
person, any act or omission of any public officer or employee, office or agency, when such act or
omission appears to be illegal, unjust, improper or inefficient, the Ombudsman is authorized to
call on prosecutors or lawyers in the government service for assistance.
Judge Ubiadas was not onlyawareof complainant’s designation, hence, belying his explanation
that he must have overlooked the same. It also shows his ignorance of the provision of the
Ombudsman Act which does not require the presence of a special reason for the designation or
deputization by the Ombudsman of any prosecutor or government lawyer to assist him.

ZUNO VS CABEBE

DOCTRINE:
​The importance of a bail hearing and a summary of evidence cannot be downplayed,
these are considered aspects of procedural due process for both the prosecution and the defense;
its absence will invalidate the grant or denial of bail.

FACTS:
​The instant administrative case stemmed from the sworn complaintof Chief State
Prosecutor Jovencito R. Zuño of the Department of Justice, against Judge Alejandrino C.
Cabebe,then Presiding Judge, Regional Trial Court. The charges are knowingly rendering an
unjust judgment, gross ignorance of the law and partiality.
​In his complaint, Chief State Prosecutor Zuño alleged that Criminal Case for illegal
possession of prohibited or regulated drugs was filed with the Regional Trial Court, against Rey
DaquepArcangel, VictorinoGametMalabed, William Roxas Villanueva, all police officers,
Jocelyn Malabed Manuel and Pelagio Valencia Manuel. Upon arraignment, all the accused,
assisted by their counsel de parte, pleaded not guilty to the crime charged.
​On May 6, 2002, the accused filed a motion to dismiss invoking as ground the right of the
accused to a speedy trial. On November 5, 2002, respondent judge motupropioissued an Order
granting bail to the accused, fixing the bail for each at P70,000.00 in cash or property bond at
P120,000.00, except for accused Evelyn Manuel whose bail was fixed at P20,000.00 in cash.
Respondent judge issued the Order without the accused’s application or motion for bail.
The prosecution then filed a motion for reconsideration. Instead of acting thereon, respondent
judge issued an order inhibiting himself from further proceeding with the case, realizing that
what he did was patently irregular. Complainant thus prays that respondent judge be dismissed
from the service with forfeiture of all benefits and be disbarred from the practice of law.
​In his comment, respondent denied the charges. While admitting that he issued the Order
granting bail to the accused without any hearing, “the same was premised on the constitutional
right of the accused to a speedy trial.” The prosecution did not object to the grant of bail to the
accused.
​On March 26, 2003, respondent judge compulsorily retired.
​In his Report dated July 7, 2003, Deputy Court Administrator Jose P. Perez found
respondent judge liable for gross ignorance of the law and recommended that a fine
ofP20,000.00 be imposed upon him, with a stern warning that a repetition of the same or similar
offense will be dealt with more severely.

ISSUE:
​Whether or not the judge is guilty of gross ignorance of the law by granting bail without
hearing.

HELD:
​YES. In Docena-Caspe vs. Judge Arnulfo O. Bugtas,we held that jurisprudence is replete
with decisions on the procedural necessity of a hearing, whether summary or otherwise, relative
to the grant of bail, especially in cases involving offenses punishable by death, reclusion
perpetua, or life imprisonment, where bail is a matter of discretion. Under the present Rules, a
hearing is mandatory in granting bail whether it is a matter of right or discretion.It must be
stressed that the grant or the denial of bail in cases where bail is a matter of discretion, hinges on
the issue of whether or not the evidence of guilt of the accused is strong, and the determination
of whether or not the evidence is strong is a matter of judicial discretion which remains with the
judge. In order for the latter to properly exercise his discretion, he must first conduct a hearing to
determine whether the evidence of guilt is strong. In fact, even in cases where there is no petition
for bail, a hearing should still be held.
​There is no question that respondent judge granted bail to the accused without conducting
a hearing, in violation of Sections 8 and 18, Rule 114 of the Revised Rules of Criminal
Procedure.
​In Cortes vs. Catral, we laid down the following rules outlining the duties of the judge in
case an application for bail is filed:
​1. In all cases whether bail is a matter of right or discretion, notify the prosecutor of the
​ ​ ​ earing of the application for bail or require him to submit his
h
recommendation (Section ​ ​ ​ 8, Rule 114 of the Revised Rules of Criminal
1
Procedure);
​2. Where bail is a matter of discretion, conduct a hearing of the application for bail
​ ​ r​ egardless of whether or not the prosecution refuses to present evidence to
show that the ​ ​ ​guilt of the accused is strong for the purpose of enabling the
court to exercise its sound ​ ​ ​discretion (Section 7 and 8, id.);
​3. Decide whether the guilt of the accused is strong based on the summary of evidence of
​ ​ t​ he prosecution;
​4. If the guilt of the accused is not strong, discharge the accused upon the approval of the
​ ​ ​ ail bond (Section 19, id.); otherwise the petition should be denied.
b
​Based on the above-cited procedure, after the hearing, the court’s order granting or
refusing bail must contain a summary of the evidence of the prosecution and based thereon, the
judge should formulate his own conclusion as to whether the evidence so presented is strong
enough to indicate the guilt of the accused.
​Respondent judge did not follow the above Rules and procedure enumerated in Cortes.
He did not conduct a hearing before he granted bail to the accused, thus depriving the
prosecution of an opportunity to interpose objections to the grant of bail. Irrespective of his
opinion on the strength or weakness of evidence to prove the guilt of the accused, he should have
conducted a hearing and thereafter made a summary of the evidence of the prosecution. The
importance of a bail hearing and a summary of evidence cannot be downplayed, these are
considered aspects of procedural due process for both the prosecution and the defense; its
absence will invalidate the grant or denial of bail.
​Respondent’s contention is bereft of merit. There is no indication in the records of the
criminal case that the prosecution has intentionally delayed the trial of the case. Even assuming
there was delay, this does not justify the grant of bail without a hearing. This is utter disregard of
the Rules. The requirement of a bail hearing has been incessantly stressed by this Court. In the
same vein, the Code of Judicial Conduct enjoins judges to be conversant with the law and the
Rules and maintain professional competence; and by the very nature of his office, should be
circumspect in the performance of his duties. He must render justice without resorting to
shortcuts clearly uncalled for. Obviously, respondent failed to live up to these standards.
WHEREFORE, respondent Judge Alejandrino C. Cabebe, now retired, is found guilty of
violation of Supreme Court Rules and is hereby fined in the sum of Twenty Thousand Pesos
(P20,000.00), the same to be deducted from his retirement benefits.

GOVERNMENT OF HONGKONG SPEC. ADM. REGION VS OLALIA

DOCTRINE:
​The modern trend in public international law is the primacy placed on the worth of the
individual person and the sanctity of human rights.
​If bail can be granted in deportation cases, the Court sees no justification why it should
not also be allowed in extradition cases—clearly, the right of a prospective extraditee to apply for
bail must be viewed in the light of the various treaty obligations of the Philippines concerning
respect for the promotion and protection of human rights.

FACTS:
​Private respondent Muñoz was charged before Hong Kong Court. Warrants of arrest were
issued and by virtue of a final decree the validity of the Order of Arrest was upheld. The
petitioner Hong Kong Administrative Region filed a petition for the extradition of the private
respondent. In the same case, a petition for bail was filed by the private respondent.
​The petition for bail was denied by reason that there was no Philippine law granting the
same in extradition cases and that the respondent was a high “flight risk”. Private respondent
filed a motion for reconsideration and was granted by the respondent judge subject to the
following conditions:
22. Bail is set at Php750,000.00 in cash with the condition that accused hereby undertakes that he
will appear and answer the issues raised in these proceedings and will at all times hold
himself amenable to orders and processes of this Court, will further appear for judgment. If
accused fails in this undertaking, the cash bond will be forfeited in favor of the government;
23. Accused must surrender his valid passport to this Court;
24. The Department of Justice is given immediate notice and discretion of filing its own motion
for hold departure order before this Court even in extradition proceeding; and
25. Accused is required to report to the government prosecutors handling this case or if they so
desire to the nearest office, at any time and day of the week; and if they further desire,
manifest before this Court to require that all the assets of accused, real and personal, be filed
with this Court soonest, with the condition that if the accused flees from his undertaking, said
assets be forfeited in favor of the government and that the corresponding lien/annotation be
noted therein accordingly.
​Petitioner filed a motion to vacate the said order but was denied by the respondent judge.
Hence, this instant petition.

ISSUE:
​Whether or not a potential extraditee is entitled to post bail
HELD:
​A potential extraditee is entitled to bail.
Ratio Decidendi
​Petitioner alleged that the trial court committed grave abuse of discretion amounting to
lack or excess of jurisdiction in admitting private respondent to bail; that there is nothing in the
Constitution or statutory law providing that a potential extraditee has a right to bail, the right
being limited solely to criminal proceedings.
​On the other hand, private respondent maintained that the right to bail guaranteed under
the Bill of Rights extends to a prospective extraditee; and that extradition is a harsh process
resulting in a prolonged deprivation of one’s liberty.
​In this case, the Court reviewed what was held in Government of United States of
America v. Hon. Guillermo G. Purganan, Presiding Judge, RTC of Manila, Branch 42, and Mark
B. Jimenez, a.k.a. Mario Batacan Crespo GR No. 153675 April 2007, that the constitutional
provision on bail does not apply to extradition proceedings, the same being available only in
criminal proceedings. The Court took cognizance of the following trends in international law:
26. the growing importance of the individual person in public international;
27. the higher value now being given to human rights;
28. the corresponding duty of countries to observe these universal human rights in fulfilling their
treaty obligations; and
29. the duty of this Court to balance the rights of the individual under our fundamental law, on
one hand, and
the law on extradition, on the other.
​In light of the recent developments in international law, where emphasis is given to the
worth of the individual and the sanctity of human rights, the Court departed from the ruling in
Purganan, and held that an extraditee may be allowed to post bail.

PEOPLE VS SANDIGANBAYAN

DOCTRINE:
​Even if the capital offense charged is bailable owing to the weakness of the evidence of
guilt, the right to bail may justifiably still be denied if the probability of escape is great. A grant
of bail does not prevent the trier of facts, the same Anti-Graft Court, from making a final
assessment of the evidence after full trial on the merits.

FACTS:
​On November 2000, as an offshoot of the impeachment proceedings against Joseph
Ejercito Estrada, then President of the Republic of the Philippines, five criminal complaints
against the former President and members of his family, his associates, friends and conspirators
were filed with the Office of the Ombudsman.
​On April 4, 2001, the Ombudsman issued a Joint Resolution finding probable cause
warranting the filing with the Sandiganbayan of several criminal Information against the former
President and the other respondents therein. One of the
​Informations was for the crime of plunder under Republic Act [RA] No. 7080 and among
the respondents was herein petitioner Jose Jinggoy Estrada, then mayor of San Juan, Metro
Manila.
​The Information was amended and filed on April 18, 2001. Docketed as Criminal Case
No. 26558, the case was assigned to [the] respondent Third Division of the Sandiganbayan.
The amended information referred to, like the original, charged respondent Jinggoy, together
with the former President and several others, with plunder, defined and penalized under RA No.
7080, as amended by Section 12 of RA No. 7659.
​From the denial action of the Sandiganbayan immediately adverted to, Jinggoy interposed
a petition for certiorari before this Court claiming that the respondent Sandiganbayan committed
grave abuse of discretion in, inter alia, (a) sustaining the charge against him for alleged offenses
and with alleged conspirators with whom he is not even connected, and (b) in not fixing bail for
him. Pending resolution of this petition, docketed as G.R. No. 148965, Jinggoy filed with the
Sandiganbayan an „Urgent Second Motion for Bail for Medical Reasons. The Ombudsman
opposed the motion. For three (3) days in September 2001, the Sandiganbayan conducted
hearings on the motion for bail, with one Dr. Roberto Anastacio of the Makati Medical Center
appearing as sole witness for Jinggoy. On December 18, 2001, Jinggoy filed with the Court an
Urgent Motion praying for early resolution of his Petition for Bail on Medical/Humanitarian
Considerations. He reiterated his earlier plea for bail filed with the Sandiganbayan. On the same
day, the Court referred the motion to the Sandiganbayan for resolution and directed said court to
make a report, not later than 8:30 in the morning of December 21, 2001.
​The report was submitted as directed. Attached to the Report was a copy of the
SandiganbayanÊs Resolution dated December 20, 2001 denying Jinggoy’s motion for bail for
lack of factual basis. According to the graft court, basing its findings on the earlier testimony of
Dr. Anastacio, Jinggoy failed to submit sufficient evidence to convince the court that the medical
condition of the accused requires that he be confined at home and for that purpose that he be
allowed to post bail.
​On February 26, 2002, the Court dismissed Jinggoy’ petition.
​On April 17, 2002, Jinggoy filed before the Sandiganbayan an Omnibus Application for
Bail against which the prosecution filed its comment and opposition. Bail hearings were then
conducted, followed by the submission by the parties of their respective memoranda.
​In the herein assailed Resolution of March 6, 2003, respondent Sandiganbayan (Special
Division) granted the omnibus application for bail. Petitioner filed a motion for reconsideration
thereto which the respondent court denied via the herein equally assailed May 30, 2003
Resolution. Hence, the present petition on the submission that respondent Special Division of the
Sandiganbayan acted with grave abuse of discretion amounting to lack or excess of jurisdiction

ISSUE:
​Whether the grant of bail was proper.
HELD:
​Even if the capital offense charged is bailable owing to the weakness of the evidence of
guilt, the right to bail may justifiably still be denied if the probability of escape is great. Here,
ever since the promulgation of the assailed Resolutions a little more than four (4) years ago,
Jinggoy does not, as determined by Sandiganbayan, seem to be a flight risk.
​As we see it, the rulings in Castelo and Ty Sui Wong are not on all-fours applicable to and
of governing sway to the issue of the propriety of revoking Jinggoy’s release on bail. As it were,
the petitioner erroneously equates the provisional grant of bail to respondent Jinggoy to his
virtual acquittal in Criminal Case No. 26558. Petitioner is wrong. Castelo and Ty Sui Wong
contextually dealt with the guilt of culprits therein for the crimes of murder after all the evidence
had been adduced. Unlike in this proceeding, the propriety of a grant of bail, given the evidence
for or against the bail application, was not an issue in Castelo and Ty Sui Wong. And in the
present case, respondent Sandiganbayan is still in the process of determining the facts and merits
of the main case.
​With the view we take of this case, the respondent court did not commit grave abuse of
discretion in issuing its assailed resolutions, because the grant of bail therein is predicated only
on its preliminary appreciation of the evidence.

OKABE VS GUTIERREZ

DOCTRINE:
​Section 26, Rule 114 of the Revised Rules on Criminal Procedure is a new one, intended
to modify previous rulings of the Court that an application for bail or the admission to bail by the
accused shall be considered as a waiver of his right to assail the warrant issued for his arrest on
the legalities or irregularities thereon; Curative statutes are by their essence retroactive in
application.

FACTS:
​Charged for Estafa, Petitioner filed a verified motion for judicial determination of
probable cause and to defer proceedings/arraignment, alleging that the only documents appended
to the Information submitted by the investigating prosecutor were respondent Maruyamas
affidavit-complaint for estafa and the resolution of the investigating prosecutor; the affidavits of
the witnesses of the complainant, the respondents counter-affidavit and the other evidence
adduced by the parties were not attached thereto. The petitioner further alleged that the
documents submitted by the investigating prosecutor were not enough on which the trial court
could base a finding of probable cause for estafa against her.
The court denied the petitioners motions on the following grounds:
(a) Based on its personal examination and consideration of the Information, the affidavit-
complaint of respondent Maruyama and the resolution of the investigating prosecutor duly
approved by the city prosecutor, the court found probable cause for the petitioners arrest. Since
the petitioners motion for a determination of probable cause was made after the court had already
found probable cause and issued a warrant for the petitioners arrest, and after the latter filed a
personal bail bond for her provisional liberty, such motion was a mere surplusage;
(b) In denying her motion for a determination of probable cause, she posits that the respondent
judge acted with grave abuse of discretion amounting to excess or lack of jurisdiction.

ISSUE:
​If the RTC judge may rely on investigating prosecutor’s resolution in the determination
of probable cause for the arrest of the accused.

HELD:
​NO. In determining the existence or non-existence of probable cause for the arrest of the
accused, the judge should not rely solely on the said report.The judge should consider not only
the report of the investigating prosecutor but also the affidavit/affidavits and the documentary
evidence of the parties, the counter-affidavit of the accused and his witnesses, as well as the
transcript of stenographic notes taken during the preliminary investigation, if any, submitted to
the court by the investigating prosecutor upon the filing of the Information. The duty to make
such determination is personal and exclusive to the issuing judge. He cannot abdicate his duty
and rely on the certification of the investigating prosecutor that he had conducted a preliminary
investigation in accordance with law and the Rules of Court, as amended, and found probable
cause for the filing of the Information.
​The task of the presiding judge when the Information is filed with the court is first and
foremost to determine the existence or non-existence of probable cause for the arrest of the
accused. Probable cause is meant such set of facts and circumstances which would lead a
reasonably discreet and prudent man to believe that the offense charged in the Information or any
offense included therein has been committed by the person sought to be arrested. The purpose of
the mandate of the judge to first determine probable cause for the arrest of the accused is to
insulate from the very start those falsely charged of crimes from the tribulations, expenses and
anxiety of a public trial.
​Under Section 6, Rule 112 of the Rules of Court in relation to Section 2, Article III of the
1987 Constitution, the judge must make a personal determination of the existence or non-
existence of probable cause for the arrest of the accused. Under Section 1, Rule 112 of the Rules
on Criminal Procedure, the investigating prosecutor, in conducting a preliminary investigation of
a case cognizable by the RTC, is tasked to determine whether there is sufficient ground to
engender a well- founded belief that a crime has been committed and the respondent therein is
probably guilty thereof and should be held for trial. A preliminary investigation is for the purpose
of securing the innocent against hasty, malicious and oppressive prosecution, and to protect him
from an open and public accusation of a crime, from the trouble, expense and anxiety of a public
trial.
​If the investigating prosecutor finds probable cause for the filing of the Information
against the respondent, he executes a certification at the bottom of the Information that from the
evidence presented, there is a reasonable ground to believe that the offense charged has been
committed and that the accused is probably guilty thereof. Such certification of the investigating
prosecutor is, by itself, ineffective. It is not binding on the trial court. Nor may the RTC rely on
the said certification as basis for a finding of the existence of probable cause for the arrest of the
accused.

LEVISTE VS CA

DOCTRINE:
​Bail acts as a reconciling mechanism to accommodate both the accused’s interest in
pretrial liberty and society’s interest in assuring the accused’s presence at trial.
​An erroneously convicted accused who is denied bail loses his liberty to pay a debt to
society he has never owed; Under what circumstances an accused may obtain bail pending
appeal is a delicate balance between the interests of society and those of the accused; In the
exercise of discretion in the grant of bail pending appeal, the proper courts are to be guided by
the fundamental principle that the allowance of bail pending appeal should be exercised not with
laxity but with grave caution and only for strong reasons, considering that the accused has been
in fact convicted by the trial court.

FACTS:
​Jose Antonio Leviste was charged with the crime of murder but was convicted by the
RTC for the lesser crime of homicide. He appealed the RTC's decision to the CA then he field an
application for admission to bail pending appeal, due to his advanced age and health condition,
and claiming the absence of any risk or possibility of flight on his part.
​The CA denied his application on the ground that the discretion to extend bail during the
course of appeal should be exercised with grave caution and only for strong reasons. That bail is
not a sick pass for an ailing or aged detainee or a prisoner needing medical care outside the
prison facility.
​On this matter, Levisete questioned the ruling of the CA and averred that the CA
committed grave abuse of discretion in the denial of his application for bail considering that none
of the conditions justifying denial of bail under the Sec. 5 (3) Rule 114 of the Rules of Court was
present. That when the penalty imposed by the trial court is more than six years but not more
than 20 years and the circumstances in the above-mentioned provision are absent, bail must be
granted to an appellant pending appeal.

ISSUE:
​Whether or not the CA committed grave abuse of discretion in denying the application for
bail of Leviste.

HELD:
​No, under Sec 5 of Rule 114 bail is discretionary, upon conviction by the RTC of an
offense not punishable by death, reclusion perpetua, or life imprisonment. Under par. 3 of the
same rule if the penalty impose is more than 6 years the accused shall be denied bail, or his bail
be cancelled upon a showing by the prosecution, with notice to the accused, of the following or
other circumstances:
30. that he is a recidivist, quasi-recidivist, or habitual delinquent, or has committed the crime
aggravated by the circumstance of reiteration;
31. that he has previously escaped from legal confinement, evaded sentence, or violated the
conditions of his bail without a valid justification;
32. that he committed the offense while under probation, parole, or conditional pardon;
33. that the circumstances of his case indicate the probability of flight if released on bail; or
34. that there is undue risk that he may commit another crime during the pendency of the appeal.
​That bail is expressly declared to be discretionary pending appeal and it cannot be said
that CA committed grave abuse of discretion. After conviction by the trial court, the presumption
of innocence terminates and, accordingly, the constitutional right to bail ends, from then on the
grant of bail is subject to judicial discretion.

ENRILE VS SANDIGANBAYAN

DOCTRINE:
​Primary objective of bail – The strength of the Prosecution's case, albeit a good measure
of the accused's propensity for flight or for causing harm to the public, is subsidiary to the
primary objective of bail, which is to ensure that the accused appears at trial.
​Bail is a right and a matter of discretion – Right to bail is afforded in Sec. 13, Art III of
the 1987 Constitution and repeted in Sec. 7, Rule 114 of the Rules of Criminal Procedure to wit:
“No person charged with a capital offense, or an offense punishable by reclusion perpetua or life
imprisonment, shall be admitted to bail when evidence of guilt is strong, regardless of the stage
of the criminal prosecution.”

FACTS:
​On June 5, 2014, Petitioner Juan Ponce Enrile was charged with plunder in the
Sandiganbayan on the basis of his purported involvement in the Priority Development Assistance
Fund (PDAF) Scam. Initially, Enrile in an Omnibus Motion requested to post bail, which the
Sandiganbayan denied. On July 3, 2014, a warrant for Enrile's arrest was issued, leading to
Petitioner's voluntary surrender.
​Petitioner again asked the Sandiganbayan in a Motion to Fix Bail which was heard by the
Sandiganbayan. Petitioner argued that: (a) Prosecution had not yet established that the evidence
of his guilt was strong; (b) that, because of his advanced age and voluntary surrender, the penalty
would only be reclusion temporal, thus allowing for bail and; (c) he is not a flight risk due to his
age and physical condition. Sandiganbayan denied this in its assailed resolution. Motion for
Reconsideration was likewise denied.

ISSUES:
35. Whether or not bail may be granted as a matter of right unless the crime charged is
punishable by reclusion perpetua where the evidence of guilt is strong.
36. Whether or not petitioner is bailable because he is not a flight risk.
HELD:
1. ​YES. Bail as a matter of right – due process and presumption of innocence.
Article III, Sec. 14 (2) of the 1987 Constitution provides that in all criminal prosecutions, the
accused shall be presumed innocent until the contrary is proved. This right is safeguarded by the
constitutional right to be released on bail.
​The purpose of bail is to guarantee the appearance of the accused at trial and so the
amount of bail should be high enough to assure the presence of the accused when so required,
but no higher than what may be reasonably calculated to fulfill this purpose.
Bail as a matter of discretion
​Right to bail is afforded in Sec. 13, Art III of the 1987 Constitution and repeted in Sec. 7,
Rule 114 of the Rules of Criminal Procedure to wit:
​Capital offense of an offense punishable by reclusion perpetua or life imprisonment, not
bailable. — No person charged with a capital offense, or an offense punishable by reclusion
perpetua or life imprisonment, shall be admitted to bail when evidence of guilt is strong,
regardless of the stage of the criminal prosecution.
​The general rule: Any person, before conviction of any criminal offense, shall be
bailable.
Exception: Unless he is charged with an offense punishable with reclusion perpetua [or life
imprisonment] and the evidence of his guilt is strong.
​Thus, denial of bail should only follow once it has been established that the evidence of
guilt is strong. Where evidence of guilt is not strong, bail may be granted according to the
discretion of the court.
Thus, Sec. 5 of Rule 114 also provides:
​Bail, when discretionary. — Upon conviction by the Regional Trial Court of an offense
not punishable by death, reclusion perpetua, or life imprisonment, admission to bail is
discretionary. The application for bail may be filed and acted upon by the trial court despite the
filing of a notice of appeal, provided it has not transmitted the original record to the appellate
court. However, if the decision of the trial court convicting the accused changed the nature of the
offense from non-bailable to bailable, the application for bail can only be filed with and resolved
by the appellate court.
​Should the court grant the application, the accused may be allowed to continue on
provisional liberty during the pendency of the appeal under the same bail subject to the consent
of the bondsman.
​If the penalty imposed by the trial court is imprisonment exceeding six (6) years, the
accused shall be denied bail, or his bail shall be cancelled upon a showing by the prosecution,
with notice to the accused, of the following or other similar circumstances:
a. That he is a recidivist, quasi-recidivist, or habitual delinquent, or has committed the crime
aggravated by the circumstance of reiteration;
b. That he has previously escaped from legal confinement, evaded sentence, or violated the
conditions of his bail without valid justification;
c. That he committed the offense while under probation, parole, or conditional pardon;
d. That the circumstances of his case indicate the probability of flight if released on bail; or
e. That there is undue risk that he may commit another crime during the pendency of the appeal.
​The appellate court may, motu proprio or on motion of any party, review the resolution of
the Regional Trial Court after notice to the adverse party in either case.
​Thus, admission to bail in offenses punished by death, or life imprisonment, or reclusion
perpetua subject to judicial discretion. In Concerned Citizens vs. Elma, the court held: “[S]uch
discretion may be exercised only after the hearing called to ascertain the degree of guilt of the
accused for the purpose of whether or not he should be granted provisional liberty.” Bail hearing
with notice is indispensable (Aguirre vs. Belmonte). The hearing should primarily determine
whether the evidence of guilt against the accused is strong.
The procedure for discretionary bail is described in Cortes vs. Catral:
f. In all cases, whether bail is a matter of right or of discretion, notify the prosecutor of the
hearing of the application for bail or require him to submit his recommendation (Section 18,
Rule 114 of the Rules of Court as amended);
g. Where bail is a matter of discretion, conduct a hearing of the application for bail regardless of
whether or not the prosecution refuses to present evidence to show that the guilt of the
accused is strong for the purpose of enabling the court to exercise its sound discretion;
(Section 7 and 8, supra)
h. Decide whether the guilt of the accused is strong based on the summary of evidence of the
prosecution;
i. If the guilt of the accused is not strong, discharge the accused upon the approval of the
bailbond (Section 19, supra) Otherwise petition should be denied.

2. ​YES. Petitioner's poor health justifies his admission to bail. The Supreme Court took
note of the Philippine's responsibility to the international community arising from its
commitment to the Universal Declaration of Human Rights. We therefore have the responsibility
of protecting and promoting the right of every person to liberty and due process and for detainees
to avail of such remedies which safeguard their fundamental right to liberty. Quoting from
Government of Hong Kong SAR vs. Olalia, the SC emphasized:
​x x x uphold the fundamental human rights as well as value the worth and dignity of
every person. This commitment is enshrined in Section II, Article II of our Constitution which
provides: “The State values the dignity of every human person and guarantees full respect for
human rights.” The Philippines, therefore, has the responsibility of protecting and promoting the
right of every person to liberty and due process, ensuring that those detained or arrested can
participate in the proceedings before a court, to enable it to decide without delay on the legality
of the detention and order their release if justified. In other words, the Philippine authorities are
under obligation to make available to every person under detention such remedies which
safeguard their fundamental right to liberty. These remedies include the right to be admitted to
bail. (emphasis in decision)
​Sandiganbayan committed grave abuse of discretion. They arbitrarily ignored the
objective of bail to ensure the appearance of the accused during the trial and unwarrantedly
disregarded the clear showing of the fragile health and advanced age of Petitioner. As such the
Sandiganbayan gravely abused its discretion in denying the Motion to Fix Bail. It acted
whimsically and capriciously and was so patent and gross as to amount to an evasion of a
positive duty [to allow petitioner to post bail].

RULE 115 – RIGHTS OF THE ACCUSED

DEL CASTILLO VS PEOPLE

DOCTRINE:
​While it is not necessary that the property to be searched or seized should be owned by
the person against whom the search warrant is issued, there must be sufficient showing that the
property is under appellants control or possession.

FACTS:
​Pursuant to a confidential information that petitioner Del Castillo was engaged in selling
shabu, police officers headed by SPO3 Bienvenido Masnayon, after conducting surveillance and
test-buy operation at the house of petitioner, secured a search warrant from the RTC. Upon
arrival to the residence of Del Castillo to implement the search warrant, SPO3 Masnayon
claimed that he saw petitioner run towards a small structure, a nipa hut, in front of his house.
Masnayon chased him but to no avail, because he and his men were not familiar with the
entrances and exits of the place. They all went back to the residence of Del Castillo and
requested his men to get a barangay tanod and a few minutes thereafter, his men returned with
two barangay tanods who searched the house of petitioner including the nipa hut where the
petitioner allegedly ran for cover. His men who searched the residence of the petitioner found
nothing, but one of the barangay tanods was able to confiscate from the nipa hut several articles,
including four (4) plastic packs containing white crystalline substance.
​Thus, an information was filed against Del Castillo for violation of Section 16, Article III
of R.A. 6425 and was found guilty by the RTC and affirmed by the Court of Appeals. Petitioner
filed with the Supreme Court the petition for certiorari contending among others that CA erred in
finding him guilty beyond reasonable doubt of illegal possession of prohibited drugs, because he
could not be presumed to be in possession of the same just because they were found inside the
nipa hut.

ISSUE:
​Whether the search made in the nipa hut is valid.
HELD:
​No, The records are void of any evidence to show that petitioner owns the nipa hut in
question nor was it established that he used the said structure as a shop. The RTC, as well as the
CA, merely presumed that petitioner used the said structure due to the presence of electrical
materials, the petitioner being an electrician by profession.
​The prosecution must prove that the petitioner had knowledge of the existence and
presence of the drugs in the place under his control and dominion and the character of the drugs.
With the prosecution’s failure to prove that the nipa hut was under petitioner’s control and
dominion, there casts a reasonable doubt as to his guilt. In considering a criminal case, it is
critical to start with the law’s own starting perspective on the status of the accused — in all
criminal prosecutions, he is presumed innocent of the charge laid unless the contrary is proven
beyond reasonable doubt. Proof beyond reasonable doubt, or that quantum of proof sufficient to
produce a moral certainty that would convince and satisfy the conscience of those who act in
judgment, is indispensable to overcome the constitutional presumption of innocence.

MIGUEL VS SANDIGANBAYAN

DOCTRINE:
​To be heard does not only mean oral arguments in court; one may be heard also through
pleadings. Where opportunity to be heard, either through oral arguments or pleadings, has been
accorded, no denial of procedural due process exists.

FACTS:
​Then Vice Mayor Mercelita M. Lucido and other local officials of Koronadal City, South
Cotabato filed a letter- complaint with the Office of the Ombudsman-Mindanao (Ombudsman)
charging the petitioner, among others, with violation of Republic Act (R.A.) No. 3019, in
connection with the consultancy services for the architectural aspect, the engineering design, and
the construction supervision and management of the proposed Koronadal City public market
In a June 27, 1996 order, the Ombudsman directed the petitioner, among others, to submit his
counter-affidavit. On October 23, 1996, after moving for an extension, the petitioner filed his
counter-affidavit.
​The Sandiganbayan ordered the Office of the Special Prosecutor (OSP) to conduct a
reinvestigation. On August 21, 2000, the petitioner, through counsel, followed suit and orally
moved for a reinvestigation, which the Sandiganbayan likewise granted. The Sandiganbayan
gave the petitioner ten (10) days within which to file his counter-affidavit with the OSP.
​The petitioner asked for extension twice and still failed to file his counter-affidavit.
Despite the extension period asked and given, the petitioner failed to file his counter-affidavit,
prompting Prosecutor Norberto B. Ruiz to declare that the petitioner had waived his right to
submit countervailing evidence (April 25, 2001 resolution). On July 31, 2001, then Ombudsman
Aniano Desierto approved the resolution.
​The OSP filed a Motion to Suspend the petitioner Pendente Lite. The petitioner filed his
Vigorous Opposition based on the obvious and fatal defect of the information in failing to allege
that the giving of unwarranted benefits and advantages was done through manifest partiality,
evident bad faith or gross inexcusable negligence, since he was not accorded a pre- suspension
hearing.

ISSUE:
​Whether the information against petitioner and petitioner's suspension valid.
HELD:
​Yes. In deference to the constitutional right of an accused to be informed of the nature
and the cause of the accusation against him, Section 6, Rule 110 of the Revised Rules of
Criminal Procedure (Rules) requires, inter alia, that the information shall state the designation of
the offense given by the statute and the acts or omissions imputed which constitute the offense
charged. Additionally, the Rules requires that these acts or omissions and its attendant
circumstances must be stated in ordinary and concise language and in terms sufficient to enable a
person of common understanding to know what offense is being charged x x x and for the court
to pronounce judgment.
​The test of the informations sufficiency is whether the crime is described in intelligible
terms and with such particularity with reasonable certainty so that the accused is duly informed
of the offense charged. In particular, whether an information validly charges an offense depends
on whether the material facts alleged in the complaint or information shall establish the essential
elements of the offense charged as defined in the law. The raison detre of the requirement in the
Rules is to enable the accused to suitably prepare his defense.
​In arguing against the validity of the information, the petitioner appears to go beyond the
standard of a person of common understanding in appreciating the import of the phrase acting
with evident bad faith and manifest partiality. A reading of the information clearly reveals that
the phrase acting with evident bad faith and manifest partiality was merely a continuation of the
prior allegation of the acts of the petitioner, and that he ultimately acted with evident bad faith
and manifest partiality in giving unwarranted benefits and advantages to his co-accused private
individuals. This is what a plain and non-legalistic reading of the information would yield.
Suspension is valid.
​Section 13 of R.A. No. 3019 reads:
​Section 13. Suspension and loss of benefits. Any public officer against whom any
criminal prosecution under a valid information under this Act or under the provisions of the
Revised Penal Code on bribery is pending in court, shall be suspended from office. Should he be
convicted by final judgment, he shall lose all retirement or gratuity benefits under any law, but if
he is acquitted, he shall be entitled to reinstatement and to the salaries and benefits which he
failed to receive during suspension, unless in the meantime administrative proceedings have been
filed against him.
​Since a pre-suspension hearing is basically a due process requirement, when an accused
public official is given an adequate opportunity to be heard on his possible defenses against the
mandatory suspension under R.A. No. 3019, then an accused would have no reason to complain
that no actual hearing was conducted. It is well settled that to be heard does not only mean oral
arguments in court; one may be heard also through pleadings. Where opportunity to be heard,
either through oral arguments or pleadings, has been accorded, no denial of procedural due
process exists.

PEOPLE VS LARA

DOCTRINE:
​The right to counsel is deemed to have arisen at the precise moment custodial
investigation begins and being made to stand in a police line-up is not the starting point or a part
of custodial investigation.
FACTS:
PROSECUTION:
3 witnesses: Enrique Sumulong, SPO1 Bernard Cruz and PO3 Efren Calix
SUMULONG:
• May 31, 2001, 9:00 AM, he withdrew the amount of P230,000.00 from the Metrobank-Mabini
Branch, Pasig City to defray the salaries of the employees of San Sebastian and while at around
10:30 AM, while the pickup he was riding was at the intersection of Mercedes and Market
Avenues, Pasig City, Lara suddenly appeared at the front passenger side of the pick-up and
pointed a gun at him stating, "Akin na ang pera, iyong bag, nasaan?”;
• Bautista, one of those who accompanied him told him not to give the bag. He threw the bag in
Bautista's direction and Bautista alighted from the pick-up and ran. Seeing Bautista, Lara ran
after him while firing his gun
• He then ran towards Mercedes Plaza and called up the office of San Sebastian to relay the
incident and when he went back to where the pick-up was parked, he went to the rear portion
of the vehicle and saw blood on the ground;
• He was informed by one bystander that Bautista was shot and the bag was taken away from
him; Information charging Lara with robbery with homicide was filed with the RTC. Following
Lara's plea of not guilty, trial ensued.
• June 7, 2001: While on his way to Pasig City, he saw Lara walking along Dr. Pilapil Street, San
Miguel, Pasig City and he alerted the police and Lara was thereafter arrested. At the police
station, he, Atie and Manacob (other companions at pickup) identified Lara as the one who
shot and robbed them of San Sebastian's money.
SPO1 CRUZ:
• Around 7:55 PM on June 7, 2001, Sumulong went to the police station and informed him that
he saw Lara walking along Dr. Pilapil Street
• 4 police officers and Sumulong went to Dr. Pilapil Street where they saw Lara, who Sumulong
identified and they then approached Lara and invited him for questioning.
• At the police station, Lara was placed in a line-up where he was positively identified by
Sumulong, Manacob and Atie; and after being identified, Lara was informed of his rights and
subsequently detained.
PO3 CALIX:
• May 31, 2001: he was informed of a robbery that took place and he, together with 3 other
police officers, proceeded to the crime scene wherein upon arriving one of the police officers
who were able to respond ahead of them, handed to him 11 pieces of empty shells and 6
deformed slugs of a 9mm pistol;
• As part of his investigation, he interviewed Sumulong, Atie, Manacob at the police station; and
before Bautista died, he was able to interview Bautista at the hospital where the latter was
brought after the incident.
DEFENSE:
LARA
• May 31, 2001: he was at his house, digging a sewer trench while his brother, Wilfredo, was
constructing a comfort room which was corroborated by his sister, Edjosa Manalo and
neighbor, Simplicia Delos Reyes.
• June 7, 2001 and at around 7:00 in the evening, while he was at the house of one of his cousins,
police officers arrived and asked him if he was Arturo Lara and after confirming that he was
Arturo Lara, the police officers asked him to go with them to the Barangay Hall. He voluntarily
went with them and while inside the patrol car, one of the policemen said,"You are lucky, we
were able to caught you in your house, if in another place we will kill you”.
• He was brought to the police station and not the barangay hall as he was earlier told where he
was investigated for robbery with homicide and when he told the police that he was at home
when the subject incident took place, the police challenged him to produce witnesses but when
his witnesses arrived at the station, one of the police officers told them to come back the
following day.
• While he was at the police line-up holding a name plate, a police officer told Sumulong and
Atie, "Ituru nyo na yan at uuwi na tayo"; and when his witnesses arrived the following day,
they were told that he will be subjected to an inquest.
RTC
• GUILTY of robbery with homicide sentenced to suffer the penalty of imprisonment of reclusion
perpetua, with all the accessory penalties prescribed by law.
• Rejected Lara's defense of alibi as follows because Enrique Sumulong positively identified
accused Arturo Lara as the person who carted away the payroll money of San Sebastian Allied
Services, and the one who shot Joselito Bautista which caused his instantaneous death on the
same day. Also, it is not impossible for him to be at the place.
ARGUMENTS:
• On appeal, Lara pointed out several errors that supposedly attended his conviction. First, that he
was arrested without a warrant under circumstances that do not justify a warrantless arrest
rendered void all proceedings including those that led to his conviction.
• Second, he was not assisted by counsel when the police placed him in a line-up to be identified
by the witnesses for the prosecution in violation of Section 12, Article III of the Constitution.
The police line-up is part of custodial investigation and his right to counsel had already
attached.
• Third, the prosecution failed to prove his guilt beyond reasonable doubt. Specifically, the
prosecution failed to present a witness who actually saw him commit the alleged acts.
Sumulong merely presumed that he was the one who shot Bautista and who took the bag of
money from him. The physical description of Lara that Sumulong gave to the police was
different from the one he gave during the trial, indicating that he did not have a fair glimpse of
the perpetrator. Moreover, this gives rise to the possibility that it was his unidentified
companion who shot Bautista and took possession of the money. Hence, it cannot be
reasonably claimed that his conviction was attended with moral certainty.
• Fourth, the trial court erred in discounting the testimony of his witnesses. Without any showing
that they were impelled by improper motives in testifying in his favor, their testimonies should
have been given the credence they deserve. While his two (2) witnesses were his sister and
neighbor, this does not by itself suggest the existence of bias or impair their credibility.
CA: AFFIRMED conviction.
​- AUTOMATIC APPEAL TO SC as the penalty imposed was reclusion perpetua.
ISSUE:
​Whether the identification made by Sumulong, Atie and Manacob in the police line-up is
inadmissible because Lara stood therein without the assistance of counsel? (NO, identification
not custodial investigation)

HELD:
​NO. There was no legal compulsion to afford him a counsel during a police line-up since
the latter is not part of custodial investigation and this does not constitute a violation of his right
to counsel.
​That he stood at the police line-up without the assistance of counsel did not render
Sumulong's identification of Lara inadmissible. The right to counsel is deemed to have arisen at
the precise moment custodial investigation begins and being made to stand in a police line-up is
not the starting point or a part of custodial investigation.
​People v. Amestuzo: The guarantees of Sec. 12 (1), Art. III of the 1987 Constitution, or
the so-called Miranda rights, may be invoked only by a person while he is under custodial
investigation. Custodial investigation starts when the police investigation is no longer a general
inquiry into an unsolved crime but has begun to focus on a particular suspect taken into custody
by the police who starts the interrogation and propounds questions to the person to elicit
incriminating statements.
​Police line-up is not part of the custodial investigation; hence, the right to counsel
guaranteed by the Constitution cannot yet be invoked at this stage.
​The right to be assisted by counsel attaches only during custodial investigation and
cannot be claimed by the accused during identification in a police line-up because it is not part of
the custodial investigation process. This is because during a police line-up, the process has not
yet shifted from the investigatory to the accusatory and it is usually the witness or the
complainant who is interrogated and who gives a statement in the course of the line-up.
​An exception to this rule is when the accused had been the focus of police attention at the
start of the investigation. In the case at bench, appellant was identified in a police line-up by
prosecution witnesses from a group of persons gathered for the purpose. However, there was no
proof that appellant was interrogated at all or that a statement or confession was extracted from
him. During the police line-up, the accusatory process had not yet commenced.
​Assuming there was interrogation, any allegation of violation of rights during custodial
investigation is relevant and material only to cases in which an extrajudicial admission or
confession extracted from the accused becomes the basis of their conviction. Here, appellant
was convicted based on the testimony of a prosecution witness and not on his alleged
uncounseled confession or admission.

SANICO VS PEOPLE

DOCTRINE:
​If the incompetence of counsel was so great and the error committed as a result was so
serious that the client was prejudiced by a denial of his day in court, the litigation ought to be
reopened to give to the client another chance to present his case.
FACTS:
​The petitioner Sanico and Marsito Batiquin were criminally charged for trespassing and
theft of minerals in the Municipal Circuit Trial Court in Cebu. The MCTC found both accused
guilty beyond reasonable doubt.
​Sanico’s counsel filed a notice of appeal in the MCTC. Consequently, the RTC, Branch
25, in Danao City ordered Sanico to file his memorandum on appeal. Sanico did not comply;
hence, the RTC ruled for the dismissal of the appeal.
​Later on, one Atty. Dennis Cañete, another lawyer acting for Sanico, filed a motion for
reconsideration vis-à-vis the dismissal of the appeal, stating that Sanico had not filed the
memorandum on appeal because he had been beset with problems due to his wife’s debilitating
illness which eventually claimed her life, as well as his counsel, Atty. Baring’s own medical
condition which caused her to forget how she got this case and whom to contact as principal
counsel hereof.
​The RTC denied the motion for reconsideration because of its lack of verification and
affidavit of merit; and because the supposed sickness of Sanico’s wife and the lapses of Atty.
Baring were not justifiable reasons.
​Sanico, through Atty. Cañete, filed a petition for review in the CA, contesting his
conviction, and assailing the dismissal of his appeal for failure to file the memorandum on
appeal.
CA denied the petition for review on the following grounds, namely:
j. the docket fees were not paid;
k. there was no proper proof of service of a copy of the petition for review on the adverse party;
l. the petitioner did not furnish to the RTC a copy of the petition for review;
m. there was no affidavit of service;
n. no written explanation for not resorting to personal filing was filed;
o. the documents appended to the petition were only plain photocopies of the certified true
copies;
p. no copies of pleadings and other material portions of the record were attached;
q. the verification and certification of non-forum shopping were defective due to failure to
contain a statement that the allegations therein were based on the petitioner’s personal
knowledge;
r. the verification and certification of non-forum shopping did not contain competent evidence of
identity of the petitioner; and
s. the serial number of the commission of the notary public and the office address of the notary
public were not properly indicated.
​The petitioner moved for reconsideration, but his motion was denied. In the meantime,
the counsel for respondent Jennifer S. Tenio filed an Ex Parte Motion for Entry of Judgment,
which the RTC authorized the issuance of. Sanico filed an omnibus motion to recall the order
and to quash the entry of judgment, but the RTC denied the omnibus motion, noting that Sanico
did not provide the RTC with a copy of his petition for review; hence, the RTC had no way of
knowing about the pendency of his petition for review in the CA; and that in any case, the CA
had already denied his petition for review, while his motion for reconsideration had yet to be
acted upon by the CA. Sanico’s motion for reconsideration was denied by the RTC. Hence, this
petition for review on certiorari.

ISSUE:
​Whether or not the negligence of the petitioner’s counsel should be binding on the
petitioner.

HELD:
​No, without doubt, the petitioner could reasonably expect that his counsel would afford to
him competent legal representation. The mere failure of the counsel to observe a modicum of
care and vigilance in the protection of the interests of the petitioner as the client — as manifested
in the multiple defects and shortcomings discovered in the petition for review — was gross
negligence in any language because the defects were plainly avoidable by the simple application
of the relevant guidelines existing in the Rules of Court. If the incompetence of counsel was so
great and the error committed as a result was so serious that the client was prejudiced by a denial
of his day in court, the litigation ought to be reopened to give to the client another chance to
present his case. The legitimate interests of the petitioner, particularly the right to have his
conviction reviewed by the RTC as the superior tribunal, should not be sacrificed in the altar of
technicalities.

PEOPLE VS AYSON

DOCTRINE:
​The rights guaranteed a person under Art. III, Sec. 12 of the Constitution are not available
when he is not under custodial investigation. Thus, a statement or confession voluntarily given
by an employee during an administrative investigation that he had malversed his employer’s
funds is admissible although without a prior information of said rights and without the assistance
of counsel.

FACTS:
​Felipe Ramos was a ticket freight clerk of the Philippine Airlines, assigned at its Baguio
City station. It was alleged that he was involved in irregularities in the sales of plane tickets, the
PAL management notified him of an investigation to be conducted. That investigation was
scheduled in accordance with PAL's Code of Conduct and Discipline, and the Collective
Bargaining Agreement signed by it with the Philippine Airlines Employees' Association
(PALEA) to which Ramos pertained. A letter was sent by Ramos stating his willingness to settle
the amount of P76,000. The findings of the Audit team were given to him, and he refuted that he
misused proceeds of tickets also stating that he was prevented from settling said amounts. He
proffered a compromise however this did not ensue. Two months after a crime of estafa was
charged against Ramos. Ramos pleaded not guilty. Evidence by the prosecution contained
Ramos’ written admission and statement, to which defendants argued that the confession was
taken without the accused being represented by a lawyer. Respondent Judge did not admit those
stating that accused was not reminded of his constitutional rights to remain silent and to have
counsel. A motion for reconsideration filed by the prosecutors was denied. Hence this appeal.

ISSUE:
​Whether or Not the respondent Judge correct in making inadmissible as evidence the
admission and statement of accused.

HELD:
​No, the handwritten admission and statement of accused were declared to be admissible.
Right Against Self-Incrimination
​The first right, against self-incrimination, mentioned in Section 20, Article IV of the 1973
Constitution, is accorded to every person who gives evidence, whether voluntarily or under
compulsion of subpoena, in any civil, criminal, or administrative proceeding. The right is NOT
to "be compelled to be a witness against himself.”
​The precept set out in that first sentence has a settled meaning.It prescribes an "option of
refusal to answer incriminating questions and not a prohibition of inquiry.” It simply secures to a
witness, whether he be a party or not, the right to refue to answer any particular incriminatory
question, i.e., one the answer to which has a tendency to incriminate him for some crime.
However, the right can be claimed only when the specific question, incriminatory in character, is
actually put to the witness. It cannot be claimed at any other time. It does not give a witness the
right to disregard a subpoena, to decline to appear before the court at the time appointed, or to
refuse to testify altogether. The witness receiving a subpoena must obey it, appear as required,
take the stand, be sworn and answer questions. It is only when a particular question is addressed
to him, the answer to which may incriminate him for some offense, that he may refuse to answer
on the strength of the constitutional guaranty.
​That first sentence of Section 20, Article IV of the 1973 Constitution does not impose on
the judge, or other officer presiding over a trial, hearing or investigation, any affirmative
obligation to advise a witness of his right against self-incrimination. It is a right that a witness
knows or should know, in accordance with the well-known axiom that everyone is presumed to
know the law, that ignorance of the law excuses no one. Furthermore, in the very nature of
things, neither the judge nor the witness can be expected to know in advance the character or
effect of a question to be put to the latter.
​The right against self-incrimination is not self- executing or automatically operational. It
must be claimed. If not claimed by or in behalf of the witness, the protection does not come into
play. It follows that the right may be waived, expressly, or impliedly, as by a failure to claim it at
the appropriate time.

VILLAREAL VS PEOPLE

DOCTRINE:
FACTS: In February 1991, seven freshmen law students of the Ateneo de Manila University
School of Law signified their intention to join the Aquila Legis Juris Fraternity (Aquila
Fraternity). The neophytes, including victim, Lenny Villa, were subjected to initiation rites. After
the second day of initiation rites has ended, accused non-resident or alumni fraternity members
Fidelito Dizon (Dizon) and Artemio Villareal (Villareal) demanded that the rites be reopened.
The head of initiation rites, Nelson Victorino (Victorino), initially refused. Upon the insistence of
Dizon and Villareal, however, he reopened the initiation rites. The fraternity members, including
Dizon and Villareal, then subjected the neophytes to "paddling" and to additional rounds of
physical pain. Lenny received several paddle blows, one of which was so strong it sent him
sprawling to the ground. The neophytes heard him complaining of intense pain and difficulty in
breathing. After their last session of physical beatings, Lenny could no longer walk. He had to be
carried by the auxiliaries to the carport. Again, the initiation for the day was officially ended, and
the neophytes started eating dinner. They then slept at the carport. After an hour of sleep, the
neophytes were suddenly roused by Lennys shivering and incoherent mumblings. Initially,
Villareal and Dizon dismissed these rumblings, as they thought he was just overacting. When
they realized, though, that Lenny was really feeling cold, some of the Aquilans started helping
him. They removed his clothes and helped him through a sleeping bag to keep him warm. When
his condition worsened, the Aquilans rushed him to the hospital. Lenny was pronounced dead on
arrival. Consequently, a criminal case for homicide was filed against 35 Aquilans.
The case however did not proceed until 2002. The unexplained length of time in which the case
remained dormant caused the dismissal of the case for being in violation of the accuseds’
constitutionally guaranteed right to a speedy trial.

ISSUE:
​Whether the CA correctly dismissed the case for violation of the accuseds’ right to speedy
trial
HELD:
​Yes. The SC did not see any grave abuse of discretion in the CAs dismissal of the case
against accused Escalona, Ramos, Saruca, and Adriano on the basis of the violation of their right
to speedy trial.
​The absence of the records in the trial court [was] due to the fact that the records of the
case were elevated to the Court of Appeals, and the prosecutions failure to comply with the order
of the court a quo requiring it to secure certified true copies of the same. What is glaring from the
records is the fact that as early as September 21, 1995, the court a quo already issued an Order
requiring the prosecution, through the Department of Justice, to secure the complete records of
the case from the Court of Appeals. The prosecution did not comply with the said Order as in
fact, the same directive was repeated by the court a quo in an Order dated December 27, 1995.
Still, there was no compliance on the part of the prosecution. It is not stated when such order was
complied with. It appears, however, that even until August 5, 2002, the said records were still not
at the disposal of the trial court because the lack of it was made the basis of the said court in
granting the motion to dismiss filed by co-accused Concepcion.
​It is likewise noticeable that from December 27, 1995, until August 5, 2002, or for a
period of almost seven years, there was no action at all on the part of the court a quo. Except for
the pleadings filed by both the prosecution and the petitioners, the latest of which was on January
29, 1996, followed by petitioner Sarucas motion to set case for trial on August 17, 1998 which
the court did not act upon, the case remained dormant for a considerable length of time. This
prolonged inactivity whatsoever is precisely the kind of delay that the constitution frowns upon.
The right to a speedy trial is violated when the proceeding is attended with unjustified
postponements of trial, or when a long period of time is allowed to elapse without the case being
tried and for no cause or justifiable motive.

RULE 116 – ARRAIGNMENT AND PLEA

PEOPLE VS ESTOMACA

DOCTRINE:
​Section 1(a) of Rule 116 requires that the arraignment should be made in open court by
the judge himself or by the clerk of court furnishing the accused a copy of the complaint or
information with the list of witnesses stated therein, then reading the same in the language or
dialect that is known to him, and asking him what his plea is to the charge. The requirement that
the reading be made in a language or dialect that the accused understands and knows is a
mandatory requirement, just as the whole of said Section 1 should be strictly followed by trial
courts. This the law affords the accused by way of implementation of the all-important
constitutional mandate regarding the right of an accused to be informed of the precise nature of
the accusation leveled at him and is, therefore, really an avenue for him to be able to hoist the
necessary defense in rebuttal thereof. It is an integral aspect of the due process clause under the
Constitution.

FACTS:
​Estomaca is an illiterate laborer accused of raping his own daughter, Estelita. 5
complaints were filed and 2 of them are being challenged. These instances happened sometime
in 1993 and 1994 which both took place in their residence in Iloilo. The court imposed the
penalty of Reclusion Perpetua for the sexual assault in 1993 and the penalty of death for the
alleged rape in 1994.
​The records show, however, that there was an irregularity in his arraignment where the
complaints were supposed to have all been read to him in the Ilonggo/local dialect. In this said
arraignment, the accused pleaded guilty to all 5 complaints but subsequently stated the he wasn’t
guilty of the other 3 cases against him

ISSUE:
​Whether the arraignment was valid or not
HELD:
​No. The transcript of the arraignment shows that it merely consisted of the bare reading
of the 5 complaints. It was reported in the transcript that “Reading the information/complaint to
the accused in Ilonggo/local dialect”. Since it was stated in the singular, the court speculates
whether all 5 complaints were actually ready, translated or explained to him on a level within his
comprehension considering his limited education. Parenthetically, there was no statement of
record that appellant fully understood that medium of expression. This assumes added
significance since Ilonggo, or properly called Hiligaynon, is a regional language, spoken in a
major part of Iloilo province, Negros Occidental and, with variations, in Capiz. Within a
province or major geographical area using a basic regional language, there may be other local
dialects spoken in certain parts thereof. If said indication in the aforequoted portion of the
transcript intended to convey that Ilonggo is merely a local dialect and was also the idiom
referred to, the same is egregious error; it would be different if local dialect was used to denote
an alternative and different medium but, inexplicably, without identifying what it was. The
significance of this distinction is found right in the provision of Section 1(a) of Rule 116 which,
cognizant of the aforestated linguistic variations, deliberately required that the complaint or
information be read to the accused in the language or the dialect known to him, to ensure his
comprehension of the charges. The Court takes judicial notice, because it is either of public
knowledge or readily capable of unquestionable demonstration, that in the central and
northwestern part of Iloilo province and all the way up to and throughout Antique, including
necessarily San Joaquin where the offenses were committed and of which appellant and his
family are natives, the local dialect is known as kinaray-a. Barring previous exposure to or as a
consequence of extended social or commercial intercourse, kinaray-a is not readily
understandable to nor spoken by those born to the Hiligaynon regional language or who have
lived in the areas under its sway for an appreciable period of time. The converse is true for those
whose native tongue is the dialect ofkinaray-a,since they are generally not well-versed in
Ilonggo, or Hiligaynon. Since all the complaints are not only in English but in technical legal
language, we are again at sea as to whether and how the indictments were translated to Ilonggo
and/or to kinaray-a, or that the appellant was truly and honestly made aware of the charges and,
especially, the consequences of his guilty plea thereto. The records are silent and do not reveal
anything on this point, nor how the dialogue between the presiding judge and appellant was
translated. Yet a mans life is at stake while this Court wrestles with that dilemma created by an
omission of official duty.
​Since no valid judgments can be had upon an invalid arraignment, the cases were set
aside and were remanded to the trial court.

PEOPLE VS PANGILINAN

DOCTRINE:
​Procedural defect of belated arraignment was cured when the counsel of the accused
participated in the trial without raising any objection that his client had yet to be arraigned.

FACTS:
​Alfredo Pangilinan was charged with 2 informations for the rape of his daughter, AAA.
He was arrested and detained with no bail recommended.
​He then filed a petition for bail. The petition for bail was heard and prosecution presented
evidence. The Regional Trial Court denied petition Defense then presented its evidence.
The RTC after having discovered that Pangilinan was not yet arraigned, scheduled his
arraignment. Pangilinan pleaded not guilty. RTC convicted him and sentenced him to death.

ISSUE:
​Whether or not the arraignment was valid.

HELD:
​YES. Pangilinan assails his conviction because he was not properly arraigned. Since he
was arraigned only after the case was submitted for decision, said irregularity, he argues, is a
procedural error which is prejudicial to the appellant and is tantamount to denial of his
constitutional right to be informed of the accusation against him. He claims that his subsequent
arraignment did not cure the defect in the trial proceedings because at the time the petition for
bail was heard, the trial court had not yet acquired jurisdiction over his person.
​The accused is mistaken. When the hearings for his petition for bail were conducted, the
trial court had already acquired jurisdiction over his person. Settled is the rule that jurisdiction
over the person of the accused is acquired upon his arrest or voluntary appearance.19 In the case
at bar, the trial court acquired jurisdiction over the person of the appellant when he was arrested
on 19 March 1997. His arrest, not his arraignment, conferred on the trial court jurisdiction over
his person. Arraignment is the formal mode and manner of implementing the constitutional right
of an accused to be informed of the nature and cause of the accusation against him. The purpose
of arraignment is, thus, to apprise the accused of the possible loss of freedom, even of his life,
depending on the nature of the crime imputed to him, or at the very least to inform him of why
the prosecuting arm of the State is mobilized against him.
​Admittedly, appellant was arraigned after the case was submitted for decision. The
question is: Were appellant’s rights and interests prejudiced by the fact that he was arraigned
only at this stage of the proceedings? We do not think so. Appellant’s belated arraignment did
not prejudice him. This procedural defect was cured when his counsel participated in the trial
without raising any objection that his client had yet to be arraigned. In fact, his counsel even
cross-examined the prosecution witnesses. His counsel’s active participation in the hearings is a
clear indication that he was fully aware of the charges against him; otherwise, his counsel would
have objected and informed the court of this blunder. Moreover, no protest was made when
appellant was subsequently arraigned. The parties did not question the procedure undertaken by
the trial court. It is only now, after being convicted and sentenced to two death sentences, that
appellant cries that his constitutional right has been violated. It is already too late to raise this
procedural defect. This Court will not allow it.
​In People v. Cabale and People v. Atienza where the same issue was raised under similar
circumstances, we held that while the arraignment of appellant was conducted after the cases had
been submitted for decision, the error is non- prejudicial and has been fully cured. Since
appellant’s rights and interests were not prejudiced by this lapse in procedure, it only follows that
his constitutional right to be informed of the nature and cause of the accusation against him was
not violated.

DAAN VS SANDIGANBAYAN

DOCTRINE:
37. Plea bargaining in criminal cases is a process whereby the accused and the prosecution work
out a mutually satisfactory disposition of the case subject to court approval—it usually
involves the defendant’s pleading guilty to a lesser offense or to only one or some of the
counts of a multi-count indictment in return for a lighter sentence than that for the graver
charge.
38. Ordinarily, plea bargaining is made during the pre-trial stage of the proceedings but it may
also be made during the trial proper and even after the prosecution has finished presenting its
evidence and rested its case.
FACTS:
​Daan (together with other co-accused) was charged for three counts of malversation of
public funds and three counts of falsification of public document by a public officer or employee
before the Sandiganbayan.
​In the falsification cases, the accused offered to withdraw their plea of "not guilty" and
substitute the same with a plea of "guilty", provided, the mitigating circumstances of confession
or plea of guilt and voluntary surrender will be appreciated in their favor. In the alternative, if
such proposal is not acceptable, said accused proposed instead to substitute their plea of "not
guilty" to the crime of falsification of public document by a public officer or employee with a
plea of "guilty", but to the lesser crime of falsification of a public document by a private
individual. On the other hand, in the malversation cases, the accused offered to substitute their
plea of "not guilty" thereto with a plea of "guilty", but to the lesser crime of failure of an
accountable officer to render accounts.
​Prosecution found this acceptable. Insofar as the malversation cases are concerned, the
prosecution was likewise amenable to the offer of said accused to plead "guilty" to the lesser
crime of failure of an accountable officer to render accounts.
​The Sandiganbayan, however, denied petitioner’s Motion to Plea Bargain, despite
favorable recommendation by the prosecution, on the main ground that no cogent reason was
presented to justify its approval.
​The Sandiganbayan likewise denied the ensuing Motion for Reconsideration. Daan filed a
Rule 65 before the Supreme Court.

ISSUE:
​Whether or not The Sandiganbayan committed grave abuse of discretion in denying
Daan’s motion to plea bargain.

HELD:
​YES. Plea bargaining in criminal cases is a process whereby the accused and the
prosecution work out a mutually satisfactory disposition of the case subject to court approval. It
usually involves the defendant's pleading guilty to a lesser offense or to only one or some of the
counts of a multi-count indictment in return for a lighter sentence than that for the graver charge.
Plea bargaining is authorized under Section 2, Rule 116 of the Revised Rules of Criminal
Procedure, to wit:
​SEC. 2. Plea of guilty to a lesser offense. — At arraignment, the accused, with the
consent of the offended party and the prosecutor, may be allowed by the trial court to plead
guilty to a lesser offense which is necessarily included in the offense charged. After arraignment
but before trial, the accused may still be allowed to plead guilty to said lesser offense after
withdrawing his plea of not guilty. No amendment of the complaint or information is necessary.
(sec. 4, cir. 38-98)
​Ordinarily, plea bargaining is made during the pre-trial stage of the proceedings. Sections
1 and 2, Rule 118 of the Rules of Court, require plea bargaining to be considered by the trial
court at the pre-trial conference,viz:
​SEC. 1. Pre-trial; mandatory in criminal cases. – In all criminal cases cognizable by the
Sandiganbayan, Regional Trial Court, Metropolitan Trial Court, Municipal Trial Court in Cities,
Municipal Trial Court and Municipal Circuit Trial Court, the court shall, after arraignment and
within thirty (30) days from the date the court acquires jurisdiction over the person of the
accused, unless a shorter period is provided for in special laws or circulars of the Supreme Court,
order a pre-trial conference to consider the following:
(t) plea bargaining;
(u) stipulation of facts;
(v) marking for identification of evidence of the parties;
(w) waiver of objections to admissibility of evidence;
(x) modification of the order of trial if the accused admits the charge but interposes a lawful
defense; and
(y) such matters as will promote a fair and expeditious trial of the criminal and civil aspects of the
case.
SEC. 2. Pre-trial agreement. – All agreements or admissions made or entered during the pre-trial
conference shall be reduced in writing and signed by the accused and counsel, otherwise, they
cannot be used against the accused. The agreements covering the matters referred to in section 1
of this Rule shall be approved by the court. (Emphasis supplied)
But it may also be made during the trial proper and even after the prosecution has finished
presenting its evidence and rested its case. Thus, the Court has held that it is immaterial that plea
bargaining was not made during the pre-trial stage or that it was made only after the prosecution
already presented several witnesses.
​Section 2, Rule 116 of the Rules of Court presents the basic requisites upon which plea
bargaining may be made, i.e., that it should be with the consent of the offended party and the
prosecutor,10 and that the plea of guilt should be to a lesser offense which is necessarily included
in the offense charged. The rules however use word may in the second sentence of Section 2,
denoting an exercise of discretion upon the trial court on whether to allow the accused to make
such plea.11 Trial courts are exhorted to keep in mind that a plea of guilty for a lighter offense
than that actually charged is not supposed to be allowed as a matter of bargaining or compromise
for the convenience of the accused.
​In People of the Philippines v. Villarama, the Court ruled that the acceptance of an offer
to plead guilty to a lesser offense is not demandable by the accused as a matter of right but is a
matter that is addressed entirely to the sound discretion of the trial court. However, Villarama
involved plea bargaining after the prosecution had already rested its case.
​As regards plea bargaining during the pre-trial stage, as in the present case, the trial
court's exercise of its discretion should neither be arbitrary nor should it amount to a capricious
and whimsical exercise of discretion. Grave abuse of discretion implies such capricious and
whimsical exercise of judgment as is equivalent to lack of jurisdiction or, in other words, where
the power is exercised in an arbitrary manner by reason of passion, prejudice, or personal
hostility; and it must be so patent or gross as to amount to an evasion of a positive duty or to a
virtual refusal to perform the duty enjoined by law, or to act at all in contemplation of law.
​In the present case, the Sandiganbayan rejected petitioner's plea offer on the ground that
petitioner and the prosecution failed to demonstrate that the proposal would redound to the
benefit of the public. The Sandiganbayan believes that approving the proposal would "only serve
to trivialize the seriousness of the charges against them and send the wrong signal to potential
grafters in public office that the penalties they are likely to face would be lighter than what their
criminal acts would have merited or that the economic benefits they are likely to derive from
their criminal activities far outweigh the risks they face in committing them; thus, setting to
naught the deterrent value of the laws intended to curb graft and corruption in government."
Apparently, the Sandiganbayan has proffered valid reasons in rejecting petitioner's plea offer.
However, subsequent events and higher interests of justice and fair play dictate that petitioner's
plea offer should be accepted. The present case calls for the judicious exercise of this Court's
equity jurisdiction.
​In the cases at bar, there is no dispute that JOSELITO RANIERO J. DAAN has already
restituted the total amount of P18,860.00 as per official receipt issued by the provincial
government of Leyte dated February 26, 2002. In short, the damage caused to the government
has already been restituted by the accused.
​There is also no dispute that accused DAAN voluntarily surrendered in the instant cases.
Moreover, the accused is also willing to plead guilty to a lesser offense which to our mind, merits
consideration.
PEOPLE VS JANJALANI

TOPIC: Valentine’s Day Bombing by Abu Sayyaf; Trinidad and Baharan plead guilty for
multiple murder but plead not guilty for frustrated multiple murder.

DOCTRINE:
39. All trial judges must refrain from accepting with alacrity an accused’s plea of guilty, for
while justice demands a speedy administration, judges are duty bound to be extra solicitous
in seeing to it that when an accused pleads guilty, he understands fully the meaning of his
plea and the import of an inevitable conviction; The requirement for a judge to conduct a
searching inquiry applies more so in cases of re-arraignment.
40. The requirement to conduct a searching inquiry should not be deemed satisfied in cases in
which it was the defense counsel who explained the consequences of a “guilty” plea to the
accused—the conduct of a searching inquiry remains the duty of judges, as they are
mandated by the rules to satisfy themselves that the accused had not been under coercion or
duress; mistaken impressions; or a misunderstanding of the significance, effects, and
consequences of their guilty plea.
41. Convictions based on an improvident plea of guilt are set aside only if such plea is the sole
basis of the judgment.
FACTS:
​An RRCG bus was plying its usual route, from its Navotas bus terminal towards its
Alabang bus terminal via EDSA. Around 6:30 to 7:30 in the evening, while they were about to
move out of the Guadalupe-EDSA southbound bus stop, the bus conductor noticed two men
running after the bus. The two insisted on getting on the bus, so the conductor obliged and let
them in.
​According to Elmer Andales, the bus conductor, he immediately became wary of the two
men, because, even if they got on the bus together, the two sat away from each other—one sat
two seats behind the driver, while the other sat at the back of the bus. At the time, there were
only 15 passengers inside the bus. He also noticed that the eyes of one of the men were reddish.
When he approached the person near the driver and asked him whether he was paying for two
passengers, the latter looked dumb struck by the question. He then stuttered and said he was
paying for two and gave PhP20. Andales grew more concerned when the other man seated at the
back also paid for both passengers. At this point, Andales said he became more certain that the
two were up to no good, and that there might be a holdup.
​Andales said he became more suspicious because both men kept on asking him if the bus
was going to stop at Ayala Avenue.
​As soon as the bus reached the stoplight at the corner of Ayala Avenue and EDSA, the
two men insisted on getting off the bus. According to Andales, the bus driver initially did not
want to let them off the bus, because a Makati ordinance prohibited unloading anywhere except
at designated bus stops. Eventually, the bus driver gave in and allowed the two passengers to
alight. The two immediately got off the bus and ran towards Ayala Avenue. Moments after,
Andales felt an explosion. He then saw fire quickly engulfing the bus. He ran out of the bus
towards a nearby mall. After a while, he went back to where the bus was. He saw their bus
passengers either lying on the ground or looking traumatized. A few hours after, he made a
statement before the Makati Police Station narrating the whole incident.
​Shortly before the explosion, the spokesperson of the Abu Sayyaf Group—Abu Solaiman
— announced over radio station DZBB that the group had a Valentine’s Day “gift” for former
President Gloria Macapagal-Arroyo. After the bombing, he again went on radio and warned of
more bomb attacks.
​As stipulated during pretrial, accused Trinidad gave ABS-CBN News Network an
exclusive interview sometime after the incident, confessing his participation in the Valentine’s
Day bombing incident. In another exclusive interview on the network, accused Baharan likewise
admitted his role in the bombing incident. Finally, accused Asali gave a television interview,
confessing that he had supplied the explosive devices for the 14 February 2005 bombing. The
bus conductor identified the accused Baharan and Trinidad, and confirmed that they were the two
men who had entered the RRCG bus on the evening of 14 February.
​Members of the Abu Sayyaf Group—namely Khaddafy Janjalani, Gamal B. Baharan,
Angelo Trinidad, Gappal Bannah Asali, Jainal Asali, Rohmat Abdurrohim a.k.a. Abu Jackie or
Zaky, and other “John” and “Jane Does”—were then charged with multiple murder and multiple
frustrated murder. Only Baharan, Trinidad, Asali, and Rohmat were arrested, while the other
accused remain at-large.
​On their arraignment for the multiple murder charge, Baharan, Trinidad, and Asali all
entered a plea of guilty. On the other hand, upon arraignment for the multiple frustrated murder
charge, accused Asali pled guilty. Accused Trinidad and Baharan pled not guilty. Rohmat pled
not guilty to both charges.
​In the light of the pretrial stipulations, the trial court asked whether accused Baharan and
Trinidad were amenable to changing their “not guilty” pleas to the charge of multiple frustrated
murder, considering that they pled “guilty” to the heavier charge of multiple murder, creating an
apparent inconsistency in their pleas. Defense counsel conferred with accused Baharan and
Trinidad and explained to them the consequences of the pleas. The two accused acknowledged
the inconsistencies and manifested their readiness for re-arraignment. After the Information was
read to them, Baharan and Trinidad pled guilty to the charge of multiple frustrated murder.

ISSUE:
​Whether or not the trial court gravely erred in accepting accused-appellants’ plea of guilt
despite insufficiency of searching inquiry into the voluntariness and full comprehension of the
consequences of the said plea.

HELD:
​Yes. The SC ruled that the trial court judges are required to observe the following
procedure under Section 3, Rule 116 of the Rules of Court:
​SEC. 3. Plea of guilty to capital offense; reception of evidence.—When the accused
pleads guilty to a capital offense, the court shall conduct a searching inquiry into the
voluntariness and full comprehension of the consequences of his plea and shall require the
prosecution to prove his guilt and the precise degree of culpability. The accused may also present
evidence in his behalf.
The requirement to conduct a searching inquiry applies more so in cases of re-arraignment. In
People v. Galvez, the Court noted that since accused-appellant’s original plea was “not guilty,”
the trial court should have exerted careful effort in inquiring into why he changed his plea to
“guilty.” Likewise, the requirement to conduct a searching inquiry should not be deemed
satisfied in cases in which it was the defense counsel who explained the consequences of a
“guilty” plea to the accused, as it appears in this case. In People v. Alborida, this Court found
that there was still an improvident plea of guilty, even if the accused had already signified in
open court that his counsel had explained the consequences of the guilty plea; that he understood
the explanation of his counsel; that the accused understood that the penalty of death would still
be meted out to him; and that he had not been intimidated, bribed, or threatened.
​The conduct of a searching inquiry remains the duty of judges, as they are mandated by
the rules to satisfy themselves that the accused had not been under coercion or duress; mistaken
impressions; or a misunderstanding of the significance, effects, and consequences of their guilty
plea. This requirement is stringent and mandatory. Nevertheless, the High Court is not unmindful
of the context under which the re-arraignment was conducted or of the factual milieu
surrounding the finding of guilt against the accused. The Court observes that accused Baharan
and Trinidad previously pled guilty to another charge— multiple murder—based on the same act
relied upon in the multiple frustrated murder charge. The Court further notes that prior to the
change of plea to one of guilt, accused Baharan and Trinidad made two other confessions of guilt
—one through an extrajudicial confession (exclusive television interviews, as stipulated by both
accused during pretrial), and the other via judicial admission (pretrial stipulation). Considering
the foregoing circumstances, we deem it unnecessary to rule on the sufficiency of the “searching
inquiry” in this instance. Remanding the case for re-arraignment is not warranted, as the
accused’s plea of guilt was not the sole basis of the condemnatory judgment under consideration.

ABS-CBN CORP. VS GOZON

TOPIC:
​SEC. 11. Suspension of arraignment.—Upon motion by the proper party, the arraignment
shall be suspended in the following cases: (a) The accused appears to be suffering from an
unsound mental condition which effectively renders him unable to fully understand the charge
against him and to plead intelligently thereto. In such case, the court shall order his mental
examination and, if necessary, his confinement for such purpose; (b) There exists a prejudicial
question; and (c) A petition for review of the resolution of the prosecutor is pending at either the
Department of Justice, or the Office of the President; provided that the period of suspension shall
not exceed sixty (60) days counted from the filing of the petition with the reviewing office.

DOCTRINE:
​The rule therefore in this jurisdiction is that once a complaint or information is filed in
Court any disposition of the case as to its dismissal or the conviction or acquittal of the accused
rests in the sound discretion of the Court. Although the fiscal retains the direction and control of
the prosecution of criminal cases even while the case is already in Court he cannot impose his
opinion on the trial court. The Court is the best and sole judge on what to do with the case before
it. The determination of the case is within its exclusive jurisdiction and competence. A motion to
dismiss the case filed by the fiscal should be addressed to the Court who has the option to grant
or deny the same. It does not matter if this is done before or after the arraignment of the accused
or that the motion was filed after a reinvestigation or upon instructions of the Secretary of Justice
who reviewed the records of the investigation.

FACTS:
​The controversy arose from GMA-7’s news coverage on the homecoming of Filipino
overseas worker and hostage victim Angelo dela Cruz.
​As summarized by the CA: Overseas Filipino worker Angelo dela Cruz was kidnapped
by Iraqi militants and as a condition for his release, a demand was made for the withdrawal of
Filipino troops in Iraq. After negotiations, he was released by his captors and was scheduled to
return to the country in the afternoon of 22 July 2004. Occasioned by said homecoming and the
public interest it generated, both GMA Network, Inc. and petitioner made their respective
broadcasts and coverage of the live event.
​Allegedly, GMA-7 did not receive any notice or was not aware that Reuters was airing
footages of ABS-CBN. GMA-7’s news control room staff saw neither the “No Access
Philippines” notice nor a notice that the video feed was under embargo in favor of ABS-CBN.
​ABS-CBN filed the Complaint for copyright infringement against Dela Pena-Reyes and
Manalastas to which the prosecutor found probable cause to indict the latter. Respondents filed
the Petition for Review before the Department of Justice. DOJ Secretary Raul Gonzalez ruled in
favor of respondents and held that good faith may be raised as a defense in the case. Both parties
moved for reconsideration of the Gonzalez Resolution. The trial court granted the Motion to
Suspend Proceedings filed earlier by Dela Peña-Reyes and Manalastas. The Motion to Suspend
was granted.

ISSUE:
​Whether or not the Trial Court is correct in granting the Motion to Suspend.

HELD:
​The trial court should have proceeded with respondents Dela Peña-Reyes and Manalastas’
arraignment after the 60-day period from the filing of the Petition for Review before the
Department of Justice.
​The trial court erred when it did not act on the criminal case during the interim period. It
had full control and direction of the case.
​The suspension of the arraignment should always be within the limits allowed by law.
The doctrine in Crespo v. Judge Mogul was reiterated in Mayor Balindong v. Court of Appeals
where this court reminded the Department of Justice Secretary to refrain from entertaining
petitions for review when the case is already pending with this court: In order to avoid a situation
where the opinion of the Secretary of Justice who reviewed the action of the fiscal may be
disregarded by the trial court, the Secretary of Justice should, as far as practicable, refrain from
entertaining a petition for review or appeal from the action of the fiscal, when the complaint or
information has already been filed in the Court. The matter should be left entirely for the
determination of the Court. The trial court should have proceeded with respondents Dela Peña-
Reyes and Manalastas’ arraignment after the 60-day period from the filing of the Petition for
Review before the Department of Justice on March 8, 2005. It was only on September 13, 2010
that the temporary restraining order was issued by the Court of Appeals. The trial court erred
when it did not act on the criminal case during the interim period. It had full control and
direction of the case. As Judge Mogul reasoned in denying the motion to dismiss in Crespo,
failure to proceed with the arraignment “disregards the requirements of due process and erodes
the Court’s independence and integrity.”

ENRILE VS PEOPLE

DOCTRINE:
​The grant or denial of a motion for bill of particulars is discretionary on the court where
the Information is filed. As usual in matters of discretion, the ruling of the trial court will not be
reversed unless grave abuse of discretion or a manifestly erroneous order amounting to grave
abuse of discretion is shown.
​The rule requires the information to describe the offense with sufficient particularity to
apprise the accused of the crime charged with and to enable the court to pronounce judgment.
The particularity must be such that persons of ordinary intelligence may immediately know what
the Information means. The general function of a bill of particulars, whether in civil or criminal
proceedings, is to guard against surprises during trial. It is not the function of the bill to furnish
the accused with the evidence of the prosecution. Thus, the prosecutor shall not be required to
include in the bill of particulars matters of evidence relating to how the people intend to prove
the elements of the offense charged or how the people intend to prove any item of factual
information included in the bill of particulars.

FACTS:
​The Office of the Ombudsman filed an Information for plunder against Enrile, Napoles,
et. al before the Sandiganbayan. Enrile responded by filing before the Sandiganbayan (1) an
urgent omnibus motion (motion to dismiss for lack of evidence on record to establish probable
cause and ad cautelam motion for bail), and (2) a supplemental opposition to issuance of warrant
of arrest and for dismissal of Information,. The Sandiganbayan heard both motions. Thereafter,
the prosecution filed a consolidated opposition to both motions.
​The Sandiganbayan denied Enrile’s motions and ordered the issuance of warrants of
arrest on the plunder case against the accused. Soon after, Enrile received a notice of hearing
informing him that his arraignment would be held before the Sandiganbayan. The day before his
arraignment, Enrile filed a motion for bill of particular as well as a motion for deferment of
arraignment since he was to undergo medical examination at the PGH.
The Sandiganbayan denied his motion for Bill of Particulars on the following grounds:
42. the details that Enrile desires are “substantial reiterations” of the arguments he raised in his
supplemental opposition to the issuance of warrant of arrest and for dismissal of information;
and
43. the details sought are evidentiary in nature and are best ventilated during trial.
​Enrile then filed a Motion for Reconsideration, but the same was denied.Thus, he filed a
petition for certiorari before the High Court claiming that the Sandiganybayan gravely abused its
discretion in denying his motion for bill of particulars.

ISSUE:
​Whether or not the Sandiganbayan abused its discretion in denying Enrile’s motion for
bill of particulars;

HELD:
​YES. The grant or denial of a motion for bill of particulars is discretionary on the court
where the Information is filed. As usual in matters of discretion, the ruling of the trial court will
not be reversed unless grave abuse of discretion or a manifestly erroneous order amounting to
grave abuse of discretion is shown.
​In the light of all these considerations, we hold that the Sandiganbayan’s denial of the
petitioner’s motion for a bill of particulars, on the ground that the details sought to be itemized or
specified are all evidentiary without any explanation supporting this conclusion constitutes grave
abuse of discretion. Some of the desired details are material facts that must be alleged to enable
the petitioner to properly plead and prepare his defense. The Sandiganbayan should have
diligently sifted through each detail sought to be specified, and made the necessary determination
of whether each detail was an ultimate or evidentiary fact, particularly after Enrile stated in his
Reply that the “desired details” could not be found in the bundle of documents marked by the
prosecution. We cannot insist or speculate that he is feigning ignorance of the presence of these
desired details; neither can we put on him the burden of unearthing from these voluminous
documents what the desired details are. The remedy of a bill of particulars is precisely made
available by the Rules to enable an accused to positively respond and make an intelligent
defense.
​A bill of particulars guards against the taking of an accused by surprise by restricting the
scope of the proof; it limits the evidence to be presented by the parties to the matters alleged in
the Information as supplemented by the bill. It is for this reason that the failure of an accused to
move for a bill of particulars deprives him of the right to object to evidence which could be
lawfully introduced and admitted under an information of more or less general terms which
sufficiently charges the defendants with a definite crime.
Sec, 9 Rule 116, Bill of particulars.—The accused may, before arraignment, move for a bill of
particulars to enable him properly to plead and prepare for trial. The motion shall specify the
alleged defects of the complaint or information and the details desired.
In general, a bill of particulars is the further specification of the charges or claims in an action,
which an accused may avail of by motion before arraignment, to enable him to properly plead
and prepare for trial. In civil proceedings, a bill of particulars has been defined as a
complementary procedural document consisting of an amplification or more particularized
outline of a pleading, and is in the nature of a more specific allegation of the facts recited in the
pleading. The purpose of a motion for bill of particulars in civil cases is to enable a party to
prepare his responsive pleading properly. In criminal cases, a bill of particulars details items or
specific conduct not recited in the Information but nonetheless pertain to or are included in the
crime charged. Its purpose is to enable an accused: to know the theory of the government’s case;
to prepare his defense and to avoid surprise at the trial; to plead his acquittal or conviction in bar
of another prosecution for the same offense; and to compel the prosecution to observe certain
limitations in offering evidence.
​In dealing with a motion for a bill of particulars in a criminal case, judges should observe
that: (a) the remedy is distinct from a motion to quash in the sense that it presupposes that the
acts or offenses constituting the offense (or the ultimate facts that comprise the crime’s
component elements) are already stated in the Information, albeit may be couched in vague
language; (b) the remedy is, as mentioned, not meant to supply evidential information (or
evidentiary facts); and (c) the particulars to be allowed are only those details that would allow a
man of ordinary intelligence, as may be reasonable under the circumstances, to, again, properly
plead during his arraignment and to prepare his defense for trial. Accordingly, the analysis
involved in motion for bill of particulars should go beyond a simple ultimate facts-evidentiary
facts dichotomy.
​If the Information is lacking, a court should take a liberal attitude towards its granting and
order the government to file a bill of particulars elaborating on the charges. Doubts should be
resolved in favor of granting the bill to give full meaning to the accused’s Constitutionally
guaranteed rights. Notably, the government cannot put the accused in the position of disclosing
certain overt acts through the Information and withholding others subsequently discovered, all of
which it intends to prove at the trial. This is the type of surprise a bill of particulars is designed to
avoid. The accused is entitled to the observance of all the rules designated to bring about a fair
verdict. This becomes more relevant in the present case where the crime charged carries with it
the severe penalty of capital punishment and entails the commission of several predicate criminal
acts involving a great number of transactions spread over a considerable period of time.

RULE 117 – MOTION TO QUASH

PEOPLE VS LACSON

DOCTRINE:
​The interpretation of Sec. 8 of Rule 117 to the effect that in the event that the accused is
prosecuted anew with the same offense, albeit under an identical information, the new
proceedings being conducted as if the accused has been charged afresh has in its favor the
soundest policy considerations based no less on the fundamental objectives of procedural rules
Section 1 (g) of Rule 116—Unless a shorter period is provided by special law or Supreme Court
circular, the arraignment shall be held within thirty (30) days from the date the court acquires
jurisdiction over the person of the accused. The time of the pendency or a motion to quash or for
a bill of particulars or other causes justifying suspension of the arraignment shall be excluded in
computing the period.

FACTS:
​Before the Court is the petitioners’ Motion for Reconsideration of the Resolution dated
May 28, 2002, remanding this case to the Regional Trial Court (RTC) of Quezon City, Branch
81, for the determination of several factual issues relative to the application of Section 8 of Rule
117 on the dismissal of Criminal Cases Nos. Q-99-81679 to Q-99-81689 filed against the
respondent and his co-accused with the said court. The respondent and his co-accused were
charged with multiple murder for the shooting and killing of eleven persons bandied as members
of the Kuratong Baleleng Gang. The respondent opposed petitioners’ motion for reconsideration.
The Court ruled in the Resolution sought to be reconsidered that the provisional dismissal of
Criminal Cases Nos. Q-99- 81679 to Q-99-81689 were with the express consent of the
respondent as he himself moved for said provisional dismissal when he filed his motion for
judicial determination of probable cause and for examination of witnesses. The Court also held
therein that although Section 8, Rule 117 of the Revised Rules of Criminal Procedure could be
given retroactive effect, there is still a need to determine whether the requirements for its
application are attendant.

ISSUE:
​Whether or not the requirements for valid motion to quash were attendant
HELD:
​NO. Sec. 8. Provisional dismissal.—A case shall not be provisionally dismissed except
with the express consent of the accused and with notice to the offended party.
​The provisional dismissal of offenses punishable by imprisonment not exceeding six (6)
years or a fine of any amount, or both, shall become permanent one (1) year after issuance of the
order without the case having been revived. With respect to offenses punishable by imprisonment
of more than six (6) years, their provisional dismissal shall become permanent two (2) years after
issuance of the order without the case having been revived.
​Having invoked said rule before the petitioners-panel of prosecutors and before the Court
of Appeals, the respondent is burdened to establish the essential requisites of the first paragraph
thereof, namely:
44. The prosecution with the express conformity of the accused or the accused moves for a
provisional (sin perjuicio) dismissal of the case; or both the prosecution and the accused
move for a provisional dismissal of the case;
45. the offended party is notified of the motion for a provisional dismissal of the case;
46. the court issues an order granting the motion and dismissing the case provisionally;
47. the public prosecutor is served with a copy of the order of provisional dismissal of the case.
​The foregoing requirements are conditions sine qua non to the application of the time-bar
in the second paragraph of the new rule. The raison d’ etre for the requirement of the express
consent of the accused to a provisional dismissal of a criminal case is to bar him from
subsequently asserting that the revival of the criminal case will place him in double jeopardy for
the same offense or for an offense necessarily included therein.
​Although the second paragraph of the new rule states that the order of dismissal shall
become permanent one year after the issuance thereof without the case having been revived, the
provision should be construed to mean that the order of dismissal shall become permanent one
year after service of the order of dismissal on the public prosecutor who has control of the
prosecution6 without the criminal case having been revived. The public prosecutor cannot be
expected to comply with the timeline unless he is served with a copy of the order of dismissal.
​Express consent to a provisional dismissal is given either viva voce or in writing. It is a
positive, direct, unequivocal consent requiring no inference or implication to supply its meaning.
Where the accused writes on the motion of a prosecutor for a provisional dismissal of the case
No objection or With my conformity, the writing amounts to express consent of the accused to a
provisional dismissal of the case. The mere inaction or silence of the accused to a motion for a
provisional dismissal of the case or his failure to.
​A motion of the accused for a provisional dismissal of a case is an express consent to
such provisional dismissal. If a criminal case is provisionally dismissed with the express consent
of the accused, the case may be revived only within the periods provided in the new rule. On the
other hand, if a criminal case is provisionally dismissed without the express consent of the
accused or over his objection, the new rule would not apply. The case may be revived or refiled
even beyond the prescribed periods subject to the right of the accused to oppose the same on the
ground of double jeopardy or that such revival or refiling is barred by the statute of limitations.
​The case may be revived by the State within the time-bar either by the refiling of the
Information or by the filing of a new Information for the same offense or an offense necessarily
included therein. There would be no need of a new preliminary investigation. However, in a case
wherein after the provisional dismissal of a criminal case, the original witnesses of the
prosecution or some of them may have recanted their testimonies or may have died or may no
longer be available and new witnesses for the State have emerged, a new preliminary
investigation must be conducted before an Information is refiled or a new Information is filed.
​In this case, the respondent has failed to prove that the first and second requisites of the
first paragraph of the new rule were present when Judge Agnir, Jr. dismissed Criminal Cases
Nos. Q-99-81679 to Q-99-81689. Irrefragably, the prosecution did not file any motion for the
provisional dismissal of the said criminal cases. For his part, the respondent merely filed a
motion for judicial determination of probable cause and for examination of prosecution witnesses
alleging that under Article III, Section 2 of the Constitution and the decision of this Court in
Allado v. Diokno, among other cases, there was a need for the trial court to conduct a personal
determination of probable cause for the issuance of a warrant of arrest against respondent and to
have the prosecution’s witnesses summoned before the court for its examination. The respondent
contended therein that until after the trial court shall have personally determined the presence of
probable cause, no warrant of arrest should be issued against the respondent and if one had
already been issued, the warrant should be recalled by the trial court. He then prayed therein that:
48. a judicial determination of probable cause pursuant to Section 2, Article III of the
Constitution be conducted by this Honorable Court, and for this purpose, an order be issued
directing the prosecution to present the private complainants and their witnesses at a hearing
scheduled therefor; and
49. warrants for the arrest of the accused-movants be withheld, or, if issued, recalled in the
meantime until the resolution of this incident. Other equitable reliefs are also prayed for.
​The respondent did not pray for the dismissal, provisional or otherwise, of Criminal
Cases Nos. Q-99-81679 to Q-99-81689. Neither did he ever agree, impliedly or expressly, to a
mere provisional dismissal of the cases.

PANAGUITON VS DOJ

DOCTRINE:
​The prescriptive period is interrupted by the institution of proceedings for preliminary
investigation against the accused. Petitioner’s filing of his complaint-affidavit before the office
of the City Prosecutor on 24 August 1995 signified the commencement of the proceedings for
the prosecution of the accused and thus effectively interrupted the prescriptive period for the
offenses they had been charged under B.P. Blg. 22.

FACTS:
​Cawili borrowed money from petitioner. As payment for the loan, Cawili and Tongson
jointly issued three checks in favor of petitioner. The checks were dishonored, either for
insufficiency of funds or by closure of the account. Petitioner filed a complaint for violation of
BP 22. The prosecutor found probable cause only with respect to Cawili as Tongson’s defense
that his signatures on the checks had been falsified. Petitioner then filed a partial appeal before
the DOJ even while the case against Cawili was already filed in court. The Chief State
Prosecutor directed the city prosecutor to conduct a reinvestigation and to refer the falsified
document to the NBI. After reinvestigation, still only probable cause with respect to Cawili was
sustained. In the city prosecutor’s resolution, it was held that the case with respect to Tongson
had already prescribed pursuant to Act No. 3326 which provides that violations penalized by B.P.
Blg. 22 shall prescribe after four (4) years. n this case, the four (4)-year period started on the date
the checks were dishonored, or on 20 January 1993 and 18 March 1993. The filing of the
complaint before the Quezon City Prosecutor on 24 August 1995 did not interrupt the running of
the prescriptive period, as the law contemplates judicial, and not administrative proceedings.
Thus, considering that from 1993 to 1998, more than four (4) years had already elapsed and no
information had as yet been filed against Tongson, the alleged violation of B.P. Blg. 22 imputed
to him had already prescribed. Ultimately, the DOJ held that the action on the crime has
prescribed.
​In justifying its resolution, the DOJ explained that Act No. 3326 applies to violations of
special acts that do not provide for a prescriptive period for the offenses thereunder. Since B.P.
Blg. 22, as a special act, does not provide for the prescription of the offense it defines and
punishes, Act No. 3326 applies to it, and not Art. 90 of the Revised Penal Code which governs
the prescription of offenses penalized thereunder.

ISSUE:
​Whether or not the running of the prescriptive period tolled upon the filing of the
information in court or upon the filing of the complaint with the prosecutor for preliminary
investigation.

HELD:
​Act No. 3326 is the law applicable to offenses under special laws which do not provide
their own prescriptive periods. Act 3326 provides:
​“Sec. 2. Prescription shall begin to run from the day of the commission of the violation of
the law, and if the same be not known at the time, from the discovery thereof and the institution
of judicial proceedings for its investigation and punishment.
​The prescription shall be interrupted when proceedings are instituted against the guilty
person, and shall begin to run again if the proceedings are dismissed for reasons not constituting
jeopardy.”
​It must be pointed out that when Act No. 3326 was passed on 4 December 1926,
preliminary investigation of criminal offenses was conducted by justices of the peace, thus, the
phraseology in the law, "institution of judicial proceedings for its investigation and punishment,"
and the prevailing rule at the time was that once a complaint is filed with the justice of the peace
for preliminary investigation, the prescription of the offense is halted. “the term ‘proceedings’
should now be understood either executive or judicial in character”. To rule otherwise would
deprive the injured party the right to obtain vindication on account of delays that are not under
his control.

PEOPLE VS DUMLAO

DOCTRINE:
​Insufficiency of evidence is not one of the grounds of a Motion to Quash—insufficiency
of evidence is a ground for dismissal of an action only after the prosecution rests its case.

FACTS:
​An Amended Information was filed before the Sandiganbayan charging respondents
Dumlao and Lao, Aber P. Canlas, Jacobo C. Clave, Roman A. Cruz, Jr. and Fabian C. Ver with
violation of Section 3(g) of Republic Act No. 3019, as amended, otherwise known as the Anti-
Graft and Corrupt Practices Act. The accusatory portion of the information reads:
​“That on or about May 10, 1982, or for sometime prior or subsequent thereto, in Manila,
Philippines, and within the jurisdiction of this Honorable Court, the accused Hermenegildo C.
Dumlao, Aber Canlas, Jacobo C. Clave, Roman A. Cruz, Jr., and Fabian C. Ver, being then the
members of the Board of Trustees of the Government Service Insurance System (GSIS) which is
a government corporation and therefore all public officers, conspiring and confederating together
and mutually helping one another, while in the performance of their official functions, did then
and there willfully, unlawfully and criminally enter into contract of lease-purchase with Emilio
G. Lao, a private person whereby the GSIS agreed to sell to said Emilio G. Lao, a GSIS acquired
property consisting of three parcels of land with an area of 821 square meters together with a 5-
storey building situated at 1203 A. Mabini St., Ermita, Manila, known as the Government
Counsel Centre for the sum of P2,000,000.00 with a down payment of P200,000.00 with the
balance payable in fifteen years at 12% interest per annum compounded yearly, with a yearly
amortization of P264,278.37 including principal and interest granting Emilio G. Lao the right to
sub-lease the ground floor for his own account during the period of lease, from which he
collected yearly rentals in excess of the yearly amortization which contract is manifestly and
grossly disadvantageous to the government.”
​Respondent Dumlao, with the assistance of counsel de parte, pleaded not guilty to the
offense charged. A Joint Stipulation of Facts and Admission of Exhibits was submitted to the
court on 10 January 2005; after which the court issued Pre-Trial Order; Respondent Dumlao then
filed a Motion to Dismiss/Quash on the ground that the facts charged do not constitute an
offense. He argued that the allegedly approved Board Resolution was not in fact approved by the
GSIS Board of Trustees, contrary to the allegations in the information. Since the signatures of
Fabian Ver, Roman Cruz, Aber Canlas and Jacobo Clave did not appear in the minutes of the
meeting held on 23 April 1982, he said it was safe to conclude that these people did not
participate in the alleged approval of the Lease- Purchase Agreement. This being the case, he
maintained that there was no quorum of the board to approve the supposed resolution authorizing
the sale of the GSIS property. There being no approval by the majority of the Board of Trustees,
there can be no resolution approving the Lease-Purchase Agreement. The unapproved resolution,
he added, proved his innocence. He further contended that the person to be charged should be
Atty. Luis Javellana, who sold the subject property to respondent Lao without the proper
authority.Sandiganbayan ruled in favor of respondent; the People through the Office of the
Special Prosecutor, filed a petition for certiorari under Rule 45 seeking the reversal and setting
aside of the Sandiganbayan Resolution dismissing the case against respondent Dumlao, arguing
that it was denied its right to due process when the court a quo dismissed the case against
respondent Dumlao after pre-trial and before it could present its witnesses and formally offer its
exhibits.

ISSUE:
​Whether or not facts alleged, if hypothetically admitted, would establish the essentials
elements of the crime defined by law.

HELD:
​Insufficiency of evidence is not one of the grounds of a Motion to Quash. Insufficiency of
evidence is a ground for dismissal of an action only after the prosecution rests its case. Section
23, Rule 119 of the Revised Rules of Criminal Procedure provides:
​Sec. 23. Demurrer to evidence. After the prosecution rests its case, the court may
dismiss the action on the ground of insufficiency of evidence (1) on its own initiative after giving
the prosecution the opportunity to be heard or (2) upon demurrer to evidence filed by the accused
with or without leave of court.
​In the case under consideration, the Sandiganbayan dismissed the case against respondent
for insufficiency of evidence, even without giving the prosecution the opportunity to present its
evidence. In so doing, it violated the prosecutions right to due process. It deprived the
prosecution of its opportunity to prosecute its case and to prove the culpability of the accused.
It was therefore erroneous for the Sandiganbayan to dismiss the case under the premises. Not
only did it not consider the ground invoked by respondent Dumlao; it even dismissed the case on
a ground not raised by him, and not at the appropriate time. The dismissal was thus without basis
and untimely.

SORIANO VS PEOPLE

DOCTRINE:
​The fundamental test in considering a motion to quash anchored on Section 3 (a),19 Rule
117 of the1985 Rules on Criminal Procedure, is the sufficiency of the averments in the
information; that is, whether the facts alleged, if hypothetically admitted, would establish the
essential elements of the offense charged as defined by law.20 The trial court may not consider a
situation contrary to that set forth in the criminal complaint or information. Facts that constitute
the defense of the petitioners against the charge under the information must be proved by them
during trial. Such facts or circumstances do not constitute proper grounds for a motion to quash
the information on the ground that the material averments do not constitute the offense.

FACTS:
​State Prosecutor Josefino A. Subia charged Soriano in the Regional Trial Court (RTC) of
Malolos, Bulacan, with violation of Section 83 of Republic Act No. 337 (R.A. No. 337) or the
General Banking Act, as amended by Presidential Decree No. 1795, or Violation of the Director,
Officer, Stockholder or Related Interest (DOSRI) Rules (DOSRI Rules)
​An information for estafa thru falsification of commercial document was also filed
against Soriano and Ilagan. The informations were docketed as Criminal Case Nos. 1719-M-
2000 and 1720-M-2000, respectively, and were raffled to Branch 14, presided by Judge Petrita
Braga Dime.
​Another information for violation of Section 83 of R.A. No. 337, as amended, was filed
against Soriano, this time, covering the P15,000,000.00 loan obtained in the name of Rogelio
Mañaol.
​Soriano and Ilagan were also indicted for estafa thru falsification of commercial
document for obtaining said loan. The cases were docketed as 1980-M-2000 and 1981-M-2000,
respectively, and were raffled to Branch 77, presided by Judge Aurora Santiago-Lagman.
​Petitioners Soriano and Ilagan filed an MQ before both salas. Petitioners argued that the
prosecutor charged more than one offense for a single act. Soriano was charged with violation of
DOSRI rules and estafa thru falsification of commercial document for allegedly securing
fictitious loans. They further argued that the facts as alleged in the information do not constitute
an offense. Both salas of RTC denied the MQs Petitioners filed a Rule 65 before the CA but it
was dismissed. Petitioners filed a Rule 45 before the SC
ISSUE:
​Whether the 2 judges correctly denied the Motion to Quash

HELD:
​YES. Petitioners assail the validity of the informations against them on the ground that
more than one (1) offense is charged. They point that Soriano was charged with violation of
DOSRI Rules and with estafa thru falsification of commercial document for allegedly obtaining
loans from RBSM. Thus, they claim that the informations were duplicitous; hence, they should
be quashed.
​Indisputably, duplicity of offenses in a single information is a ground to quash the
Information under Section 3(e), Rule 11713 of the 1985 Rules of Criminal Procedure. The Rules
prohibit the filing of a duplicitous information to avoid confusing the accused in preparing his
defense.
​By duplicity of charges is meant a single complaint or information that charges more than
one offense.15 Section 13 of Rule 110 of the 1985 Rules on Criminal Procedure clearly states:
Duplicity of Offense. – A complaint or information must charge but one offense, except only in
those cases in which existing laws prescribe a single punishment for various offenses.
Otherwise stated, there is duplicity (or multiplicity) of charges when a single Information
charges more than one offense.
​In this case, however, Soriano was faced not with one information charging more than
one offense, but with more than one information, each charging a different offense - violation of
DOSRI rules in one, and estafa thru falsification of commercial documents in the others. Ilagan,
on the other hand, was charged with estafa thru falsification of commercial documents in
separate informations. Thus, petitioners erroneously invoke duplicity of charges as a ground to
quash the Informations.
​Petitioners also contend that Soriano should be charged with one offense only, because all
the charges filed against him proceed from and are based on a single act of obtaining fictitious
loans. Thus, Soriano argues that he cannot be charged with estafa thru falsification of
commercial document, considering that he is already being prosecuted for obtaining a DOSRI
loan. The contention has no merit. Jurisprudence teems with pronouncements that a single act or
incident might offend two or more entirely distinct and unrelated provisions of law, thus
justifying the filing of several charges against the accused.
​In Loney v. People, this Court, in upholding the filing of multiple charges against the
accused, held: As early as the start of the last century, this Court had ruled that a single act or
incident might offend against two or more entirely distinct and unrelated provisions of law thus
justifying the prosecution of the accused for more than one offense. The only limit to this rule is
the Constitutional prohibition that no person shall be twice put in jeopardy of punishment for
"the same offense." In People v. Doriquez, we held that two (or more) offenses arising from the
same act are not "the same" —
​x x x if one provision [of law] requires proof of an additional fact or element which the
other does not, x x x. Phrased elsewise, where two different laws (or articles of the same code)
define two crimes, prior jeopardy as to one of them is no obstacle to a prosecution of the other,
although both offenses arise from the same facts, if each crime involves some important act
which is not an essential element of the other.
​Consequently, the filing of the multiple charges against petitioners, although based on the
same incident, is consistent with settled doctrine.
​As aptly pointed out by the BSP in its memorandum, there are differences between the
two (2) offenses. A DOSRI violation consists in the failure to observe and comply with
procedural, reportorial or ceiling requirements prescribed by law in the grant of a loan to a
director, officer, stockholder and other related interests in the bank, i.e. lack of written approval
of the majority of the directors of the bank and failure to enter such approval into corporate
records and to transmit a copy thereof to the BSP supervising department. The elements of abuse
of confidence, deceit, fraud or false pretenses, and damage, which are essential to the prosecution
for estafa, are not elements of a DOSRI violation. The filing of several charges against Soriano
was, therefore, proper.
​Petitioners next question the sufficiency of the allegations in the informations, contending
that the same do not constitute an offense.
​The fundamental test in considering a motion to quash anchored on Section 3 (a), Rule
117 of the1985 Rules on Criminal Procedure, is the sufficiency of the averments in the
information; that is, whether the facts alleged, if hypothetically admitted, would establish the
essential elements of the offense charged as defined by law. The trial court may not consider a
situation contrary to that set forth in the criminal complaint or information. Facts that constitute
the defense of the petitioners against the charge under the information must be proved by them
during trial. Such facts or circumstances do not constitute proper grounds for a motion to quash
the information on the ground that the material averments do not constitute the offense.
​We have reviewed the informations and find that they contain material allegations
charging Soriano with violation of DOSRI rules and estafa thru falsification of commercial
documents.
​In Criminal Case Nos. 1719 & 1980 for violation of DOSRI rules, the informations
alleged that Soriano was the president of RBSMI, while Ilagan was then its general manager; that
during their tenure, Soriano, with the direct participation of Ilagan, and by using the names of
Virgilio Malang and Rogelio Mañaol, was able to indirectly obtain loans without complying with
the requisite board approval, reportorial and ceiling requirements, in violation of Section 83 of
R.A. No. 37722 as amended.
​Similarly, the informations in Criminal Case Nos. 1720 & 1981 charge petitioners with
estafa thru falsification of commercial document. They allege that petitioners made it appear that
Virgilio J. Malang and Rogelio Mañaol obtained loans and received the proceeds thereof when
they did not in fact secure said loans or receive the amounts reflected in the promissory notes and
other bank records.
​The information in Criminal Case No. 1720 further alleges the elements of estafa under
Article 315 (1)(b)23 of the RPC to wit: (i) that money, goods or other personal property be
received by the offender in trust, or on commission, or for administration, or under any other
obligation involving the duty to make delivery of or to return the same; (ii) that there be
misappropriation or conversion of such money or property by the offender, or denial on his part
of such receipt; (iii) that such misappropriation or conversion or denial is to the prejudice of
another; and (iv) that there is demand made by the offended party to the offender.
​The information in Criminal Case No. 1981, on the other hand, further alleged the
following essential elements of estafa under Article 315 (2) (a)24 of the RPC: (i) that there must
be a false pretense, fraudulent act or fraudulent means; (ii) that such false pretense, fraudulent act
or fraudulent means must be made or executed prior to or simultaneously with the commission of
the fraud; (iii) that the offended party must have relied on the false pretense, fraudulent act, or
fraudulent means—that is, he was induced to part with his money or property because of the
false pretense, fraudulent act, or fraudulent means; and (iv) that, as a result thereof, the offended
party suffered damage. The informations in Criminal Case Nos. 1720 & 1981, thus, charge
petitioners with the complex crime of estafa thru falsification of commercial documents.
​Verily, there is no justification for the quashal of the Information filed against petitioners.
The RTC committed no grave abuse of discretion in denying the motions.
​In fine, the Court has consistently held that a special civil action for certiorari is not the
proper remedy to assail the denial of a motion to quash an information. The proper procedure in
such a case is for the accused to enter a plea, go to trial without prejudice on his part to present
the special defenses he had invoked in his motion to quash and if after trial on the merits, an
adverse decision is rendered, to appeal therefrom in the manner authorized by law.25 Thus,
petitioners should not have forthwith filed a special civil action for certiorari with the CA and
instead, they should have gone to trial and reiterated the special defenses contained in their
motion to quash. There are no special or exceptional circumstances in the present case that would
justify immediate resort to a filing of a petition for certiorari. Clearly, the CA did not commit any
reversible error, much less, grave abuse of discretion in dismissing the petition.

CEREZO VS PEOPLE

DOCTRINE:
​Well-entrenched is the rule that once a case is filed with the court, any disposition of it
rests on the sound discretion of the court. In thus resolving a motion to dismiss a case or to
withdraw an Information, the trial court should not rely solely and merely on the findings of the
public prosecutor or the Secretary of Justice. It is the court’s bounden duty to assess
independently the merits of the motion, and this assessment must be embodied in a written order
disposing of the motion. While the recommendation of the prosecutor or the ruling of the
Secretary of Justice is persuasive, it is not binding on courts.

FACTS:
​In 2002, Joseph Cerezo filed a complaint for libel against Juliet Yaneza, Pablo Abunda,
Jr., and Vicente Afulugencia, as well as Oscar Mapalo (Mapalo). Finding probable cause to indict
Yaneza, et al., the Quezon City Prosecutor’s Office (OP-QC) filed the corresponding Information
against them before the RTC. Yaneza, et al. thereafter filed a Motion for Reconsideration and/or
Motion to Re-evaluate Prosecution’s Evidence before the OP-QC. In its resolution, the OP-QC
reversed its earlier finding and recommended the withdrawal of the Information. Consequently, a
Motion to Dismiss and Withdraw Information was filed before the RTC.During the intervening
period, Yaneza, et al. were arraigned. All of them entered a "not guilty" plea. In deference to the
prosecutor’s last resolution, the RTC ordered the criminal case dismissed. Aggrieved, Cerezo
moved for reconsideration of the said Order, arguing that the OP-QC resolution has not yet
attained finality, considering that the same was the subject of a Petition for Review filed before
the Department of Justice (DOJ). The RTC deferred action on the said motion to await the
resolution of the DOJ. In 2006, the Secretary of Justice promulgated his resolution reversing and
setting aside the OP-QC’s resolution, and directing the latter to refile the earlier Information for
libel. The RTC issued its Order granting Cerezo's motion for reconsideration, conformably with
the resolution of the DOJ Secretary. Yaneza, et al. moved for reconsideration, but the motion was
denied by the RTC. Relentless, Yaneza, et al. elevated their predicament to the CA through a
Petition for Certiorari under Rule 65 of the Rules of Court. The CA annulled the impugned RTC
Orders. Cerezo interposed the appeal when his motion for reconsideration of the CA Decision
was denied.

ISSUES:
50. Whether the dismissal of the case by the RTC was valid.
51. Whether double jeopardy set in after the RTC's dismissal of the case.
HELD:
52. No, because there was independent assessment of the merits of the motion on the part of the
RTC.
53. No. The elements of double jeopardy are not attendant in the present case.
​Well-entrenched is the rule that once a case is filed with the court, any disposition of it
rests on the sound discretion of the court. In thus resolving a motion to dismiss a case or to
withdraw an Information, the trial court should not rely solely and merely on the findings of the
public prosecutor or the Secretary of Justice. It is the court’s bounden duty to assess
independently the merits of the motion, and this assessment must be embodied in a written order
disposing of the motion. While the recommendation of the prosecutor or the ruling of the
Secretary of Justice is persuasive, it is not binding on courts.
​By relying solely on the manifestation of the public prosecutor and the resolution of the
DOJ Secretary, the trial court abdicated its judicial power and refused to perform a positive duty
enjoined by law. The said Orders were thus stained with grave abuse of discretion and violated
the complainant’s right to due process. They were void, had no legal standing, and produced no
effect whatsoever.
​This Court must therefore remand the case to the RTC, so that the latter can rule on the
merits of the case to determine if a prima facie case exists and consequently resolve the Motion
to Dismiss and Withdraw Information anew.
​It is beyond cavil that double jeopardy did not set in. Double jeopardy exists when the
following requisites are present: (1) a first jeopardy attached prior to the second; (2) the first
jeopardy has been validly terminated; and (3) a second jeopardy is for the same offense as in the
first. A first jeopardy attaches only (a) after a valid indictment; (b) before a competent court; (c)
after arraignment; (d) when a valid plea has been entered; and (e) when the accused has been
acquitted or convicted, or the case dismissed or otherwise terminated without his express
consent.
​Since we have held that the Order granting the motion to dismiss was committed with
grave abuse of discretion, then respondents were not acquitted nor was there a valid and legal
dismissal or termination of the case. Ergo, the fifth requisite which requires the conviction and
acquittal of the accused, or the dismissal of the case without the approval of the accused, was not
met. Thus, double jeopardy has not set in.

CO VS NEW PROSPERITY PLASTIC PRODUCTS

DOCTRINE:

FACTS:
​Respondent New Prosperity Plastic Products, represented by Elizabeth Uy (Uy), is the
private complainant in Criminal Case Nos. 206655-59, 206661-77 and 209634 for Violation of
Batas Pambansa (B.P.) Bilang 22 filed against petitioner William Co (Co), which were raffled to
the MeTC Branch. 49 of Caloocan City.
​In the absence of Uy and the private counsel, the cases were provisionally dismissed on
June 9, 2003 in open court pursuant to Section 8, Rule 117 of the Revised Rules of Criminal
Procedure (Rules).
​On July 2, 2004, Uy, through counsel, filed a Motion to Revive the Criminal Cases.9
Hon. Belen B. Ortiz, then Presiding Judge of the MeTC Branch 49, granted the motion on
October 14, 2004 and denied Co’s motion for reconsideration.
​On March 17, 2005, Co filed a petition for certiorari and prohibition with prayer for the
issuance of a temporary restraining order (TRO)/writ of preliminary injunction (WPI) before the
RTC of Caloocan City challenging the revival of the criminal cases.
​It was, however, dismissed for lack of merit on May 23, 2005. Co’s motion for
reconsideration was, subsequently, denied on December 16, 2005. Co then filed a petition for
review on certiorari under Rule 45 before the Supreme Court, which was docketed as G.R. No.
171096. We dismissed the petition per Resolution dated February 13, 2006.There being no
motion for reconsideration filed, the dismissal became final and executory on March 20, 2006.
​Before the MeTC Branch 50 where Criminal Case Nos. 206655-59, 206661-77 and
209634 were re-raffled after the inhibition of Judge Ortiz, Co filed a “Motion for Permanent
Dismissal” on July 13, 2006. Uy opposed the motion, contending that the motion raised the same
issues already resolved with finality by this Court in G.R. No. 171096. In spite of this, Judge
Esteban V. Gonzaga issued an Order dated September 4, 2006 granting Co’s motion. When the
court subsequently denied Uy’s motion for reconsideration on November 16, 2006, Uy filed a
petition for certiorari before the RTC of Caloocan City. On January 28, 2008, Hon. Judge
Adoracion G. Angeles of the RTC Branch 121 acted favorably on the petition, annulling and
setting aside the Orders dated September 4, 2006 and November 16, 2006 and directing the
MeTC Branch 50 to proceed with the trial of the criminal cases.Co then filed a petition for
certiorari before the CA, which, as aforesaid, dismissed the petition and denied his motion for
reconsideration. Hence, this present petition with prayer for TRO/WPI.
​Petitioner’s contention: Co argues that the June 9, 2003 Order provisionally dismissing
Criminal Case Nos. 206655-59, 206661-77 and 209634 should be considered as a final dismissal
on the ground that his right to speedy trial was denied. He reasons out that from his arraignment
on March 4, 2002 until the initial trial on June 9, 2003, there was already a “vexatious,
capricious and oppressive” delay, which is in violation of Section 6 of Republic Act 8493
(Speedy Trial Act of 1998) and Section 2, Paragraph 2, Rule 119 of the Revised Rules of
Criminal Procedure mandating that the entire trial period should not exceed 180 days from the
first day of trial. As the dismissal is deemed final, Co contends that the MeTC lost its jurisdiction
over the cases and cannot reacquire jurisdiction over the same based on a mere motion because
its revival would already put him in double jeopardy.

ISSUE:
​Whether or not the dismissal of the criminal cases against petitioner on the ground of
denial of his right to speedy trial constitutes final dismissal of these cases;

HELD:
​The petition is unmeritorious. Co’s charge that his right to a speedy trial was violated is
baseless. Obviously, he failed to show any evidence that the alleged “vexatious, capricious and
oppressive” delay in the trial was attended with malice or that the same was made without good
cause or justifiable motive on the part of the prosecution. This Court has emphasized that
“‘speedy trial’ is a relative term and While the Court recognizes the accused's right to speedy
trial and adheres to a policy of speedy administration of justice, we cannot deprive the State of a
reasonable opportunity to fairly prosecute criminals. Unjustified postponements which prolong
the trial for an unreasonable length of time are what offend the right of the accused to speedy
trial. Necessarily a flexible concept.”
​In determining whether the accused's right to speedy trial was violated, the delay should
be considered in view of the entirety of the proceedings.The factors to balance are the following:
(a) duration of the delay; (b) reason therefor; (c) assertion of the right or failure to assert it; and
(d) prejudice caused by such delay. Surely, mere mathematical reckoning of the time involved
would not suffice as the realities of everyday life must be regarded in judicial proceedings which,
after all, do not exist in a vacuum, and that particular regard must be given to the facts and
circumstances peculiar to each case. “While the Court recognizes the accused's right to speedy
trial and adheres to a policy of speedy administration of justice, we cannot deprive the State of a
reasonable opportunity to fairly prosecute criminals. Unjustified postponements which prolong
the trial for an unreasonable length of time are what offend the right of the accused to speedy
trial.”

PEOPLE VS DE LEON

DOCTRINE:
​A Buy-bust operation is legal and has been proved to be an effective method of
apprehending drug peddlers, provided due regard to constitutional and legal safeguards is
undertaken.

FACTS:
​Rodante de Leon was charged with violation of RA 9165 (Comprehensive Dangerous
Drugs Act of 2002).
​A buy-bust team was formed to act on the illegal activities of De Leon upon the report of
a confidential informant at the Anti- Illegal Drug Special Operation Task Force at the Novaliches
Police Station in Quezon City. De Leon was arrested upon selling to PO2 Magcalayo, posing as
buyer, shabu for P200. The seized drug was identified as a prohibited drug and subsequently
presented in evidence.
​The RTC and CA both found De Leon guilty of the crime charged.
ISSUE:
​Whether or not the buy-bust operation, allegedly full of irregularities was legal.

HELD:
​YES. A buy-bust operation is a form of entrapment whereby ways and means are resorted
to for the purpose of trapping and capturing the lawbreakers in the execution of their criminal
plan. In this jurisdiction, the operation is legal and has been proved to be an effective method of
apprehending drug peddlers, provided due regard to constitutional and legal safeguards is
undertaken.
​The Court upheld the presumption of regularity in the performance of official duties and
ruled that the elements of the crime were sufficiently established, thereby affirming the decision
of the CA
​115 of 115

Vous aimerez peut-être aussi